首页 英语真题

英语真题

举报
开通vip

英语真题英语真题 2001-2013年江苏专转本英语历年真题 【本资料的参考答案附在每一套真题的末尾注意查看】 目录 2001年江苏省普通高校“专转本”统一考试试卷 大学英语 ........ 2 2002年江苏省普通高校“专转本”统一考试试卷 大学英语 ...... 11 2003年江苏省普通高校“专转本”统一考试试卷 大学英语 ...... 22 2004年江苏省普通高校“专转本”统一考试试卷 大学英语 ...... 33 2005年江苏省普通高校“专转本”统一考试试卷 大学英语 ...... 44 2006年江...

英语真题
英语真题 2001-2013年江苏专转本英语历年真题 【本资料的参考答案附在每一套真题的末尾注意查看】 目录 2001年江苏省普通高校“专转本”统一考试试卷 大学英语 ........ 2 2002年江苏省普通高校“专转本”统一考试试卷 大学英语 ...... 11 2003年江苏省普通高校“专转本”统一考试试卷 大学英语 ...... 22 2004年江苏省普通高校“专转本”统一考试试卷 大学英语 ...... 33 2005年江苏省普通高校“专转本”统一考试试卷 大学英语 ...... 44 2006年江苏省普通高校“专转本”统一考试试卷 大学英语 ...... 54 2007年江苏省普通高校“专转本”统一考试试卷 大学英语 ...... 64 2008年江苏省普通高校“专转本”统一考试试卷 大学英语 ...... 76 2009年江苏省普通高校“专转本”统一考试试卷 大学英语 ...... 89 2010年江苏省普通高校“专转本”统一考试试卷 大学英语 .... 103 2011年江苏省普通高校“专转本”统一考试试卷 大学英语 .... 121 2012年江苏省普通高校“专转本”统一考试试卷 大学英语 .... 137 2013年江苏省普通高校“专转本”统一考试试卷 大学英语 .... 148 (本资料由于制作是将仓促,错误在所难免,敬请读者批评指正) 资料工本费:20元 2001年江苏省普通高校“专转本”统一考试试卷 大学英语 第?卷(共100分) 注意事项: 1(答第?卷前,考生务必将自己的姓名、准考证号、考试科目用铅笔涂写在答题卡上。 2(每小题选出答案后,用2B铅笔把答题卡上对应题目的答案标号涂黑。如需改动,用橡皮擦干净后, 再选涂其他答案标号。如果答案不涂写在答题卡上,成绩无效。 Part I Reading Comprehension(40,,40minutes) Directions: In this part there are four passages. Each passage is followed by a number of comprehension questions. Read the passage and answer the questions. Then mark your answer by blackening the corresponding letter on the Answer Sheet. Passage 1 Questions l to 5 are based on the following passage. Do you want to say what you think in a letter to the President of the United States? You'll get a reply from him-written in ink, not typed-after only a few days. The President gets about 4,000 letters every week. He answers everyone who writes to him on special White House Paper. But he doesn't need a lot of time for it. In fact, he only gives 20 minutes a week to looking at his personal correspondence. He has the most modern secretary in world to help him. His computer, worth $800,000; which has its own room on the first floor of the White House. It has a bank of electronic pens which write like the President writes, in his favorite light blue ink. Each letter the President receives gets a number, according to the type of answer it needs. The pens then write the correct reply for it, according to the number. Each letter takes less than a second to write. A White House official said, "It's not important that the letters come from a computer. Each letter says what the President wants to say. 1. ____for a reply from the President. A. You have to wait a long time B. You have to wait at least one month C. You only have to wait several days D. You only have to wait a few weeks 2. The reply from the President ____. A. is always printed B. is always written in ink C. is always typed D. is always written by himself 3. It takes the computer ____to write ten letters. A. no more than ten seconds B. less than ten seconds C. a little more than ten seconds D. at least one second 4. The computer can be described as ____. A. expensive but efficient B. heavy and inefficient C. possessing a beautiful handwriting D. the President's most reliable secretary 5. It can be inferred from the passage that ____. A. the President never reads any letters written to him by ordinary people B. the President hires a very efficient secretary to deal with his correspondence C. the President does not really care about the letters he receives every week D. the President is assured that the computer expresses his views in the letters Passage 2 Questions 6 to 10 are based on the following passage. In order to learn to be one's true self, it is necessary to obtain a wide and extensive knowledge of what has 2 been said and done in the world, critically to inquire into it; carefully to consider it; clearly to analyze it; and earnestly to carry it out. It matters not what you learn, but when you once leam a thing, you must never give it up until you have mastered it. It matters not what you inquire into, but when you once inquire into a thing, you must never give it up until you have thoroughly understood it. It matters not what you try to think of, but when you once try to think of a thing, you must never give it up until you have got what you want. It matters not what you try to carry out, but when you once try to carry out a thing, you must never giveit up until you have done it thoroughly and well. If another man succeeds by one effort, you will use a hundred efforts. If another man succeeds by ten efforts, you will use a thousand. 6. According to the author, first of all one must ____. A. analyze B. inquire C. obtain knowledge D. act 7. According to the author, ____. A. learning is not important B. knowledge means little C. thinking is not necessary D. it is not important what we learn 8. The end of learning should be ____. A. thought B. mastery C. inquiry D. analysis 9. According to the author, another man's succeeds should ____. A. make greater efforts B. not be taken into consideration C. make us nervous D. cause one to stop trying 10. The author implies but does not say that ____. A. the way to knowledge is through specialization B. one has to know everything to be successful C. success depends not so much on natural ability as it does on effort D. success in one's profession is least important in one's life Passage 3 Questions l1 to 15 are based on the following passage. About 70 million Americans are trying to lose weight. That is almost l out of every 3 people in the United States. Some people go on diets. This means they eat less of certain foods, especially fats and sugars. Other people exercise with special equipment, take diet pills, or even have surgery. Losing weight is hard work, and it can also cost a lot of money. So why do so many people in the United States want to lose weight? Many people in the United States worry about not looking young and attractive. For many people, looking good also means being thin. Other people worry about their health. Many doctors say being overweight is not healthy. But are Americans really fat? Almost 30 million Americans weigh at least 20 percent more than their ideal weight. In fact, the United States is the most overweight country in the world. "The stored fat of adult Americans weighs 2.3 trillion pounds," says University of Massachusetts anthropologist(人类学家) George Americans. He says burning off that stored energy would produce enough power for 900,000 cars to go 12,000 miles. Losing weight is hard work, but most people want to find a fast and easy way to take off fat. Bookstores sell lots of diet books. These books tell readers how to lose weight. Each year, dozens of new hooks like these are written. Each one boasts to help people to get rid of fat. 11. Which of the following is NOT mentioned as a way of losing weight? A. To eat less fats and sugars. B. To take much exercise. C. To have surgery. D. To work hard. 12. Many Americans are trying to lose weight because ____. 3 A. they want to look attractive B. they want to keep fit C. they are misled by doctors D. both A and B 13. The figures given in the third paragraph suggest that ____. A. Americans are dependent on cars B. cars consume a lot of money C. Americans need losing weight D. excess of fat can be source of energy 14. It can be inferred from the last paragraph that ____. A. diet hooks are not always effective B. diet books are usually helpful C. there are lots of ways of losing weight D. bookstores are keeping their promises 15. It can be concluded from the passage that ____. A. people think too much of their appearance B. there is not a sure way of losing weight as yet C. surgery is the fastest way of losing weight D. going on diet is a safe way of losing weight Passage 4 Questions 16 to 20 are based on the following passage. I recently wrote an autobiography in which I recalled many old memories. One of them was from my school days, when our ninth grade teacher, Miss Raber, would pick out words from Readers Digest to test our vocabulary. Today, more than 45 years later, I always check out "It pays to Enrich Your Word Power" first when the Digest comes each month. I am impressed with that idea, word power. Readers Digest knows the power that words have to move people to entertain, inform and inspire. The Digest editors know that the big word isn't always the best word. Take just one example, a Quotable Quote from the February 1985 issue: "Time is a playful thing. It slips quickly and drinks the day like a bowl of milk. " Seventeen words, only two of them more than one syllable, yet how much they convey ! That's usually how it is with Reader s Digest. The small and simple can be profound. As chairman of a foundation to restore the Statue of Liberty, I've been making a lot of speeches lately. I try to keep them fairly short. I use small but vivid words: words like "hoe" , "guts", "faith" and "dreams". Those are words that move people and say so much about the spirit of America. Don't get me wrong. I'm not against using big words, when it is right to do so, but I have also learned that a small word can work a small miracle-if it's the right word, in the right place, at the right time. It's a"secret" that I hope I will never forget. 16. The passage is mainly about ____. A. one of the many old memories B. using simple words to express profound ideas C. Readers Digest and school speeches D. how to make effective speeches 17. It seems that Reader's Digest is a magazine popular with ____. A. people of all ages B. teenagers C. school teachers D. elderly readers 18. The example the author gives in the second paragraph might mean ____. A. one spends his day playing and drinking B. don't waste your time as one does C. time slips easily if you don't make good use of it D. time is just like drinking milk from a bowl 19. The author's "secret" is ____. A. to avoid using big words at any time B. to use words that have the power to move people C. to work a miracle by using a small word D. to use small and simple words where possible 4 20. According to the author, well-chosen words can give people ____. A. hope, courage, and ideas B. confidence, determination, and strength C. pleasure, knowledge, and encouragement D. entertainment, information, and power Part ? Vocabulary and Structure(20,,25minutes) Directions: There are 40 incomplete sentences in this part. For each sentence there are four choices marked A, B, C and D. Choose the ONE answer that best completes the sentence. Then mark the corresponding letter on the Answer Sheet with a single line through the center. 21. It's still early in the morning. There isn't ____in the office. A. anyone B. everyone C. nobody D. any people 22. ____is not known what they discussed in the meeting. A. That B. He C. This D. It 23. The sad news broke her ____and she has been gloomy ever since. A. feelings B. emotions C. mind D. heart 24. He is ____much of a gentleman to fight. A. so B. as C. very D. too 25. Not until this term ____to realize how important this subject is to his future career as a diplomat. A. he began B. did he begin C. he has begun D. that he has begun 26. ____who would like to go on the trip should put their names on the list. A. Those B. These C. Somebody D. The ones 27. A bottle weights less after air is taken out, ____proves that air has weight. A. we B. it C. which D. what 28. How long ____? A. you suppose did it last B. do you suppose it last C. did you suppose it last D. you suppose it last 29. Smith had some trouble ____the man's accent. A. to understand B. understanding C. for understanding D. with understanding 30. The next few days could be ____for the peace negotiation. A. maximum B. practical C. critical D. urgent 31. He ____quite a lot when he was young. A. used to travel B. used to traveling C. was used to travel D. would use to travel 32. You ____me your telephone number in case someone wants to contact you. A. had better give B. had better given C. had better to give D. had better gave 33. Mary used to ____the room with Linda. A. separate B. divide C. hold D. share 34. " Must we hand in our exercise-books now?" "No, you ____. A. mustn't B. don't C. needn't D. can't 35. She pulled away from the window ____anyone should see them. A. lest B. even though C. unless D. only if 36. Not a ____has been found so far that can help the police find the criminal. A. fact B. clue C. symbol D. sign 5 37. She would make a teacher far superior ____the average. A. over B. than C. beyond D. to 38. Radio is different from television in ____it sends and receives pictures. A. which B. that C. what D. this 39. Tom and Jack have returned but ____students of the group haven't come back yet. A. other B. the others C. others D. another 40. It ____half a year since we ____to study in this university. A. is; come B. is; have come C. has been; came D. has been; have come 41. The fact that something is cheap doesn't ____mean it is of low quality. A. necessarily B. especially C. essentially D. practically 42. They set off by car and ____the nearest town. A. made for B. made after C. made out D. made to 43. Take this baggage and ____you can find enough room. A. put it which B. put it in which C. put it at where D. put it wherever 44. He doesn't want ____that he's going away. A. to be known B. him to be known C. that to be known D. it to be known 45. The noise around was terrible, but I had to ____it. A. keep away from B. keep up with C. live with D. live on 46. He ____that his guests were bored, although they were listening politely. A. impressed B. sensed C. inferred D. identified 47. On Sundays I prefer ____at home to ____out. A. to stay; go B. stay; going C. staying; going D. staying; go 48. I'd like to write to him, but what's the ____? He never writes back. A. significance B. business C. point D. purpose 49. There were ____opinions as to the best location for the new school. A. disagreeing B. conflicting C. rejecting D. reverting 50. ____by the news of his father's death, he could hardly utter a word. A. To be stunned B. Stunned C. To stun D. Stunning 51. ____, we'd better make some changes in the plan. A. That is the case B. That to be the case C. That been the case D. That being the case 52. They have equipped the office with the ____business machines. A. last B. latter C. latest D. later 53. The police found that George had still another ____of income. A. origin B. source C. basis D. means 54. An open-minded teacher doesn't always ____one single teaching method. A. set aside B. take over C. take on D. stick to 55. Much to the student's ____, the exam was postponed. A. burden B. concern C. relief D. requirement 6 56. Children normally feel a lot of ____about their first day at school. A. anxiety B. difference C. feelings D. trouble 57. The weather was ____hot that she decided to have the barber ____her hairstyle. A. rather; to change B. so; change C. much too; change D. too; changed 58. She ____meet her former instructor on the bus. A. delighted to B. happened to C. pleased to D. tended to 59. Just as no two words are truly synonymous. ____no two different expressions can mean exactly the same thing. A. rather B. also C. yet D. so 60. The new engineer's suggestions were ____in the revised plan. A. entitled B. engaged C. embodied D. estimated Part III Cloze(10,,15minutes) Directions: There are 20 blanks in the following passage. For each blank there are four choices marked A, B, C and D. You should choose the ONE that best fits into the passage. Then mark the corresponding letter on the Answer Sheet with a single line through the center. Other experiments have shown 61 the brain needs time to "digest" 62 has been learned. The time necessary 63 this is 5 to 10 minutes. After a break of this period of time the memory will have 64 what has just been learnt, and more will be remembered. During this break, 65 is important to exercise the right side of the brain, 66 the left side is used during a learning period. 67 you should relax in some way. 68 music, breathing in fresh air, and 69 at a picture, are all ways of using 70 side of the brain. 61. A. that B. what C. which D. it 62. A. that B. how C. what D. which 63. A. in B. for C. about D. to 64. A. attracted B. added C. brought D. absorbed 65. A. this B. it C. that D. such 66. A. because of B. due to C. because D. just as 67. A. So that B. Now that C. However D. Therefore 68. A. Hearing B. Attending C. Absorbing D. Listening to 69. A. looking B. look C. to look D. looked 70. A. other B. the other C. another D. others Historians can't tell us when, where or 71 the first food was cooked. In earliest 72 when people had eaten their food 73 , an fire was used only to provide heat and light. The first primitive cooks were74 women, 75 preparing food and making clothing were considered women's work. 76 most of the great chefs in history have been men. This might have been because chefs learned 77 work in the kitchens of rich families 78 in restaurants and women didn't often take jobs outside their homes, or it might have been because kitchen equipment was so heavy and difficult to work with 79 only strong men could do it. In modem times, great female chefs have become known, and some of the best cook books 80 by women. 71. A. who B. which C. how D. what 72. A. time B. times C. ages D. period 73. A. uncooked B. being cooked C. uncooking D. to be uncooked 74. A. probable B. possible C. probably D. likewise 75. A. while B. since C. so that D. unless 7 76. A. But B. Therefore C. And D. Moreover 77. A. his B. its C. their D. theirs 78. A. as well B. either C. both D. or 79. A. as B. such C. so D. that 80. A. have written B. have been written C. were written D. are written 第二卷(共30分) Part ? Vocabulary (10,,10minutes) Directions: Complete each of the following sentences with the proper word derived from the one given in the bracket. 1. Mother will need ____ (add) help while there are so many guests. 2. His face ____ (dark) with anger when he heard the bad news. 3. Many people keep their ____ (save) in banks. 4. Don't take the boat out in this ____ (storm) weather. 5. These studies give ____ (leading) results because they haven't looked at enough cases. 6. Smoking during pregnancy could ____ (danger) your baby's health. 7. Because of her ____ (refuse) to provide the information, I had to abandon the project. 8. My parents will he ____ (disappoint) with me if I fail the exam. 9. He is ardently ____ (act) for the cause of environmental protection. 10. The singer gave an ____ (accept) performance, but it was not outstanding. Part ? Sentence Completion(20,,30minutes) Directions: Put the underlined part in each sentence into English. Section A 1. 他全神贯注在工作上that he didn't hear anybody knocking at the door. 2. 这条河从前是干净的but it has got polluted since the chemical plant was built. 3. If he is not at the office, 就一定在别的什么地方。我刚才还看见过他。 4. 只要照着告诉你的做,every thing will be fine. 5. 那个工厂面临的问题is how to improve the quality of products. 6. 事实上直到读了你的信that I knew the true state of affairs. 7. If you want pass your exams, 你就得集中精力提高昕的技能。 8. She did not ask about his name, 以后也没有再遇见过他。 9. 我认为司机对那场事故负有责任。It was due to his carelessness that it all happened. 10. 我们到达那里时又累又饿,and worst of all we couldn't find any food. Section B Directions: Put the underlined part in each sentence into Chinese. 1. Each time I saw Mrs. Smith, I mistook her for Laura as they look very much alike. 2. When she heard the ringing, Lucy rose from her chair and made her way to the door, while her husband sat rooted in his seat. 3. I always took it for granted that they would succeed in their experiment. 4. Given better instructions, the water melons could have grown bigger. 5. I don't think you should act on his advice immediately. 6. The train was held up by the storm, thus causing the delay. 7. He thought that the bad working conditions would result in the strike of the workers. 8. The normal temperature for a human being is about 37?, no matter in what part of the world he lives. 9. When in Rome do as the Rome do. 8 10. Albert Einstein will always be remembered as the founder of the Theory of the Relativity. 2001年江苏省普通高校“专转本”统一考试答案及解析 Part I Reading Comprehension 1.参考答案: B 详细解答: B。由第一节第二句可知。 2.参考答案: C 详细解答: C。由第一节第二句可知。 3.参考答案: C 详细解答: C。由―Every letter takes less than a second to write(‖可知。 4.参考答案: A 详细解答: A。由第三节第一句可知,该计算机很昂贵;由以下内容可知,它速度很快,效率很高。 5.参考答案: D 详细解答: D。由本文最后一节可知,每封信中所说均是总统本人想说的。 6.参考答案: C详细解答: C。由―it’s necessary to obtain a wide and extensive knowledge of what has been said and done in the world‖可知。 7.参考答案: D 详细解答: D。由第二节第一句可知。 8.参考答案: B 详细解答: B。由―you must never give it up until you have mastered it‖可知。 参考答案: C 详细解答: C。第三节意在告诉人们,不要去看别人的成功,重要的是自己付出艰苦的努力。 10.参考答案: C 详细解答: C。最后一节主要讨论勤奋。 11.参考答案: D详细解答: D。本题采用排除法。 12.参考答案: D详细解答: D。由第二节―about not looking young and attractive"和"about their health‖可知。 13.参考答案: C详细解答: C。由第三节第一句可知,很多美国人都超出理想体重。 14.参考答案: A详细解答: A。减肥是项艰难的工程,但很多人都想通过一种又快又容易的方法,减肥方面的书因此畅销书店。作者暗示,这些书并不那么可靠。 15.参考答案: B 详细解答: B。因为减肥尚无定法,所以很多人都相信书店里减肥方面的书。 16.参考答案: B 详细解答: B。由最后一节可知。 17.参考答案: A 详细解答: A。该杂志很讲究用词,甚至小学教师都拿它来作为典范,可知各种人群都很喜欢它。 18.参考答案: C 详细解答: C。―时间是个顽皮的东西。它稍纵即逝,像喝牛奶一样吞掉每一天。‖ 19.参考答案: D 详细解答: D。指代前文所说的内容。 20.参考答案: D详细解答: D。由第二节第三句可知。 Part? Vocabulary and Structure 21.参考答案: A 详细解答: A。否定句中用anyone,表示―任何人‖。 22.参考答案: D 详细解答: D。固定句式。 23.参考答案: D 详细解答: D。固定短语,break one′s heart,义为―伤了某人的心‖。 24.参考答案: D 详细解答: D。too…to…结构。 25.参考答案: B 详细解答: B。Not until开头的句型主句主谓倒装。 26.参考答案: A 详细解答: A。定语从句所修饰的中心词是主句主语。 27.参考答案: A 详细解答: A。定语从句所修饰的中心词是主句主语。 28.参考答案: B 详细解答: B。do you suppose是插入语。 29.参考答案: B详细解答: B。固定短语,have trouble doing sth.,为―做某事有困难之义‖。 30.参考答案: C 详细解答: C。critical为―关键的‖之义。 31.参考答案: A 详细解答: A。used to do sth(,义为―过去常常做某事‖。 32.参考答案: A 详细解答: A。had better后接动词原形。 33.参考答案: D 详细解答: D。share,表示―共用‖之义。 9 34.参考答案: C 详细解答: C。needn′t表示―不需要‖之义。 35.参考答案: A 详细解答: A。lest为―防止‖之义,even though为―即便‖之义,unless为―除非‖之义, only if为―只有当‖之义。 36.参考答案: B 详细解答: B。警方需要寻找的是―线索‖。 37.参考答案: D 详细解答: D。固定短语superior to,义为―优于‖。 38.参考答案: B 详细解答: B。in that表示―在于‖之义。 39.参考答案: A 详细解答: A。后接名词,用other。 40.参考答案: C详细解答: C。固定句型,主句用现在完成时,since引导的从句中用过去时。 41.参考答案: A详细解答: A。necessarily为―必要地‖之义,especially为―特别地‖之义,essentially为―根本上地‖,practically为―实事求是地‖。 42.参考答案: A详细解答: A。make for为―前往‖之义。 43.参考答案: D详细解答: D。定语从句。 44.参考答案: D详细解答: D。it指代后面从句内容。 45.参考答案: C详细解答: C。live with为―共同生存‖之义,keep away from为―远离‖之义,keep up with为―赶上‖之义,live on为―靠某物生存‖之义。 46.参考答案: B 详细解答: B。他―意识到‖他的客人觉得无聊。 47.参考答案: C 详细解答: C。固定句型,prefer doing sth(to doing sth( 48.参考答案: C 详细解答: C。point为―点,目的‖之义。 49.参考答案: B 详细解答: B。关于新学校的选址,有―互相矛盾的‖意见。 50.参考答案: B 详细解答: B。分词短语作状语。 51.参考答案: D 详细解答: D。独立主格结构,主语接分词。 52.参考答案: C 详细解答: C。他们给办公室配备了―最新的‖商业机器。 53.参考答案: B 详细解答: B。警方发现他还有另一个收入―来源‖。 54.参考答案: D 详细解答: D。set aside为―留出‖之义,take over为―接管,接替‖之义,take on为―呈现‖之义,stick to为―坚持‖之义。 55.参考答案: C 详细解答: C。让那个学生―松了一口气‖的是,考试延期了。 56.参考答案: A 详细解答: A。学生通常在第一天上学时会感到―焦虑‖。 57.参考答案: B 详细解答: B。so…that…义为―如此……而……‖,have sb(do sth(义为―让某人做某事‖。 58.参考答案: B 详细解答: B。delighted to为―很高兴做某事‖之义,happen to义为―恰好做某事‖之义, pleased to为―乐意做某事‖之义,tend to为―倾向于做某事‖之义。 59.参考答案: D 详细解答: D。固定句型―just as…so...‖,义为―正如……,……也‖。 60.参考答案: C 详细解答: C。那位新工程师的建议在修改后的 计划 项目进度计划表范例计划下载计划下载计划下载课程教学计划下载 中得以―体现‖。 Part III Cloze 61.参考答案: A 详细解答: A。that引导定语从句。 62.参考答案: C 详细解答: C。定语修饰语在从句中作主语。 63.参考答案: B 详细解答: B。for表示目的。 64.参考答案: D 详细解答: D。attract义为―吸引‖,add义为―增加‖,bring义为―带来‖,absorb义为―吸收‖。 65.参考答案: B 详细解答: B。it指代后面从句。 66.参考答案: C 详细解答: C。表明原因。 67.参考答案: D 详细解答: D。表明推导出的结论。 68.参考答案: D 详细解答: D。听音乐:listen to music。 69.参考答案: A 详细解答: A。并列结构,同样用分词。 10 70.参考答案: B 详细解答: B。大脑只有两边,所以用the other。 71.参考答案: C 详细解答: C。用how引导方式状语。 72.参考答案: B rliest times为―最早的年代‖之义。 73.参考答案: A 详细解答: A。很早时人们吃―没有煮熟的‖食物。 74.参考答案: C 详细解答: C。作者根据下半句的内容推测如此。 75.参考答案: B 详细解答: B。since表示―既然‖,说明推测的理由。 76.参考答案: A 详细解答:A。最早的厨师可能是女人,―但是‖历史上绝大多数的名厨都是男人。 77.参考答案: C 详细解答: C。厨师们学习―他们的‖厨艺。 78.参考答案: D 详细解答: D。厨师们在富有的家庭―或‖饭馆学习厨艺。 79.参考答案: D 详细解答: D. so…that…_句型。 80.参考答案: B 详细解答: B。用现在完成时态。 Part ? Vocabulary 1. additional 2. darkened 3. savings 4. stormy 5. misleading 6. endanger 7. refusal 8. disappointed 9. active 10. acceptable Part ? Sentence Completion Section A 1. He was so absorbed in his work 2. This river used to be very clean 3. he must have been somewhere else for I saw him just now. 4. As long as you act on what I told you, 5. The problem the factory is facing 6. It was not until I had read your letter 7. you'd better concentrate on improving your listening comprehension. 8. nor did she meet him later. 9. I think the driver was responsible for the accident. 10. When we arrived there we were exhausted and hungry, Section B 1. 我总是把她错认为是劳拉 2. 而她先生还坐在座位上。 3. 我总是想当然地认为 4. 如果给予更完善的指导, 5. 立刻听从他的建议。 6. 火车因为风暴耽搁了, 7. 导致了工人的罢工。 8. 不管人们生活在世界的哪个角落。 9. 入乡随俗。 10. 将作为相对论的创立者被人们永远铭记。 2002年江苏省普通高校“专转本”统一考试试卷 大学英语 第?卷(共100分) 注意事项: 1(答第?卷前,考生务必将自己的姓名、准考证号、考试科目用铅笔涂写在答题卡上。 11 2(每小题选出答案后,用2B铅笔把答题卡上对应题目的答案标号涂黑。如需改动,用橡皮擦干净后, 再选涂其他答案标号。如果答案不涂写在答题卡上,成绩无效。 Part I Reading Comprehension (40,,40minutes) Directions: In this part there are four passages. Each passage is followed by a number of comprehension questions. Read the passage and answer the questions. Then mark your answer by blackening the corresponding letter on the Answer Sheet. Passage 1 In Washington D. C. , 1600 Pennsylvania Avenue is a very special address. It is the address of the White House, the home of the President of the United States. Originally the White House was gray and was called the Presidential Palace. It was built from 1792 to 1800. At this time, the city of Washington itself was being built. It was to be the nation's new capital city. George Washington, the first President, and Pierre Charles L'Enfant, a French engineer, chose the place for the new city. L'Enfant then planned the city. The President's home was an important part of the plan. A contest was held to pick a design for the President's home. An architect named James Hoban won. He designed a large three-story house of gray stone. President Washington never lived in the Presidential Palace. The first President to live there was John Adams, the second President of the United States, and his wife. Mrs. Adams did not really like her new house. In her letters, she often complained about the cold. Fifty fireplaces were not enough to keep the house warm. In 1812 the United States and Britain went to war. In 1814 the British invaded Washington. They burned many buildings, including the Presidential Palace. After the war James Hoban. the original architect, partially rebuilt the President's home. To cover the marks of the fire, the building was painted white. Before long it became known as the White House. The White House is one of the most popular tourist attractions in the United States. Every year more than l.5 million visitors go through the five roams that are open to the public. 1. The White House was built in Washington ____. A. because a French engineer was invited to design it B. because President George Washington liked to live in it C. because the British invaders lived in it in 1812-1814 D. because it was to be the nation's capital city 2. The Presidential Palace was ____. A. painted gray and white B. made of gray stone C. made of white stone D. made very warm in winter 3. The President's home and the city of Washington were ____. A. built by the American army B. built by the British troops C. planned by George Washington D. planned by the French 4. The original home of the President needed to be rebuilt ____. A. because John Adams' wife did not like it B. because it was cold in winter even with 50 fireplaces C. because it had burned down during the war D. because George Washington was not willing to live in it 5. The new presidential home was painted white to ____. A. cover the marks of fire B. attract tourists from France C. to please Mrs. John Adams D. keep it warm in winter 12 Passage 2 There are two kinds of memory: short-term and long-term. Information in long term memory can be remembered at a later time when it is needed. The information may be kept for days or weeks. However, information in short-term memory is kept for only a few seconds, usually by repeating the information over and over. The following experiment shows how short-term memory has been studied, Henning studied how students who are learning English as a second language remember vocabulary. The subjects in his experiment were 75 college students. They represented all levels of ability in English: beginning, intermediate, advanced, and native-speaking students. To begin, the subjects listened to a recording of a native speaker reading a paragraph in English. Following the recording, the subjects took a 15-question test to see which words they remembered. Each question had four choices. The subjects had to circle the word they had heard in the recording. Some of the questions had four choices that sound alike. For example, weather, whether, wither, and wetter are four words that sound alike. Some of the questions had four choices that have the same meaning. Method, way, manner, and system would be four words with the same meaning. Finally the subjects took a language proficiency test, Henning found that students with a lower proficiency in English made more of their mistakes on words that sound alike; students with a higher proficiency made more of their mistakes on words that have the same meaning. Henning's results suggest that beginning students hold the sound of words in their short-term memory, while advanced students hold the meaning of words in their short-term memory. 6. Henning made the experiment in order to study ____. A. how students remember English vocabulary by short-term memory B. how students learn English vocabulary C. how to develop students' ability in English D. how long information in short-term memory is kept 7. Which of the following statements is TRUE according to the passage? A. Information in short term memory is different from that in long-term memory. B. Long-term memory can be achieved only by training. C. It is easier to test short-term memory than long-term memory. D. Henning gave a separate test on vocabulary to his students. 8. From Henning's results we can see that ____. A. beginners have difficulty distinguishing the pronunciation of words. B. advanced students remember words by their meaning C. it is difficult to remember words that sound alike D. it is difficult to remember words that have the same meaning 9. The word "subject" in the passage means ____. A. the college course the students take B. the theme of the listening material C. a branch of knowledge studied D. the student experimented on 10. The passage centers on ____. A. memory B. two kinds of memory C. short-term memory D. an experiment on students Passage 3 The cowboy is the hero of many movies. He is, even today, a symbol of courage and adventure. But what was the life of the cowboy really like? The cowboy's job is clear from the word cowboy. Cowboys were men who took care of cows and other cattle. The cattle were in the West and in Texas. People in the cities of the East wanted beef from these cattle. Trains could take the cattle east, But first the cattle had to get to the trains. Part of the cowboy's job was to take the cattle 13 hundreds of miles to the railroad towns. The trips were called cattle drives. A cattle drive usually took several months. Cowboys rode for sixteen hours a day. Because they rode so much, each cowboy brought along about eight horses. A cowboy changed horses several times each day, The cowboys had to make sure that the cattle arrived safely. Before starting on a drive, the cowboys branded the cattle. They burned a mark on the cattle to show who they belonged to. But these marks didn't stop rustlers, or cattle thieves, Cowboys had to protect the cattle from rustlers. Rustlers made the dangerous trip even more dangerous. Even though their work was very difficult and dangerous, cowboys did not earn much money. They were paid badly. Yet cowboys liked their way of life. They lived in a wild and open country. They lived a life of adventure and freedom. 11. A cowboy is a symbol of ____. A. courage and adventure B. a hard life and big pay C. movies in the past D. cows and other cattle 12. The cowboys' job was ____. A. to be a hero in real life B. to be a hero of the movie C. to take care of cattle D. to be a dangerous rustler 13. During a cattle drive, cowboys took a group of cows from a wild and open country to ___. A. the West states and Texas B. the cities of the East States C. the people who eat beef in the cities D. the railroad towns hundred miles away 14. On their way of cattle drives, the cowboys protected the cattle by ____. A. burning a mark on their cows B. keeping an eye on cattle thieves C. making the trip more dangerous D. looking after eight cows each person 15. Cowboys enjoyed themselves because ____. A. they liked their way of life B. they made a lot of money C. they had a very difficult job D. they were heroes in movies Passage 4 Successful innovations have driven many older technologies to extinction and have resulted in higher productivity, greater consumption of energy, increased demand for raw materials, accelerated flow of materials through the economy and increased quantities of metals and other substances in use each person. The history of industrial development abounds with examples. In 1870, horses and mules were the prime source of power on U. S. farms. One horse or mule was required to support four human beings-a ratio that remained almost constant for many decades. At that time, had a national commission been asked to forecast the horse and the population for 1970, its answer probably would have depended on whether its consultants were of an economic or technological turn of mind. Had they been "economists" , they would probably have projected the 1970 horse or mule population to be more than 50 million, Had they been "technologists" , they would have recognized that the power of steam had already been harnessed to industry and to land and ocean transport. They would have recognized further that it would be the prime source of power on the farm. It would have been difficult for them to avoid the conclusion that the horse and mulepopulation would decline rapidly. 16. According to the passage, what supplied most of the power on U. S. farms in 1870? A. Animals B. Humans C. Engines D. Water 17. Which of the following is not mentioned by the author as a consequence of new technological developments? A. Older technologies die away. B. The quality of life is improved. 14 C. Overall productivity increases. D. More raw materials he come necessary. 18. It can be inferred from the passage that by 1870 ____. A. technology began to be more economical B. the steam engine had been invented C. the U. S. horse population was about 10 million D. a national commission on agriculture had been established 19. In the second paragraph, the author suggests that "economists" could ____. A. plan the economy through yearly forecasts B. fail to consider the influence of technological innovation C. value the economic contribution of farm animals D. consult the national commission on the economy 20. What is the author's attitude toward changes brought on by technological innovation? A. He is excited about them. B. He accepts them as natural. C. He is disturbed by them. D. He questions their usefulness. Part? Vocabulary and Structure(20,,25minutes) Directions: There are 40 incomplete sentences in this part. For each sentence there are four choices marked A, B, C and D. Choose the ONE answer that best completes the sentence. Then mark the corresponding letter on the Answer Sheet with a single line through the center. 21. ____the difficulties associated with the project, we'll go on with it. A. Given B. In spite of C. Thanks to D. Because of 22. It was almost dark in the streets ____a few very powerful spotlights. A. excluding B. but for C. except D. except for 23. ____today, he would get there by Friday. A. Would he leave B. If he leaves C. Was he leaving D. Were he to leave 24. He gave me some very ____advice on buying a house. A. precious B. expensive C. wealthy D. dear 25. His goal is not to become a sportsman ____a champion in a certain field. A. but rather became B. but rather to become C. but rather becoming D. but rather to becoming 26. I just met her ____on the way home from the bookstore. A. on purpose B. by accident C. in accident D. in case 27. I don't know about him, ____comment on him behind his back. A. let alone B. let go C. leave alone D. take leave 28. My transistor radio is out of order. It ____. A. need to be repaired B. need repairing C. needs repairing D. needs to repair 29. No one could tell us anything about the ____strangers. A. conscious B. mysterious C. serious D. previous 30. Mary ____all foolish comments and kept on working. A. excluded B. ignored C. denied D. discharged 31. I agree with him ____, but not entirely. A. until a certain point B. to some point C. to some extent D. until a certain extent 15 32. People in some parts of the world often take their water for ____.They use as much water as they wish. A. granted B. sure C. certain D. pleasure 33. Color-blind people often find it difficult to ____between blue and green. A. separate B. compare C. contrast D. distinguish 34. Thousands of people in the city ____to welcome the visiting guests. A. turned off B. turned up C. turned out D. turned over 35. The mountain place is beautiful, but ____the working conditions, it is terrible. A. when mentioned B. when it comes to C. when it is said D. when it dies to 36. Are you ____spending more money on the space program? A. in favor of B. by favor of C. in favor to D. out of favor 37. In the ____of many parents, standards of education in the public school are actually falling, A. idea B. thought C. opinion D. principle 38. ____from space, our earth, with water covering 70% of its surface, appears as a " blue planet". A. Seeing B. To be seen C. Seen D. Having seen 39. This year's total output value of industry and agriculture will increase ____5 percent over last year. A. by B. to C. of D. with 40. Mary is the top student in the class. She studies harder ____. A. than any student B. than all the students C. than any other student D. than some other student 41. Many people have applied for the ____position. A. empty B. bare C. vacant D. blank 42. My new shoes cost me 50 yuan (RMB). The price was ____that of the last pair I bought a month ago. A. two time more than B. twice as much as C. as twice D. as much as twice 43. Almost everyone failed ____on the first day. A. pass his driving test B. to have passed his driving test C. to pass his driving test D. passing his driving test 44. Over the traditional festival people visit each other and ____greetings. A. exchange B. wish C. congratulate D. present 45. It was ____because he was tired out that he fell asleep standing up. A. publicly B. openly C. specifically D. obviously 46. The young man was accused of ____the lady of her money. A. stealing B. robbing C. taking D. grasping 47. No matter where our Party needs us, we will ____her call. A. give answer for B. respond to C. have response to D. answer to 48. It is astonishing that a person of your intelligence ____be cheated so easily, A. could B. should C. might D. would 49. We were completely ____when we finally reached the destination. A. worn off B. won down C. worn out D. worn away 50. Many things ____impossible in the past are common today. A. considered B. to consider C. considering D. to be considered 51. Not until many years later ____known. 16 A. was the whole truth become B. did the whole truth become C. the whole truth became D. the whole truth had become 52. We didn't know his telephone number, otherwise we ____him. A. would telephone B. would have telephoned C. had telephoned D. must have telephoned 53. There is no point ____with him, since he has already made up his mind. A. argue B. to argue C. in arguing D. of arguing 54. I appreciate ____that letter for me. A. you to write B. your writing C. you write D. that you writing 55. I'd like to ____a special seat for the concert of May 5. A. serve B. reserve C. preserve D. conserve 56. ____that your son is well again, you no longer have anything to worry about. A. Since B. Now C. When D. After 57. Generally speaking, all kinds of materials will expand when heated but will ____when cooled. A. contrast B. contract C. survive D. return 58. You won't know if it fits you until you ____it on. A. will try B. are trying C. are to try D. have tried 59. After all efforts in vain, he had to accept the result ____. A. regularly B. shallowly C. physically D. painfully 60. The rest of his life is ____to the cause of international exchange of visiting scholars. A. added B. put C. saved D. committed Part III Cloze(10,,15minutes) Directions: There are 20 blanks in the following passage. For each blank there are four choices marked A, B, C and D. You should choose the ONE that best fits into the passage. Then mark the corresponding letter on the Answer Sheet with a single line through the center. Two old gentlemen lived in a quiet street in Paris. They were friends and neighbors, and they often went for a walk together in the streets when the weather was fine. Last Saturday they went for a walk at the side 61 the river. The sun shone, the weather was warm, there 62 a lot of flowers everywhere, and there were boats on 63 water. The two men walked happily for half an hour 64 then one of them said to the other, "That's a 65 beautiful girl." "Where can you see a beautiful girl?" said the 66 "I can't see one anywhere. I can see two young67 They're walking towards us. " "The girl's walking behind us." 68 the first man quietly. "But how can you see her 69 ?" asked his friend. The first man smiled and said. "I 70 see her. but I can see the two young men's eyes." 61. A. in B. on C. of D. with 62. A. were B. was C. is D. are 63. A. a B. for C. hot D. the 64. A. as B. and C. though D. with 65. A. ugly B. more C. very D. few 66. A. another B. lady C. other D. girl 67. A. girl B. men C. man D. boats 17 68. A. said B. lived C. cried D. walked 69. A. but B. pretty C. men D. then 70. A. can B. can't C. do D. did A contract is an agreement between two or more people 71 one person agrees to do something by a specified date in return for something done by 72 . Usually the contract is a written document signed and dated by both 73 . It must state clearly the consideration, that is, what is to be given or done by one person in 74 for what is given or done by the other. If one person does 75 was promised and the other does not, that other may be sued in court and required by court in order to make compensations. He or she 76 also be required to pay for damages suffered as a result of the failure to perform. The things 77 by both parties must be stated in definite terms 78 the court will hold that contract is 79 vague and general to be enforced. 80 the time period within which the work is to be done must be definite or the court will say that the document is not a contract. 71. A. in which B. that C. which D. on that 72. A. another B. the other C. other D. others 73. A. parts B. parties C. people D. aspects 74. A. replace B. payment C. change D. exchange 75. A. that B. the thing C. what D. something 76. A. need B. ought to C. has D. may 77. A. to be done B. done C. to do D. being done 78. A. and B. else C. or D. so 79. A. so B. rather C. too D. over 80. A. Similar B. Similarly C. The same D. Too 第二卷(共30分) Part ? Vocabulary (10,,10minutes) Directions: Complete each of the following sentences with the proper word derived from the one given in the bracket. 1. We have come to the ____ (conclude) that there is no life on the planet of Venus. 2. ____ (short) after she graduated from the college, she went abroad with her parents. 3. If you are in need of help, you'd better not ____ (hesitation) to tell me. 4. The dying man said ____ (vague) to his lawyer, "I will leave all my property to the orphans. 5. A ____ (horrify) scream was heard at the dark midnight. 6. ____ (confident) is the key factor in communicating with the opposite sex. 7. There are some ____ (indicate) that they are generous young souls. 8. Please don't ____ (understand) me, My sole intention is to help you. 9. Children have a natural ____ (curious) about the world around them. 10. She needs some comfort, but I really can't think of anything ____ (suit) to say. Part ? Sentence Completion(20,,30minutes) Directions: Put the underlined part in each sentence into English. Section A 1. No agreement was reached关于该计划如何执行。 2. Practice makes perfect,正如俗话所说。 3. 如果天气许可的话,the spaceship will be launched tomorrow morning. 4. 就技术和财力而言,his proposal does sound feasible. 5. The police officer was removes from office 因为他多次玩忽职守。 6. 面对严重的财务问题,the company had to lay some of the workers. 18 7. 要不是交通堵塞的话,we should have arrived at the station in time. 8. 正是他的懒惰that resulted in his failure in the final examination. 9. 只要这本书值得一读,I'll try all means to get it. 10.老师从班上随便挑选了几位学生 and asked them to help him with his experiment. Section B Directions: Put the underlined part in each sentence into Chinese. 1. Should it be absolutely necessary for you to leave the table during the meal, you should ask the hostess to excuse you. 2. It had never occurred to him to start a family until he met Susan. 3. Strange as it may sound, I do feel that the more I learn, the less I know. 4. After his recovery from the illness, he made up his mind to give smoking for good. 5. In many case, hard work alone not necessarily lead to success. 6. I meant to cheer her up, but apparently my tactless words made her all the more depressed. 7. I was tempted to quarrel with my parents, but I managed to hold my tongue. 8. Teachers are supposed to be patient not to lose their temper no matter what mistakes their students make. 9. Usually students are expected to read and think for themselves, but sometimes they are required to work together with their classmates. 10. With population growth in an ever-shortening cycle, it is rather doubtful whether the each can provide enough food for mankind. 2002年江苏省普通高校“专转本”统一考试 大学英语答案及解析 Part I Reading Comprehension 1.参考答案:D 详细解答: D。由第二节可知,白宫建于1792年至1800年。那时,华盛顿城本身也 在建设中,并即将成为美国的新首都。 2.参考答案:B 详细解答: B。由第三节第二句可知。 3.参考答案:D 详细解答: D。由第二节末―L'Enfant then planned the city‖可知。 4.参考答案:C 详细解答: C。由第五节和第六节可知。 5.参考答案:A 详细解答: A。由第六节―To cover the marks of the fire‖可知。 6.参考答案:A 详细解答: A。由第三节可知,实验主要是研究学生如何利用短时记忆学习生词,本 文最后一句时结论。 7.参考答案: D 详细解答: D。本题可采用排除法。长久记忆和短时记忆的区别在于信息在大脑中 存储时间长短的不同;长久记忆需要通过不停地重复信息才能实现;C项文中没有提到,所以选择D项。 8.参考答案:B 详细解答: B。由最后一节第一句可知。 9.参考答案:D 详细解答: D。由第三节开头可知,subject是―实验对象‖的意思。 10.参考答案:C 详细解答: C。由最后一句可知,本文主要是研究讨论短时记忆。 11.参考答案:A 详细解答: A。由第一节可知。 12.参考答案:C 详细解答: C。由第二节前两句可知。 13.参考答案:D 详细解答: D。由第二节最后一句可知。 14.参考答案:B 详细解答: B。由第四节最后两句可知。 15.参考答案:A 详细解答: A。由最后一节后两句可知。 16. 参考答案: A 详细解答: A。由第二节第一句可知。 17.参考答案: B 详细解答: B。其余三项均在文中提到。 18.参考答案:B 详细解答: B。 由―they would have recognize that the power of steam had already been 19 harnessed to industry‖可知。 19.参考答案:B 详细解答: B。作者认为,经济学家会―规划出1970年超过五千万的马驴数量‖,因而他们也就忽略了技术革新的影响。 20.参考答案: B 详细解答: B。作者列举了技术革新给社会带来历史交替的例子,并没有特别的喜好情绪。 Part? Vocabulary and Structure 21.参考答案: B 详细解答: B。given为―考虑到,设定‖之义,in spite of为―尽管‖之义,thanks to为―幸亏‖之义,because of为―因为‖之义。 22.参考答案: D 详细解答: D。excluding为―不包括‖之义,but for为―要不是‖之义,except为―除了‖之义,except for为―除了,若不是‖之义。 23.参考答案: D 详细解答: D。现在时的虚拟语气。 24.参考答案: A 详细解答: A。precious为―宝贵的‖之义,expensive为―昂贵的‖之义,wealthy为―富有的‖之义,dear为―珍视的,昂贵的,亲爱的‖之义。 25.参考答案: B详细解答: B。but rather为―相反地‖之义,与前一分句中动词形式并列。 26.参考答案: B 详细解答: B。on purpose为―故意的‖之义,by accident为―偶然地‖之义,in accident为―在事故中‖之义,in case为―防止‖之义。 27.参考答案: A 详细解答: A。let alone为―更别提‖之义,let go为―放开‖之义,leave alone为―不管,不干涉‖之义,take leave为―道别‖之义。 28.参考答案: C 详细解答: C。need repairing相当于need to be repaired。 29.参考答案: B详细解答: B。conscious为―神志清醒的‖之义,mysterious为―神秘的‖之义,serious为―严肃的‖之义,previous为―之前的‖之义。 30.参考答案: B 详细解答: B。exclude为―不包括,排除‖之义,ignore为―忽视,忽略‖之义,deny为―拒绝给予‖之义,discharge为―执行,放出‖之义。 31.参考答案: C 详细解答: C。在某种程度上:to some extent。 32.参考答案: A 详细解答: A。认为……理所当然:take…for granted。 33.参考答案: D 详细解答: D。区分A和B: distinguish between A and B。 34.参考答案: B 详细解答: B。turn off为―关掉‖之义,turn up为―出现‖之义,turn out为―结果是‖之义,turn over为―翻过来‖之义。 35.参考答案: B 详细解答: B。当提到……:when it comes to… 36.参考答案: A 详细解答: A。赞成:in favor of。 37.参考答案: C 详细解答: C。in the opinion of sb.为―在某人看来‖之义。 38.参考答案: C详细解答: C。分词作状语。主语和分词的动作是被动关系,因此用过去分词。 39.参考答案: A详细解答: A。increase by表示增长的幅度。 40.参考答案: C 详细解答: C。than any other student表示―比其他任何一个学生‖。 41.参考答案: C 详细解答: C。位置―空缺‖用―vacant‖。 42.参考答案: B 详细解答: B。倍数的结构:倍数后接as+形容词+as。 43.参考答案: C 详细解答: C。fail to do sth. 表示未做成功某事。 44.参考答案: A 详细解答: A。exchange为―表达‖之义,wish为―祝福,祝愿‖之义,congratulate为―恭喜‖之义,present为―(使)呈现‖之义。 45.参考答案: D 详细解答: D。publicly为―公开地,公然地‖之义,openly为―公开地,公然地‖之义,specifically为―特定地,特有地‖之义。 46.参考答案: B 详细解答: B。rob sb. of sth.为―抢劫某人某物‖之义。 47.参考答案: B详细解答: B。respond to one's call为―响应某人的号召‖。 48.参考答案: B 详细解答: B。虚拟语气,后面从句中用should+动词原形。 20 49.参考答案: C 详细解答: C。wear off为―磨损,逐渐减弱‖之义,wear down为―磨损,损耗,使疲劳,使厌烦,克服‖之义,wear out为―用坏,穿破,耗尽‖之义,wear away为―磨损,衰退‖之义。 50.参考答案: A 详细解答: A。分词短语作后置定语,动词与主语是被动关系,所以用过去分词。 51.参考答案: B 详细解答: B。以not until开头,主句主谓倒装。 52.参考答案: B 详细解答: B。过去时的虚拟语气。 53.参考答案: C 详细解答: C。there is no point in doing sth.为―做某事没有作用‖之义。 54.参考答案: B 详细解答: B。appreciate后接名词或从句。 55.参考答案: B 详细解答: B。serve为―服务,提供‖之义,reserve为―保留,预定‖之义,preserve为―保护,维持‖之义,conserve为―保存,节约‖之义。 56.参考答案: B 详细解答: B。Now that为―既然‖之义。 57.参考答案: B 详细解答: B。contrast为―对比‖之义,contract为―收缩‖之义,survive为―存活‖之义,return为―归还,返回‖之义。 58.参考答案: D 详细解答: D。与主语有逻辑先后关系,因此用现在完成时态。 59.参考答案: D 详细解答: D。regularly为―有规律地‖之义,shallowly为―浅显地‖之义,physically为―身体上地,完全地‖之义,painfully为―痛苦地‖之义。 60.参考答案: D 详细解答: be committed to为―致力于‖之义。 Part III Cloze 61.参考答案: C 详细解答: C。固定搭配 62.参考答案: A 详细解答: A。与后面的名词单复数一致。 63.参考答案: D 详细解答: D。特指那条河。 64.参考答案: B 详细解答: B。顺承连接。 65.参考答案: C 详细解答: C。副词,修饰后面形容词。 66.参考答案: C 详细解答: C。the other表示两者中的―另一个‖。 67.参考答案: B 详细解答: B。根据最后一句提示可知,他看到的两个男青年。 68.参考答案: A 详细解答: A。第一个人静静地―说‖。 69.参考答案: D 详细解答: D。then表示―那么‖之义。 70.参考答案: B 详细解答: B。我―不能‖看到她,但是能看到男青年的眼睛。 71.参考答案: A 详细解答: A。定语从句,限定成分在从句中作介词宾语。 72.参考答案: B 详细解答: B。the other表示两者中的―另一方‖。 73.参考答案: B 详细解答: B。party表示―一方,当事人‖之义。 74.参考答案: D 详细解答: D。in exchange for为―交换‖之义。 75.参考答案: C 详细解答: C。what在主句中作宾语,在从句中作主语。 76.参考答案: D 详细解答: D。may后接动词原形。 77.参考答案: A详细解答: A。to be done表示某事还没做。 78.参考答案: C 详细解答: C。or为―否则‖之义。 79.参考答案: C 详细解答: C。too...to...结构。 80.参考答案: B 详细解答: B。similarly表示―同样地,类似地‖,修饰整个句子。 Part ? Vocabulary 1. conclusion 2. Shortly 3. hesitate 4. vaguely 5. horrified 6. Confidence 7. indications 8. misunderstand 9. curiosity 10. suitable Part ? Sentence Section A 1. as to how to carry out this plan. 2. as the saying goes. 3. If weather permits/Weather permitting, 4. As far as technology and finance are concerned( 5. because he ignored his duty many times. 21 6. Faced with serious financial problems 7. Had it not been for the traffic jam/But for traffic jam( 8. It was his laziness 9. As long as the book is worth reading, 10. The teacher selected a few students at random Section B 1. 如果在吃饭期间你必须得离开餐桌的话, 2. 他从来没有想过要成家 3. 虽然这听起来有些奇怪, 4. 他决心永远戒烟。 5. 仅仅凭努力工作不一定会走向成功。 6. 我原想使她高兴起来, 7. 但是我尽量克制自己没有吭声。 8. 老师应该有耐心,不应该发脾气 9. 学生应该常常独立去阅读和思考, 10. 由于人口增长的周期日益缩短 2003年江苏省普通高校“专转本”统一考试试卷 大学英语 第?卷(共100分) 注意事项: 1(答第?卷前,考生务必将自己的姓名、准考证号、考试科目用铅笔涂写在答题卡上。 2(每小题选出答案后,用2B铅笔把答题卡上对应题目的答案标号涂黑。如需改动,用橡皮擦干净后,再 选涂其他答案标号。如果答案不涂写在答题卡上,成绩无效。 Part I Reading Comprehension (40,,40minutes) Directions: In this part there are four passages. Each passage is followed by a number of comprehension questions. Read the passage and answer the questions. Then mark your answer by blackening the corresponding letter on the Answer Sheet. Passage 1 After a day of work and play, the body needs to rest. Sleep is necessary for good health. During this time, the body recovers from the activities of the previous day. The rest that you get while sleeping enables your body to prepare itself for the next day. There are four levels of sleep, each being a little deeper than the one before. As you sleep, your muscles relax little by little. Your heart beast more slowly, and your brain slows down. After you reach the fourth level, your body shifts back and forth from one level of sleep to another. Although your mind slows down, from time to time you will dream. Scientists who study sleep state that when dreaming occurs, your eyeballs begin to move quickly (although your eye-lids are closed). This stage of sleep is called REM, which stands for rapid eye movement. If you have trouble falling asleep, some people recommend breathing very slowly and very deeply. Other people believe that drinking warm milk will help make you drowsy. There is also an old suggestion that counting sheep will put you to sleep. 1. A good title for this passage is ____. A. Sleep B. Good Health C. Dreams D. Work and Rest 2. The word "drowsy" in the last paragraph means ____. A. sick B. asleep C. a little sleepy D. nosy 3. This passage suggests that not getting enough sleep might make you ____. A. dream more often B. have poor health C. nervous D. breathe quickly 4. During REM, ____. A. your eyes move often B. you dream 22 C. you are restless D. both A and B 5. The average number of hours of sleep that an adult needs is ____. A. approximately six hours B. around ten hours C. about eight hours D. not stated here Passage 2 For years, children in the industrial areas of Europe and America seldom left their smoky cities to see the beauties of the countryside. This was not because the woods and fields were always far away, but because they were too far from the city to permit people to make a day trip between morning and nightfall. In 1970, a young German schoolmaster had an idea which changed this state of affairs. He decided to turn his little schoolhouse into a dormitory or hostel for the summer holidays. Anyone who brought his sleeping bag and cooking equipment along could stay there for a very small quantity of money. The idea was a success. A few years later, the schoolhouse was much too small to hold the many young people who wanted to stay there, As a result, a dormitory was set up in an old castle nearby. This was the first Youth Hostel. Today, young students and workers of every country can meet in the hostel and get to know each other. When young people arrive at the hostel, they have only to show their card of membership in a hostel organization in their own country. This card will permit him to use the facilities of hostels all over the world for very low prices. Often, at the evening meal, a group of boys and girls from various parts of the country or the world will happen to meet at the same hostel. They may put their food together and prepare a dinner with many kinds of dishes. Sometimes a program will be organized after the meal with dances, songs, or short talks followed by a question period. One can learn a lot of things about other places, just by meeting people who come from those places. For this reason, a few weeks spent "hostelling" can be just as useful a part of one's education as classes in school. 6. The author says children in the city seldom went to the woods and fields because ____. A. these places were too far away for them to go between morning and nightfall B. it was impossible for them to go and get back in one day C. they were not old enough to take such a trip D. they were not permitted to go to these places 7. By saying "His idea was a success", the author means ____. A. the school master made a lot of money and got rich B. the school master became very famous among young students C. more and more young people went to the hostel for summer holidays D. more and more young students came to study at his school 8. According to the passage, young people can pay less for their stay in the hostel if ____. A. they share their food with each other B. they can prove they have joined the hostel organization C. they have brought enough equipment with them D. they can prove that they are from a foreign country 9. In the author's opinion, young people can learn a lot during the days they speed "hostelling" because ____. A. they meet a lot of people from different places B. they have such useful classes in the hostel as they don't have in schools C. they have more interesting activities than at school D. they are very much interested in life in the hostel 23 10. Which do you think might be the proper tide for this passage? A. Youth Organization. B. International Travel C. Education Out of School. D. Summer Holidays. Passage 3 One of the most authoritative voices speaking to us today is, of course, the voice of the advertisers. It shouts at us from the television screen and the radio loudspeakers; waves to us from every page of the newspaper; signals to us from the roadside bill-boards all day and flashes messages to us in colored lights all night. Advertising has been among England's biggest growth industries since the war. Perhaps the reason is that advertising saves the manufacturers from having to think about the customer. At the stage of designing and developing a product, there is quite enough to think about without worrying over whether anybody will want to buy it. The designer is busy enough without adding customer-appeal to all his other problems of man-hours and machine tolerance and stress factors. So they just go ahead and make the thing and leave it to the advertiser to find clever ways of making it appeal to purchasers after they have finished it, by pretending that it confers(赋予) status, or attracts love, or signifies manliness. Other manufacturers find advertising saves them from changing their product. And manufacturers hate change. The ideal product is one that goes on unchanged forever. If, therefore, for one reason or another, some alteration seems called for-how much better to change the image, the packet or the pitch made by the product, rather than go to all the inconvenience of changing the product itself. 11. Which of the following can best describe the author's attitude toward modem advertising? A. Indifferent B. Shocked C. Disappearing D. Approving 12. According to the author, which is NOT the designer's chief concern when he designs a product? A. Stress factors B. Man-hours. C. Machine tolerance D. Customer-appeal. 13. It is stated in the passage that those responsible for giving a product customer-appeal are _. A. customers B. designers C. advertisers D. manufacturers 14. According to the author, when some change in a product is necessary, a manufacturer will choose to ____. A. lower the production cost B. hire a better designer C. improve its quality D. alter its image 15. The best title for the passage might be ____. A. Advertising since the War B. Advertising and Manufacturers C. Advertising-England's Biggest Industry D. Advertising and Purchasers Passage 4 All that we really need to plot out the future of our universe are a few good measurements. This does not mean that we can sit down today and outline the future course of the universe with anything like certainty. There are still too many things we don't know about the way the universe is put together. But we do know exactly what information we need to fill in our knowledge, and we have a pretty good idea of how to go about getting it. Perhaps the best way to think of our present situation is to imagine a train coming into a switchyard( 调车场). All of the switches(转辙器)are set before the train arrives, so that its path is completely determined. Some switches we can see. others we cannot. There is no ambiguity if we can see the setting of a switch; we can say with confidence that some possible futures will not materialize and others will. At the unseen switches, however, there 24 is no such certainty. We know the train will take one of the tracks leading out, but we have no idea which one. The unseen switches are the true decision points in the future, and what happens when we arrive at them determines the entire subsequent course of events. When we think about the future of the universe, we can see our "track" many billions of years into the future, but after that there are decision points to be dealt with and possible fates to consider. The goal of science is to reduce the ambiguity at the decision points and find the true road that will be followed. 16. According to the passage, it is difficult to be certain about the dist universe because we ____. A. have too many conflicting theories B. do not have enough funding to continue our research C. are not sure how the universe is put together D. have focused our investigations on the moon and planets 17. What does the author see as the function of the universe's unseen "switches"? A. They tell us which one of the tracks the universe will use. B. They enable us to alter the course of the universe. C. They give us information about the lunar surface. D. They determine which course the universe will take in the future. 18. In the second paragraph, the word "track" could hest be replaced by which of the following? A. Band B. Rails C. Path D. Sequence 19. For whom is the author probably writing this passage? A. Train engineers. B. General audiences. C. Professors of statistics. D. Young children. 20. Which of the following statements best describes the organization of the passage? A. A statement illustrated by an analogy (类比). B. A hypothesis supported by documentation (引用文献). C. A comparison of two contrasting theories. D. A critical analysis of a common assumption. Part? Vocabulary and Structure(20,,25minutes) Directions: There are 40 incomplete sentences in this part. For each sentence there are four choices marked A, B, C and D. Choose the ONE answer that best completes the sentence. Then mark the corresponding letter on the Answer Sheet with a single line through the center. 21. A great many cancers can be cured but only if ____before they have begun spread or "colonize" in other parts of the body. A. properly treat B. properly treating C. being properly treated D. properly treated 22. ____she is a likeable girl, she is very difficult to work with. A. Since B. However C. As far as D. While 23. All the ____tourists gave the robber their money. A. frightened B. frightening C. frighten D. frightful 24. ____her age, she really did a good job in such a short time. A. Giving B. Gives C. Give D. Given 25. The soldier was ____with neglecting his duty. A. charged B. conducted C. changed D. committed 25 26. The reason why the car stopped was ____. A. because the road was not good B. that the road was not good C. due to the bad road D. because of the had road 27. You'd better hurry; ____you might be late for class. A. or B. and C. unless D. but 28. They ____on a trip into the desert in the following year. A. set in B. sent for C. set off D. sent off 29. The doctor felt John's arm to ____if the bone was broken. A. work out B. find out C. look at D. see out 30. He just ____my suggestion at the meeting yesterday. A. put away B. shut down C. showed off D. brushed aside 31. The question ____at the next meeting remains a secret. A. discussed B. to discuss C. to be discussed D. being discussed 32. His laziness ____his failure in the final exam. A. gave up B. contributed C. resulted in D. distributed 33. The teacher's ____to my statement about this poet led me to read widely about poems. A. change B. charge C. challenge D. chance 34. On most of the nights, Jane ____reading letters from her boyfriend. A. stayed off B. stayed on C. stayed out D. stayed up 35. The first-year students were learning from the army in Miyun, a suburb of Beijing near ____I lived. A. what B. where C. that D. which 36. Lynda and hundreds of young people like him ____the post of typist. A. approached B. applied for C. appealed to D. approved of 37. Prices have been ____rapidly in many cities. A. went up B. gone up C. going up D. go up 38. In this building each apartment could ____a family of six. A. house B. cover C. make D. include 39. I can hardly hear what he's saying, and ____. A. so can all the other people B. nor can all the other people C. so can hardly all the other people D. nor all the other people can 40. When he explained it again and again, the students' patience ____. A. ran over B. ran on C. ran out D. ran off 41. When her neighbor Grandma Wang became ill, the girl often ____. A. fitted in B. worked out C. held back D. helped out 42. If you really want to apply for the dangerous job, I won't ____, though I think it's a crazy idea. A. stood in your way B. stand on the way C. stand in your way D. stand by the way 43. He was trying to read; ____, the phone kept ringing. A. meanwhile B. then C. later D. afterwards 44. I am not of those people who like a strenuous(费力的) holiday; I believe in ____. A. took it easy B. taken it easy C. taking it easy D. taking it easily 45. If I don't ____them ____I should probably forget all about them. A. wrote; down B. write; down 26 C. write; for D. written; about 46. She told her little brother ____her hand tightly while they were crossing the busy street. A. hold on to B. holding on to C. held on to D. to hold on to 47. If you don't mind, I ____do my homework than play cards with you. A. had better B. prefer C. would rather D. would like 48. Their idea was to get us to ____the strike at once. A. call at B. call off C. call in D. call for 49. My car ____so I had to come by bus. A. fell down B. broke down C. fell over D. turned away 50. I could tell he was surprised from the ____on his face. A. appearance B. sight C. expression D. explanation 51. Which is ____, North America or South America? A. biggest B. the biggest C. more bigger D. bigger 52. You should observe carefully how the audience ____his speech. A. reach to B. refer to C. react to D. relate to 53. These farmers got a good harvest last year, so they ____a big sum of money for new farm machines. A. set aside B. set about C. set up D. set back 54. One of my ____sayings is "where there is a will, there is a way. " A. likely B. favorable C. alike D. favorite 55. All ____is a continuous supply of fuel oil. A. what is needed B. that is needed C. the thing is needed D. for their needs 56. I would like to take this opportunity to express my sincere ____of your help. A. appreciation B. appearance C. preparation D. appointment 57. When the ship was caught in a strong storm at the sea, the captain ____a danger signal. A. sent for B. sent off C. sent out D. sent around 58. The sea separates Taiwan ____the Chinese mainland. A. at B. into C. from D. of 59. It is decided that he ____for a bus to meet the guests from Beijing. A. call B. calls C. arrange D. arranges 60. I know it's not important but I can't help ____about it. A. to think B. thinking C. and think D. being thought Part III Cloze(10,,15minutes) Directions: There are 20 blanks in the following passage. For each blank there are four choices marked A, B, C and D. You should choose the ONE that best fits into the passage. Then mark the corresponding letter on the Answer Sheet with a single line through the center. Other experiments have shown 61 the brain needs time to "digest" 62 has been learned. The time necessary 63 this is 5 to 10 minutes. After a break of this period of time the memory will have 64 what has just been learnt, and more will be remembered. During this break, 65 is important to exercise the right side of the brain, 66 the left side is used during a learning period. 67 you should relax in some way. 68 music, breathing in fresh air, and 69 at a picture, are all ways of using 70 side of the brain. 61. A. that B. what C. which D. it 62. A. that B. how C. what D. which 27 63. A. in B. for C. about D. to 64. A. attracted B. added C. brought D. absorbed 65. A. this B. it C. that D. such 66. A. because of B. due to C. because D. just as 67. A. So that B. Now that C. However D. Therefore 68. A. Hearing B. Attending C. Absorbing D. Listening to 69. A. looking B. look C. to look D. looked 70. A. other B. the other C. another D. others There are many problems in our modern world. One very serious problem is energy. We get a great 71 of energy we need from coal, gas, and oil. However, the 72 of energy which we use is 73 every year, and we only have enough coal, gas, and oil for the next twenty or thirty years. How will we live 74 the energy which these things give us? Scientists are looking for 75 to this problem. They are looking for new 76 to produce energy. For example, they are working with new ways 77 energy from the light and heat of the sun. They are also working with plans which produce energy from 78 of the oceans. All of the new methods 79 scientists arc finding arc still very expensive, but perhaps they will help solve our energy problems 80 the future. 72. A. effect B. amount C. course D. program 73. A. increase B. increasing C. had increased D. is increasing 74. A. without B. improve C. producing D. strength 75. A. weather B. a direction C. a solution D. service 76. A. cost B. methods C. branch D. pound 77. A. show B. pay C. save D. produce 78. A. property B. remedy C. welfare D. movements 79. A. so that B. which C. whose D. of which 80. A. at B. for C. in D. from 第二卷(共30分) Part ? Vocabulary (10,,10minutes) Directions: Complete each of the following sentences with the proper word derived from the one given in the bracket. 1. What a pity that they didn't realize the entire event had been arranged for their ____ (convenient)! 2. We made plans for a visit but the subsequent ____ (difficult) with the car prevented it. 3. Max has no ____ (appreciate) of the fine things in life. 4. The problem requiring immediate solution will be given ____ (prior) at the meeting. 5. With the hospitality(热情好客) so ____ (character) of these people, they opened their house to over fifty guests. 6. Thinking is one of the most ____ (produce) activities of a human being. 7. A thief stole some ____ (value) paintings from the museum. 8. I am very doubtful whether the proposed scheme would be ____ (advantage) to us. 9. She's ____(fortune) enough to enjoy good health. 10. John is very quiet and shy, hut Bill has a very outgoing ____ (person) and loves to meet people. Part ? Sentence Completion(20,,30minutes) Directions: Put the underlined part in each sentence into English. Section A 28 1. 怪不得he speaks English fluently. He often communicates with foreigners in English. 2. 请务必you get in touch with the factory before you go there on business. 3. 尽管天气不好the football game went on and the audience stayed there, cheering for their favorite players. 4. The coat I bought last week fits me well 除了颜色外。 5. Nowad8ys most companies don't judge an employee simply根据其所受教育。 6. I tried to get out of the business,我发现这是不可能的。 7. He has a large collection of books,其中不少是英文版的。 8. As far as interest and hobbies are concerned,他们几乎没有相似之处。 9. Advanced technology is expected to benefit people而不是伤害他们。 10. 众所周知,our natural resources will be exhausted in the near future. Section B Directions: Put the underlined part in each sentence into Chinese. 1. It was very clever of her to turn his argument against himself. 2. Parents often check their children's homework and make a big fuss over grades. 3. Some people will be at a in case of a fire. 4. As is often the case. the more you use your brain, the more active it will become. 5. We must make full use of these waste material since they are still of some value. 6. As long as you keep up your English study, you'll certainly make greater progress. 7. If inflation gets any worse, people who have worked all their lives will end up with nothing. 8. Reading good books increases your contentment when you are cheerful and relieves your sorrow when you are sad. 9. Langu8ge can never be separated from society, Apart from society there is no language. 10. A good player should know how to take advantage of his opponent's weakness. 2003年江苏省普通高校“专转本”统一考试 大学英语答案及解析 Part I Reading Comprehension 1.参考答案: A详细解答: A。主旨题,从Sleep is necessary for good health,以及下两句的There are four levels of sleep,可做出正确选择。 2.参考答案: C 详细解答: C。猜测词义题。根据drowsy所在行的意思,有些人认为睡觉前喝牛 奶可使人变得困倦,想睡觉,即能推测drowsy的意思是―a little sleepy‖。 3.参考答案: B 详细解答: B。推断题。根据上下文的意思可知缺乏睡眠必然导致身体状况糟糕, 故选B。 4.参考答案: D 详细解答: D。仔细阅读,可以较容易地在文中找到REM这个阶段的两个特点, 即from time to time you will dream和rapid eye movement. 5.参考答案: D 详细解答: D。细节题。文中并没有提到一个成年人每天需要多长时间睡眠这方面 的信息。 6.参考答案: B 详细解答: B。第一段最后一句话―they were too far from the city to permit people to make a day trip between morning and nightfall(‖告诉我们孩子们不能成行的原因是森林田间的美景离他们太 远了。 7.参考答案: C 详细解答: C。根据文章意思,schoolmaster的创意成功了,结果是越来越多的人知 道他的 hostel了。 8.参考答案: B 详细解答: B。为细节题。根据文章第三段,该hostel采用了全球会员制,只要你 在自己所在国家加入该组织,就可以享有优惠的服务。 29 8.参考答案: B详细解答: B。为细节题。根据文章第三段,该hostel采用了全球会员制,只要你在自己所在国家加入该组织,就可以享有优惠的服务。 9.参考答案: A 详细解答: A。考察作者的观点。从最后一段倒数第三行just by meeting people who come from those places可得出结论。 10.参考答案: A 详细解答: A。主旨题。根据上下文意思可得出答案。此题也可用排除法。 11.参考答案: A 详细解答: A。A(indifferent(冷漠的);B(shocked(震惊的);C. disapproving(不赞成的);D(approving(赞成的)。根据分析及文章的第二段,A为正确答案。 12.参考答案: D 详细解答: D。根据―Perhaps the reason ls that advertising saves the manufacturers from having to think about the customer(‖这句话可得出答案。 13.参考答案: C 详细解答: C。根据文章第三段可知,the designer没有时间去考虑顾客的需求和癖好而把这些事情留给advertiser去做。 14.参考答案: B 详细解答: B。主旨题。主要讲广告商和生产者之间的联系 15.参考答案: B 详细解答: B。主旨题。主要讲广告商和生产者之间的联系。 16.参考答案: C 详细解答: C。根据There are still too many things we don't know about the way the universe is put together可知答案。 17.参考答案: D 详细解答: D。细节题。据最后一句话可知答案。 18.参考答案: C 详细解答: C。猜测词义题。―track‖一词可译为―路径,小路,足迹,车辙‖,根据上下文意思应为―路径‖。 19.参考答案: B 详细解答: B。本文的阅读对象应是一般读者。 20.参考答案: A 详细解答: A。细节题。根据对文章的分析我们不难看出,作者运用了类比来说明问题。 Part? Vocabulary and Structure 21.参考答案: D 详细解答: D。考查过去分词。用过去分词treated表条件。 22.参考答案: D 详细解答: D。since自从,既然;however表转折,然而;as far as到……程度,就……而言;while当……时候或表对比,D选项最合适。 23.参考答案: A 详细解答: A。过去分词用作形容词,表示状态和被动含义。一般来说,现在分词和过去分词用作形容词的主要区别在于前者表示性质和主动,而后者表示状态和被动含义。 24.参考答案: D 详细解答: D。过去分词given用作介词,表示假设或条件。类似的现在分词和过去分词已转化为介词的还有considering,provided等。 25.参考答案: A 详细解答: A。固定短语be charged with,被指控。 26.参考答案: B 详细解答: B。固定句型。The reason why…is that…(that引导的是表语从句。 27.参考答案: A 详细解答: A。or在这里用作连词,表示―否则,要不然‖。注意它和unless―除非‖的区别:l shall go there tomorrow unless I'm too busy(如果我不太忙,明天将到那儿去。 28.参考答案: C 详细解答: C。set off出发,动身;set about开始做,着手处理;set down放下,搁下,记下,记载,写下;set in开始并且(可能)持续下去;set up建立,成立。 29.参考答案: B 详细解答: B。短语辨析。work out解决,设计出,做出,计算出;find out找出,发现,查明(真相等);see out完成,贯彻。 30.参考答案: D 详细解答: D。动词短语辨析。brush aside漠视,不理,撇开不管;put away放好,储存,备用,处理掉,放弃,抛弃;shut down放下,关闭,停车;show off炫耀,夸耀,卖弄。 31.参考答案: C 详细解答: C。该题目的关键是句中的next。一般情况下,很容易选择单独的过去分词用作后置定语。但由于有了next这个单词,仅仅用discussed就不准确地表示出该动作的将来性。因此正确答案为动词不定式的过去分词用作后置定语。 32.参考答案: C详细解答: C。result in导致;distribute分发,分配;contribute捐助,捐献,贡献,投稿。 33.参考答案: B 详细解答: B。单词辨析。这种题目的关键是弄清楚除空格以外的句子含义,statement 30 非常重要,―陈述,综述‖,该题的最佳答案为B,―告诫,指示‖。 34.参考答案: D 详细解答: D。stay up doing something,熬夜做某事。 35.参考答案: B详细解答: B。介词的宾语从句的先行词。根据从句中的谓语动词live,只能用where。 36.参考答案: B 详细解答: B。该题实际上是考察对句意的理解。apply for―申请‖。句中的like him是介词短语而不是动词短语。 37.参考答案: C 详细解答: C。时态题。只能在B、C两者中选择,该题强调了动作的持续,且―价格‖和―上升‖应为主动关系,因此用现在完成时的进行时态。 38.参考答案: A 详细解答: A。生僻题,可用排除法。house用作及物动词,―给……房子住‖。 39.参考答案: B详细解答: B。根据并列连词and可排除A、C,再根据nor用于句首,句子倒装,得出正确答案。 40.参考答案: C 详细解答: C。词组辨析。run out用光,耗尽;run over跑过去,溢出,超过限度,匆匆看;run on连续,不分段,涉及;run off逃跑,流掉,进行决赛。 41.参考答案: D 详细解答: D。help out救助,协助(渡过难关)。 42.参考答案: C 详细解答: C。词组辨析。stand in one's way妨碍;stand by袖手旁观,站在一起,帮助,维持,遵守;stand in当替身,代替。 43.参考答案: A 详细解答: A。表示动作同时进行,用副词meanwhile( 44.参考答案: C 详细解答: C。believe in为后接介词的动词短语,相当于及物动词,后接名词性短语作宾语。 45.参考答案: B 详细解答: B。write down写下,记下。且该短语前有助动词do,应用原形。 46.参考答案: D 详细解答: D。固定短语tell somebody to do something。 47.参考答案: C 详细解答: C。后有连词than(前只能用would rather。would rather…than…宁愿……而不……。 48.参考答案: B 详细解答: B。短语辨析。call at打电话给某人+访问,停靠;call off取消;call in召集,召来,来访;call for要求,提倡,为……叫喊。 49.参考答案: B 详细解答: B。fall down跌倒,倒塌下来;break down破坏,拆散,(机器)损坏;fall over落在……之外,向前摔倒,叛向另一方面;turn away不准……入内,转过脸,解雇,避免,防止。 50.参考答案: C 详细解答: C。expression表达,表情,脸色;appearance出现,露面,外貌,外观,该词强调的是事物的―外观‖或―乍一出现‖。 51.参考答案: D 详细解答: D。两者之间的比较用比较级。 参考答案: C 详细解答: C。单词辨析。react to对……的反应。 52. 参考答案: C 详细解答: C。单词辨析。react to对……的反应。 53.参考答案: A 详细解答: A。短语辨析。set aside留下,留出;set a bout开始做,着手处理;set up建立(事业),成立(组织);set back阻碍,使受挫折。 54.参考答案: D 详细解答: D。likely通常作表语形容词,作前置形容词时,含义为―适合的,漂亮的‖。favorable赞成的,有利的;alike通常只用作表语;favorite喜爱的,中意的。 55.参考答案: B 详细解答: B。语法题。all不能后接what,因为what本身即具有名词性,它的替换短语为all that。 56.参考答案: A 详细解答: A。名词辨析。―表明我对你(给予我的帮助的)感谢‖,用appreciation。 57.参考答案: C 详细解答: C。动词短语辨析。send for召唤,派人去请;send off寄出,派遣,给……送行;send out发送,派遣,放出。 58.参考答案: C 详细解答: C。separate…from…把……和……分开;separate sth. into sth.把……分成…… 31 59.参考答案: C 详细解答: C。虚拟语气十短语辨析。arrange for安排 60.参考答案: B 详细解答: B。can't help doing something不禁做…… Part III Cloze 61.参考答案: A详细解答: A。完整的宾语从句用that来引导。 62.参考答案: C 详细解答: C。缺少主语的宾语从句,能作该宾语从句的主语的单词只有what。 63.参考答案: B 详细解答: B。this指代的是时间,根据后接的时间短语得出介词为for。 64.参考答案: D 详细解答: D。前文说―大脑需要时间来消化所获取的信息‖,那么,很明显,一定时间后,它就会―吸收‖这些信息。 65.参考答案: B 详细解答: B。固定句型(It is important to do something。 66.参考答案: C 详细解答: C。原因状语从句,用because来引导。 67.参考答案: D 详细解答: D。该空格前后的两句之间有因果关系,so that虽引导因果关系,但是在一个句子之内。 68.参考答案: D 详细解答: D。根据其后接动词短语的结构——现在分词短语即可得出正确答案。 69.参考答案: A 详细解答: A。同68题。 70.参考答案: B 详细解答: B。该句是特指人类大脑的右半部分。 71.参考答案: D 详细解答: D。a great deal of修饰不可数名词;a great number of修饰可数名词复数或集合名词。 72.参考答案: B 详细解答: B。我们使用的能量―数量‖正与日俱增。 73.参考答案: B详细解答: B。强调―增长‖持续进行,用现在进行时。 74.参考答案: A详细解答: A. without引导的介词短语表示假设。―如果没有了这些物质给我们提供的能量,……, 75.参考答案: C 详细解答: C。解决problem只能用solve,其名词形式为solution. 76.参考答案: B 详细解答: B. method方式,方法。该句是说科学家们正在寻找产生能源的新方法。 77.参考答案: D 详细解答: D。在76题的后半句,已给出了该单词。 78.参考答案: D 详细解答: D。该句是说海洋的潮汐―运动‖也可以用来制造能源。 79.参考答案: B详细解答: B。which引导定语从句修饰methods。 80.参考答案: C 详细解答: C。in the future未来,将来。 第二卷(共30分) Part ? Vocabulary 1. convenience 2. difficulty 3. appreciation4. priority 5. characteristic 6. productive 7. valuable8. advantageous 9. fortunate 10. personality Part ? Sentence Section A 1. No wonder 2. Please make sure/be certain that/see to it that 3. In spite of/Despite/Regardless of the bad weather 4. except for the color. 5. according to/by his educational background. 6. but in vain. 7. most of which are English versions. 8. they have little in common 9. rather than hurt them. 10. As is well-known that Section B 1. 让他自相矛盾。 2. 对他们的分数大惊小怪。 3. 失火时。 4. 事实上 5. 我们必须充分利用6. 只要你坚持学习英语7. 将会一无所有。 8. 当你悲伤的时候减轻你的痛苦。9. 脱离了社会,就没有语言之说了。 10. 利用对手的弱点 32 2004年江苏省普通高校“专转本”统一考试试卷 大学英语 第?卷(共100分) 注意事项: 1(答第?卷前,考生务必将自己的姓名、准考证号、考试科目用铅笔涂写在答题卡上。 2(每小题选出答案后,用2B铅笔把答题卡上对应题目的答案标号涂黑。如需改动,用橡皮擦干净后,再 选涂其他答案标号。如果答案不涂写在答题卡上,成绩无效。 Part I Reading Comprehension (40,,40minutes) Directions: In this part there are four passages. Each passage is followed by a number of comprehension questions. Read the passage and answer the questions. Then mark your answer by blackening the corresponding letter on the Answer Sheet. Passage 1 Some years ago the captain of a ship was very interested in medicine. He always took medicine books to sea and liked to talk about different diseases. One day a lazy sailor on his ship pretended to be ill. He lay on his bunk(铺) and groaned as if he were very sick. The captain came to see him and was very pleased to have a patient to look after. He told the man to rest for a few days and made the other sailors do his work. Three days later another sailor pretended that he had something wrong with his chest. Once more the captain looked in his medical books and told the "sick" man to have a rest. The other sailors were very angry because they had more work to do. The patients had the best food and laughed at their friends when the captain was not looking. At last the mate(船长副手) decided to cure the "sick" men. He mixed up some soap, soot(烟灰), glue(胶水) and other unpleasant things. Then he obtained permission from the captain to give his medicine to the "sick" men, When they tasted the medicine, they really did feel ill. It was so horrible that one of the patients jumped out of his bunk, ran up on deck and climbed the highest place on the ship. He did not want any more medicine. The mate told both of the men that they must take the medicine every half an hour, night and day. This soon cured them. They both said they felt better and wanted to start work again. The captain realized that the men tried to deceive him so he made them work very hard for the rest of the voyage. 1. The first sailor pretended to be ill because he wanted to ____. A. test the captain's knowledge of medicine B. be free from work C. have the best food on the ship D. play a joke on his friends 2. When the captain knew a sailor was ill, he ____. A. didn't care much B. sent for a doctor C. looked after him and told him to have a rest D. gave him some medicine 3. The patients felt better quickly because ____. A. they had been given proper treatment B. they learned that the captain had found out the truth C. they were laughed at by their friends D. the medicine the mate gave them was horrible 4. When the captain knew he had been deceived, he ____. A. told them not to do so again B. lost his temper C. made them work harder D. fired them 5. Which of the following best summarizes the passage? A. A Sudden Cure. B. Two Patients. C. Captain and Sailors. D. A Difficult Voyage. Passage 2 When aluminum was first produced about a hundred and fifty years ago, it was so difficult to separate from 33 the ores(矿石) in which it was found that its price was higher than that of gold. The price remained high until a new process was discovered for refining the metal with the aid of electricity approximately three quarters of a century later. The new method was so much cheaper that aluminum became practical for many purposes, one of which was making pots and pans. Aluminum is lightweight, rustproof and easily shaped into different forms. By mixing it with other metals, scientist have been able to produce a variety of alloys, some of which have the strength of steel but weigh only one third as much. Today, the uses of aluminum are innumerable. Perhaps its most important use is in transportation. Aluminum is found in the engines of automobiles, in the hulls of boats. It is also used in many parts of airplanes. In fact, the huge "airbus" planes would probably never have been produced if aluminum did not exist. By making vehicles lighter in weight aluminum has greatly reduced the amount of fuel needed to move them. Aluminum is also being used extensively in the building industry in some countries. Since aluminum is such a versatile(多用的) metal, it is fortunate that bauxite(铝土矿 ) , which is one of its chief sources, is also one of the earth's most plentiful substances. As the source of aluminum is almost inexhaustible, we can expect that more and more uses will be found for this versatile metal. 6. The price of aluminum was sharply reduced when people discovered a new refining process with the aid of ___. A. wind power B. solar energy C. hydraulic power D. electricity 7. Aluminum is ____. A. lightweight, rustproof but not easily shaped into different forms B. heavyweight, rustproof and easily shaped into different forms C. lightweight, rustproof and easily shaped into different forms D. lightweight and easily shaped into different forms but it is easy to become rusty 8. Which of the following is NOT TRUE? A. Aluminum is widely used in transportation. B. Aluminum is also used in many parts of airplanes. C. Aluminum is being used extensively in the building industry. D. Aluminum is not used in its pure form. 9. Aluminum is found on earth mostly in the form of ____. A. pure metal B. bauxite C. gold D. liquid 10. What is the passage talking about? A. The features of aluminum and its functions. B. The process of aluminum. C. The discovery of aluminum. D. The promising future of aluminum. Passage 3 The idea of a special day to honor mothers was first put forward in America in 1907. Two years later a woman, Mrs. John Bruce Dodd, in the state of Washington proposed a similar day to honor the head of the family-the father. Her mother died when she was very young, and her father brought her up. She loved her father very much. In response to Mrs. Dodd's idea that same year-1909, the state governor of Washington proclaimed (宣布) the third Sunday in June is Father's Day. The idea was officially approved by President Woodrow Wilson in 1916. In l924, President Calvin Coolidge recommended national observance of the occasion " to establish more intimate (亲 密的 ) relations between fathers and their children, and to impress upon fathers the full measure of their 34 obligations. "The red or white rose is recognized as the official Father's Day flower. Father's day took longer to establish on a national scale than Mother's Day, but as the idea gained popularity, tradesmen and manufacturers began to see the commercial possibilities. They encouraged sons and daughters to honor their fathers with small thank-you presents, such as a tie or a pair of socks, as well as by sending greeting cards. During the Second World War, American servicemen stationed in Britain began to request Father's Day greeting cards to send home. This generated a response with British card publishers. Though at first the British public was slow to accept this rather artificial day, it's now well celebrated in Britain on the third Sunday in June in much the same way as in America. Father's Day seems to be much less important an occasion than the Mother's Day. Not many of the children offer their fathers some presents. But the American fathers still think they are much better fated than the fathers of many other countries, who have not even a day for their sake in name only. 11. When did Father's Day officially begin to have national popularity? A. 1907 B. 1909 C. 1916 D. 1924 12. Who first started the idea of holding the Father's Day? A. Mrs. John Bruce Dodd. B. Mrs. John Bruce's Mother C. The government of Washington. D. Some businessmen. 13. What flower will be popular on Father's Day? A. Lily B. Water lily. C. Red rose or white rose. D. Sunflower 14. Which statement is true, according to this passage? A. It took even longer for Mother's Day to gain national popularity. B. The businessmen helped to make Father's Day popular, C. Father's Day is only celebrated in America, D. Father's Day is only a trick of the businessmen to make money. 15. What was the first reaction of the British public towards Father's Day? A. They thought highly of it and accepted it at once B. They just accepted it at once without any hesitation C. They just thought it a joke. D. They thought it was too artificial and took a long time to accept. Passage 4 Culture shock is an occupational disease (职业病) for people who have been suddenly transplanted abroad. Culture shock is caused by the anxiety that results from losing all familiar signs and symbols of social intercourse. Those signs are as following: How to shake hands and what to say when meeting people, when and how to give tips, how to make purchases, when to accept and words, invitations, when to take statements seriously and when not. These signs, which gestures. facial expressions. or customs. are acquired by all of us in the course of growing up and as much a part of our culture as the language we speak or the beliefs we accept. All of us depend on hundreds of these signs for the peace of our mind and day-to-day efficiency, but we do not carry most at the level of conscious awareness. Now when an individual enters a strange culture, all or most of these familiar signs are removed. No matter how broadminded or full of good will you may be a series of supports have been knocked from under you, followed by a feeling of frustration. When suffering from culture shock people first reject the environment which causes discomfort. The ways of the host country are bad because they make us feel bad. When foreigners on a strange land get together to complain about the host country and its people, you can be sure that they are suffering 35 from culture shock 16. According to the passage, culture shock is ____. A. an occupational disease of foreign people B. may lead to very serious symptoms C. actually not a disease D. incurable 17. According to the passage, culture shock results from ____. A. the sudden change of social atmosphere and customs B. the sudden change of our daily habits C. the sudden loss of our own signs and symbols D. the discomfort that we feel when faced with a foreigner 18. Which one of the following may not be a symptom of culture shock? A. You don't know how to express your gratitude. B. You don't know how to greet other people. C. You suddenly forget what a word means. D. You don't understand why a foreigner shrugs. 19. According to the passage, how would a person who stays abroad most probably react when he is frustrated by the culture shock? A. He is most likely Lo refuse to absorb the strange environment at first. B. He is ready to accept the change and adapt himself to the new environment. C. Although he takes the culture difference for granted, he still doesn't know how to do with it. D. He may begin to hate the people or things around him. 20. The main idea of this passage is that ____. A. culture shock is an occupational disease B. culture shock is caused by the anxiety of living in a strange culture C. culture shock has peculiar symptoms D. it is very hard to cope with life in a new setting Part? Vocabulary and Structure(20,,25minutes) Directions: There are 40 incomplete sentences in this part. For each sentence there are four choices marked A, B, C and D. Choose the ONE answer that best completes the sentence. Then mark the corresponding letter on the Answer Sheet with a single line through the center. 21. The teacher ____the students on a tour through the art museum. A. made B. indicated C. forced D. took 22. Tom's parents died when he was a child, so he was ____by his relatives. A. grown up B. brought up C. raised D. fed up 23. Here is my card. Let's keep in ____. A. touch B. relation C. connection D. friendship 24. So far there is no proof ____people from other planets do exist. A. which B. how C. what D. that 25. The newspapers reported yesterday several ____on the boundaries of these two countries. A. incidents B. happenings C. events D. accidents 26. We've worked out the plan and now we must put it into ____. A. fact B. reality C. practice D. deed 27. He didn't ____and so he failed the examination. A. work enough hard B. hard work enough C. hard enough work D. work hard enough 36 28. Not until Mr. Smith came to China ____what kind of country she is. A. he knew B. he didn't know C. did he know D. he couldn't know 29. Scientists say it may be ten years ____this medicine was put to use. A. since B. before C. after D. when 30. In some countries, ____is called "equality" does not really mean equal rights for all people. A. that B. what C. which D. how 31. We didn't know his telephone number, otherwise we ____him. A. would telephone B. would have telephoned C. had telephoned D. must have telephoned 32. We've missed the last bus, I'm afraid we have no ____but to take a taxi. A. way B. possibility C. choice D. selection 33. Luckily, most sheep ____the flood last month. A. endured B. survived C. lived D. passed 34. My parents always let me have my own ____of living. A. way B. method C. manner D. fashion 35. Like ____other language skills, reading requires practice. A. the most of B. much of the C. most of the D. more of the 36. It is only through practice ____one will be able to swim skillfully. A. what B. who C. that D. which 37. The brain is capable of ignoring pain messages if ____to concentrate on other activities. A. it allowed B. is it allowed C. allowed D. allowed it 38. Don't worry, I have already ____them ____the decision. A. informed; with B. informed; of C. informed for D. informed; that 39. The child was sorry ____his mother when he arrived at the station. A. to miss B. having missed C. missing D. to have missed 40. I wonder why he ____to discuss the problem at the meeting. A. declined B. rejected C. refused D. delayed 41. You can hang up what you like on these ____walls. A. bare B. empty C. blank D. vacant 42. According to a ____, the majority would rather have newspapers without a government than a government without newspapers. A. election B. campaign C. poll D. vote 43. The population of the village has decreased ____150 to 500. A. in B. at C. by D. with 44. It seems that there is ____that I can't do. A. nothing B. anything C. everything D. none 45. They are often ____caring more about animals than human beings. A. accused of B. accused with C. charged of D. charged for 46. ____a good beginning is made, the work is half done. 37 A. As soon as B. While C. As D. Once 47. George could not ____his foolish mistake. A. account in B. count on C. count for D. account for 48. We came into this field late, so we must work hard to ____the lost time. A. make up for make up for B. make out C. keep up with D. put up with 49. The new law will come into ____on the day it is passed. A. effect B. use C. service D. existence 50. We can separate the mixture into the pure chemical compounds ____it is composed. A. in which B. of that C. of which D. from which 51. Mrs. Lindon has ____that she is unable to get a job. A. such small education B. so little education C. a such little education D. a so small education 52. She can't prevent her little boy ____shooting ____birds. A. from; to B. on; at C. with; up D. from; at 53. Many countries are increasing their use of natural gas, wind and other forms of ____. A. energy B. source C. power D. material 54. A darkened sky in the daytime is usually an indication that a storm is ____. A. possibly coming B. about to take place C. close by D. expected to be severe 55. We all know that ____speak louder than words. A. movements B. performance C. operations D. actions 56. ____, he could not cover the whole distance in fifteen minutes. A. Fast as he ran B. As he ran fast C. If he ran fast D. Since he ran fast 57. Agricultural production in that country has increased ____in recent years. A. vastly B. strikingly C. considerably D. extremely 58. Peter has planned to ____some money every month so that he can buy a used car next year. A. set aside B. set up C. set in D. set along 59. Although I spoke to him many times, he never took any ____of what I said. A. attention B. notice C. warming D. observation 60. They overcame all the difficulties and fulfilled the plan three months ahead of time, ____ is something we had not expected. A. at B. what C. it D. which Part III Cloze(10,,15minutes) Directions: There are 20 blanks in the following passage. For each blank there are four choices marked A, B, C and D. You should choose the ONE that best fits into the passage. Then mark the corresponding letter on the Answer Sheet with a single line through the center. Most Americans don't like to get advice from members of their family. When they need advice, they don't usually 61 people they know. 62 , many Americans write letters to newspapers and magazines which give advice 63 many different subjects, including family problems, sex, the use64 the language, health, cooking, childcare, 38 clothes, and how to buy a house or a car. 65 newspapers regularly print letters 66 readers with problems. Along 67 the letter there are answers written 68 people who are supposed to know how to 69 such problems. Some of these writers are doctors; 70 are lawyers or educators. But two of the most famous writers of advice 71 women without special training 72 this kind of work. One of them answers letters 73 to "Dear Abby". The other is addressed 74 "Dear Ann Landers". Experience is their preparation for 75 advice. There is one writer who has not lived long 76 to have much experience. She is a girl named Angel Cavaliere, who started writing 77 for newspaper readers 78 the age of ten. Her advice to young readers now 79 regularly in the Philadelphia Bulletin in a column 80 DEAR ANGEL. 61. A. talk B. ask C. tell D. speak 62. A. Because B. Instead C. When D. As 63. A. for B. in C. on D. with 64. A. with B. on C. to D. of 65. A. Most B. These C. Those D. The 66. A. from B. for C. to D. about 67. A. in B. with C. on D. for 68. A. to B. for C. about D. by 69. A. make B. overcome C. beat D. solve 70. A. some B. many C. others D. those 71. A. is B. are C. were D. was 72. A. for B. on C. at D. by 73. A. made B. addressed C. written D. sent 74. A. with B. for C. as D. by 75. A. producing B. giving C. making D. sending 76. A. time B. yet C. way D. enough 77. A. advise B. answers C. advice D. problems 78. A. at B. on C. in D. about 80. A. called B. named C. reached D. claimed 第二卷(共30分) Part ? Vocabulary (10,,10minutes) Directions: Complete each of the following sentences with the proper word derived from the one given in the bracket. 1. Everyone had an ____ (apply) form in his hand, but no one knew which office to send it to. 2. He found it ____ (increase) difficult to read, for his eyes were failing. 3. It is reported that ____ (identify) flying objects were seen over New Jersey last night. 4. Any ____ propose) that Norman should be dismissed must be resisted. 5. The reason why the water in this area had been polluted was the ____ (poison) substances had been poured into the river. 6. They are poor but ____ (respect). 7. Flowers are always a ____ (please) sight, especially in the city. 8. The bedrooms in his new flat are ____ (taste) decorated and furnished. 9. Hardly ever have I heard music as ____ (impress) as this. 10. They are proud their son has won the first prize in the physics ____ (compete). 39 Part ? Sentence Completion(20,,30minutes) Directions: Put the underlined part in each sentence into English. Section A 1. Both the cars 一出发就抛锚。 2. There was a large were house 在河对岸。 3. In the supermarket Afl,人人自助购物。 4. 不管谁犯了法 will be punished. 5. A few people葬身于火海,but most were saved. 6. 如果你听从我的劝告, you will probably succeed. 7. The next flight doesn't go directly to Rome,而是要绕道巴黎。 8. He is so foolish 竟让认敌为友 9. 此规定不适用于你,you are under 18. 10. 除了现在,我们任何时候都能去。 Section B Directions: Put the underlined part in each sentence into Chinese. 1. You will have to answer for your foolish behavior. 2. While respected, he is not liked. 3. As long as you clear your desk by this morning, you can have tomorrow off. 4. She and I liked an hour strolling in the street. 5. He loves keeping his own opinion to himself. 6. If we are discouraged at the start, we will never succeed. 7. What fruits are in season now? 8. A fall in a pit, a gain in your wit. 9. The general manager's secretary has been on the go all day. 10. By the time he was 18 years old, he had developed to the extent that he never listened to his parents' advice. Part ? Writing(20,) 注意:报考英语专业的考生必做,非英语专业的考生不必做。 Directions: For this part, you are allowed thirty minutes to write a composition on the topic "How to Solve the Problem of Heavy Traffic". You should write no less than 150 words and base your composition on the outline (given in Chinese) below: 1(为解决交通难的问题,有人建议多建造马路; 2(有人则建议限制私家车的数量; 3(我的看法。 2004年江苏省普通高校“专转本”统一考试 大学英语答案及解析 Part I Reading Comprehension 1.参考答案: B 详细解答: B。从第二段第一句―a lazy sailor…pretended to be ill(‖可知第一个sailor 不愿意从事船上的艰苦劳动而装病偷懒。 2.参考答案: C 详细解答: C。第二段中,那位好为人―医‖的船长并没有给出什么药方,只是told the man to rest for a few days and made the other sailors do his work 3.参考答案: D 详细解答: D。前两段叙述了船长装医生,两个船员装生病,其他船员很生气,因 此副船长在征得船长同意后为他们配了―药‖,那些药是如此horrible以致于那些船员无法装病。 4.参考答案: C详细解答: C。这道题的关键是理解单词deceive的意义:欺骗。 40 5.参考答案: A 详细解答: A。综合归纳文章内容,A为主题思想。 6.参考答案: D 详细解答: D。根据第一段第二句中with the aid of electricity,可知答案为D。 7.参考答案: C 详细解答: C。第二段第一句话。 8.参考答案: D 详细解答: D。A、B、C三个选项都能直接在文章的第三段中找到,只有D选项没有在文中找到。 9.参考答案: B 详细解答: B。最后一段第一句话中的非限制性定语从句是该题的题干。答案很显然应该是该非限制性定语从句所修饰的中心词:bauxite(铝土矿)。 10.参考答案: A 详细解答: A。本文介绍了铝的特点、性质以及用途,正确答案为A。 11.参考答案: D详细解答: D。第二段第三句话。observance(法律、习俗等的)遵守,奉行。因此正确答案为D。 12.参考答案: A 详细解答: A。文章第一段倒数第二句话,给出了答案。 13.参考答案: C 详细解答: C 14.参考答案: B 详细解答: B。用排除法得出答案为B。 15.参考答案: D 详细解答: D。根据文章第四段第三句中的一个状语从句―Though at first the British public was slow to accept this rather artificial day(‖即可得出正确答案。 16.参考答案: A 详细解答: A。对第一段culture shock定义的重新描述。 17.参考答案: C 详细解答: C。答案在第二段第一句,只不过答案把lose改写成了名词loss。 18.参考答案: C 详细解答: C。根据文章第二段所列举的那些现象,与四个选项对照,没有的即为正确选项。因为A、B、D都在文中的一处找到,而C找不到。 19.参考答案: A 详细解答: A。首先可以排除B、D两个选项,关于A和C,答案在最后一段:When suffering from culture shock people first reject the environment which caused discomfort如果了解短语take…for granted(想当然的认为)就很容易排除选项C。 20.参考答案: B 详细解答: B。归纳文章主要内容,得出B为文章中心思想。 Part? Vocabulary and Structure 21.参考答案: D 详细解答: D。take…on a tour带某人参观。 22.参考答案: B 详细解答: B。bring up抚养,养育;grow up长大;feed up喂养。 23.参考答案: A 详细解答: A。keep in touch保持联系 24.参考答案: D 详细解答: D。同位语从句,由that引导。 25.参考答案: A 详细解答: A。incident特指(政治性的、国际性的)事件,争端;event指大的有深远意义的事件。 26.参考答案: C 详细解答: C。put...into practice实施,把……用于实践中。 27.参考答案: D 详细解答: D。enough用法:作副词修饰形容词或副词时,位于所修饰词之后。 28.参考答案: C 详细解答: C。否定词位于句首,主句部分倒装,助动词、情态动词或补充助动词提前。 29.参考答案: A 详细解答: A。时间状语从句。since自从…… 30.参考答案: B 详细解答: B。既引导主语从句,又在该主语从句中充当主语成分,只有what具有这一功能。 31.参考答案: B 详细解答: B。与过去事实相反的虚拟语气。条件状语从句隐含在other wise中。 32.参考答案: C 详细解答: C。固定搭配:have no choice but to do除了……没有选择。注意如but前含有实义动词do及其不同时态、语态,则不定式的to省略。 33.参考答案: B 详细解答: B。句中为survive及物动词的用法。 34.参考答案: A 详细解答: A。way of living生活方式;method偏重于―方法‖。 35.参考答案: C 详细解答: C。首先排除B、D,其次,the most应直接接名词。 36.参考答案: C 详细解答: C。强调句。 41 37.参考答案: C 详细解答: C。省略式状语从句中的被动形式。 38.参考答案: B 详细解答: B。固定搭配:inform sb. of sth.就某事通知某人。 39.参考答案: D 详细解答: D。不定式十完成时,因miss这个动作发生在主句动作之后。 40.参考答案: C 详细解答: C。首先排除D;其次A、B皆为及物动词。 41.参考答案: A 详细解答: A。bare指光秃秃的,表面没有其他东西的;empty指(内部为)中空的;blank指空白的,空虚的;vacant(职位等)空缺的。 42.参考答案: C 详细解答: C。poll指民意测验,通常还可说public opinion poll。 43.参考答案: C 详细解答: C。by表示方法、手段、数量、程度等。 44.参考答案: A 详细解答: A。双重否定。 45.参考答案: A 详细解答: A。be accused of被指控……;charge表示该意义的用法时后接with。 46.参考答案: D 详细解答: D。once 一旦… 47.参考答案: D 详细解答: D。固定短语account for解释,说明;count on指望,依靠。 48.参考答案: A 详细解答: A。make up for弥补;make out辨认出,弄明白。 49.参考答案: A 详细解答: A。come into effect奏效,实施,起作用。 50.参考答案: C 详细解答: C。固定短语:be composed of由…组成。 51.参考答案: B 详细解答: B。education为不可数名词,只能用little修饰;其次so与such的用法为so +adj./adv. , such +a +adj. +n. 52.参考答案: D 详细解答: D。prevent… from…与shoot at为固定短语。 53.参考答案: A 详细解答: A。natural gas和wind只能归类于energy能量。 54.参考答案: A 详细解答: A。主要难点在A和B,take place通常指事情有计划地发生。自然现象是不可能用take place来表示―发生‖的。 55.参考答案: D 详细解答: D。事实胜于雄辩。 56.参考答案: A 详细解答: A。让步从句中的倒装句。 57.参考答案: C 详细解答: C。vast强调的是(在尺寸、数目、数量或份量上或在区域、宽度上甚至是程度或强度上)―很大的,广大的‖;striking通常指(外表、效果等)―引人注意的,显著的‖;considerable指的是(在数目、变化上)―客观的,相当……的‖;extreme含义是―极端的,非常的‖。 58.参考答案: A 详细解答: A。set短语辨析;set aside留出(时间、金钱等)。 59.参考答案: B 详细解答: B。固定短语:take notice of与notice同义,―注意‖。 60.参考答案: D 详细解答: D。非限制性定语从句,用which引导。 Part III Cloze 61.参考答案: B 详细解答: B。上文中提到advice,此处是向人请教,用ask。实际上,该句暗含ask sb(for sth.短语。 62.参考答案: B详细解答: B。上下文理解。前面说美国人不想向自己认识的人寻求意见,后文说他们更愿意找报纸、杂志来征求意见,两者之间用instead(取而代之)连接。 63.参考答案: C 详细解答: C。介词on表示―关于‖,如:a book on radio。 64.参考答案: D 详细解答: D。of表示从属关系。 65.参考答案: A 详细解答: A。根据上下文用most比较客观。 66.参考答案: A 详细解答: A。表示来源用from,因为后面接的是with problems。 67.参考答案: B 详细解答: B。along with和…一起。 68.参考答案: D 详细解答: D。by表示写信的人。可由people后接的定语从句得到提示。 69.参考答案: D 详细解答: D。解决problem只能用动词solve或tackle。 70.参考答案: C详细解答: C。some…others… 一些人……,另外一些人……,固定句式。 71.参考答案: B 详细解答: B。主谓数的一致。 72.参考答案: A 详细解答: A。介词for表示培训的科目。 42 73.参考答案: B 详细解答: B。address to固定搭配:在(信封、包裹)上写姓名、地址等。 74.参考答案: C 详细解答: C。be addressed as用……称呼(某人)。 75.参考答案: B 详细解答: B。give advice to sb.给某人忠告。 76.参考答案: D 详细解答: D。enough修饰形容词,放在形容词之后。 77.参考答案: C 详细解答: C。根据上下文可知该小女孩是一位向别人提供忠告的人,因此用advice。 78.参考答案: A 详细解答: A。at the age of在……年龄。 79.参考答案: C 详细解答: C。根据上文所述,该小女孩的忠告regularly定期地―出现‖在报纸上, 并以专栏形式(in a column)刊登。 80.参考答案: A 详细解答: A。前两个选项的区别:name指―取名,命名‖,且常用被动式短语 be named after; call称为。 Part ? Vocabulary 1. application 2. increasingly 3. unidentified 4. proposal 5. poisonous 6. pleasant 7. tastefully 8. tastefully 9. impressive 10. competition Part ? Sentence Completion Section A 1. broke down immediately(as soon as) they set out. 2. on the other side of the river 3. everyone serves himself(herself). 4. Whoever breaks the law 5. were killed in the fire 6. If you follow my advice 7. but goes by way of Paris 8. as to regard enemies as friends.或as to take enemies for friends. 9. This regulation doesn't apply to you.或This rule is not suitable for you. 10. I can go any time but/except now. Section B 1.对你愚蠢的行为负责。 2.虽然受到尊重,但他并不受人喜爱 3.明天你可以放假,明天你可以休息一天。 4.消磨了一个小时 5.把自己的意见埋在心里。 6.如果我们一开始就泄气,失去信心 7.现在的时令水果是什么, 8.吃一堑,长一智。 9.整天不停地忙着 10. 他已经发展到从不听自己父母意见的地步。 Part ? Writing How to Solve the problem of Heavy Traffic Due to the decrease of cost and the increasing support from governments, city people are finding themselves surrounded by more and more traffic. Heavy traffic has become a common reason for being late to work. To tackle this problem, people are racking their brains to come up with all kinds solutions. some suggest that more roads be built for the increasing traffic,and others propose putting a limit on the amount of private cars. In my opinion, the fundamental way to solve this problem is to perfect and speed up the public transportation system. If faster transportation like magnetic suspension train is very convenient, more and more people will abandon the idea of purchasing private cars and resort to public transportation. The public transport system in New York has set a good example for other cities. The expensive parking fees, and more importantly,the dense routes of buses and subway make It possible that people live far from their work place. Therefore,I suggest the government set aside more investment for faster and more convenient transport system in order to solve the problem of heavy traffic. 43 2005年江苏省普通高校“专转本”统一考试试卷 大学英语 第?卷(共100分) 注意事项: 1(答第?卷前,考生务必将自己的姓名、准考证号、考试科目用铅笔涂写在答题卡上。 2(每小题选出答案后,用2B铅笔把答题卡上对应题目的答案标号涂黑。如需改动,用橡皮擦干净后,再 选涂其他答案标号。如果答案不涂写在答题卡上,成绩无效。 Part I Reading Comprehension (40,,35minutes) Directions: In this part there are four passages. Each passage is followed by a number of comprehension questions. Read the passage and answer the questions. Then mark your answer by blackening the corresponding letter on the Answer Sheet. Passage 1 Human feelings are affected by color unconsciously. Manufacturers have discovered by trial and error that sugar sells badly in green wrappings, that blue foods are considered by consumers as tasteless, and that cosmetics should never be packaged in brown. These discoveries have grown into a whole discipline of color psychology. Some of our preferences are clearly psychological. Dark blue is the color of the night sky and therefore associated with calm, while yellow is a key color in association with energy. For primitive men. activity during the day meant bunting and attacking, in which he soon saw as red, the color of blood and hunting and fire. So it was natural that green, the complementary color to red, should be associated with passive defense. Experiments have shown that colors also have a direct psychological effect. People, when exposed to bright red. show an increase in breathing rate. heart beat and blood pressure. Red is exciting. Similar exposure to pure blue has exactly the opposite effect, because it is a calming color. Because red has an implication of excitement, it is chosen as the signal for danger. Some analyses show that a vivid yellow can produce a more basic state of alertness and alarm. So fire engines and ambulances in some advanced countries are now rushing around in bright yellow colors that stop traffic dead. 1. If people are exposed to red, which of the following statements does NOT happen? A. They feel afraid. B. They breathe faster. C. Their blood pressure rises. D. Their hearts beat faster. 2. Manufacturers have discovered the secret of colors in marketing ____. A. by experimenting with different colors B. by developing the discipline of color psychology C. by trying not to make mistakes D. by accumulating their various experiences 3. Which of the following is NOT true? A. Our feelings about certain colors are purely psychological. B. Food should never be packaged in brown. C. Sugar sells badly in green wrappings, D. Color probably has an effect on us which we are not conscious of. 4. Our preferences for certain colors are ____. A. dependent on our character B. linked with the primitive men C. associated with psychology D. associated with the time of the day 5. The passage is about ____. A. color and traffic accidents B. color and manufacturers C. which color might influence human feelings D. why color affects human emotions and behavior Passage 2 44 Exchange a glance with someone, and then look away. Do you realize that you have made a statement? Hold the glance for a second longer, and you have made a different statement. Hold it for three seconds, and the meaning has changed again. For every social situation, there is permissible time that you can hold a person's gaze without being intimate, rude, or aggressive. If you are on an elevator, what gaze-time are you permitted? To answer this question, consider what you typically do. You are very likely give other passengers a quick glance to size them up(打量) and to assure them that you mean no threat. Since being close to another person signals the possibility of interaction, you need to emit a signal telling others you want to be left alone. So you cut off eye contact, which sociologist Erving Goffman ( 1963 ) calls "a dimming of the lights". You look down at the floor, at the indicator lights, anywhere but into another passenger's eyes. Should you break the rule against staring at a stranger on an elevator, you will make the other person exceedingly uncomfortable, and you are likely to feel a bit strange yourself. If you hold eye contact for more than three seconds, what are you telling another person? Much depends on the person and the situation. For instance, a man and a woman communicate interest in this manner. They typically gaze at each other for about three seconds at a time, and then drop their eyes down for three seconds, before letting their eyes meet again. But if one man gives another man a three-second-plus stare, he signals- "I know you." "I am interested in you." or "You look peculiar and I am curious about you." This type of stare often produces hostile feelings. 6. It can be inferred from the first paragraph that ____. A. every glance has its significance B. staring at a person is an expression of interest C. a gaze longer than three seconds is unacceptable D. a glance conveys more meaning than words 7. If you want to be left alone on an elevator, the best thing to do is ____. A. to look into another passenger's eyes B. to avoid eye contact with other passengers C. to signal you are nor a threat to anyone D. to keep a distance from other passengers 8. By "a dimming of the lights"(Line 9. Para l ). Erving Goffman means"____" A. closing one's eyes B. turning off the lights C. ceasing to glance at others D. reducing gaze-time to the minimum 9. If one is looked at by a stranger for too tong, he tends to feel ____. A. depressed B. curious C. uneasy D. amused 10. The passage mainly discusses ____. A. the limitations of eye contact B. the exchange of ideas through eye contact C. proper behavior in various situations D. the role of eye contact in interpersonal communication Passage 3 Cyclo-cross is a cross-country bicycle racing in open and usually quite rough country with riders often forced to dismount and carry their bicycles. The sport, originated early in the 20th century in France. was prevalent in the 1920s, but became prominent in the 1950s ( the British Cyclo-Cross Association was founded in 1954). An original European sport, cyclo-cross became popular throughout Western Europe and in the United States. World championships were initiated in 1925 ; by 1950 these were recognized by the Union Cyclist International(International Cyclists' Union). After 1967 45 amateur and professional classes were officially separated in competition. The 24-kilometer cyclo-cross course, often involving taps, is usually completed in 60 minutes. A course typically includes obstacles such as ditches, mud, fallen trees, streams, flight of stairs, fences, and gates; artificial hurdles are added to insufficiently challenging natural courses. Cyclo-cross races are usually held from September to March, adding winter weather hazards to the challenge. There is a massed start with the field assembling not more than two abreast. Helpers are often stationed around the course with spare bicycles in case the original machine encounters mechanical difficulties or becomes too weighted down by mud picked up to the course. 11. What does the word "dismount" in Paragraph One mean? A. Give up B. Give in C. Get our D. Get off. 12. According ro the passage, which of the following is NOT true about cyclo-cross? A. The cyclists sometimes have to carry the bicycles instead of riding them. B. The riders in the race are arranged side by side in two lines. C. All the riders start simultaneously on the starting line. D. The riders are allowed to change their machines, 13. Cyclo-cross became widely known ____. A. in the 1920s B. in the 1950s C. in 1925 D. in 1967 14. According to the passage, cyclo-cross races are usually held from September to March because ____. A. A. the winter is westerners' favorite season B. winter can offer cyclists more risk and danger C. the winter weather is more agreeable D. in winter the riders needn't dismount and carry their bicycle 15. Which of the following statements can you infer from the passage? A. The riders are competitive and fond of taking risks. B. People except the riders show little interest in cyclo-cross. C. Before 1967, amateur and professional classes had never been separated in competition. D. Helpers are often hired by the riders. Passage 4 In the second half of each year. many powerful storms are born in the tropical Atlantic and Caribbean seas. Of these, only about half a dozen generate the strong, circling winds of75 miles per hour or more that give them hurricane status, and several usually make their way to the coast. There they cause millions of dollars of damage, and bring death to large numbers of people. The great storms that hit the coast start as innocent circling disturbances hundreds even thousands of miles out to sea. They travel aimlessly over water warmed by the trade winds. When conditions are just right, warm moist air flows in at the bottom of such a disturbance, moves upward through it and comes out at the top. In the process, the moisture in this warm air produces rain, and with it the heat is converted to energy in the form of strong winds. As the heat increases, the young hurricane begins to swirl in a counterclockwise motion. The average life of a hurricane is only about nine days, but it contains almost more power than we can imagine. The energy in the heat released by a hurricane's rainfall in a single day would satisfy the entire electrical needs of the United States for more than six months. Water, not wind, is the main source of death and destruction in a hurricane. A typical hurricane brings 6 to 12 inch downpours resulting in sudden floods. Worst of all is the powerful movement of the sea-mountains of water moving toward the low-pressure hurricane center. The water level rises as much as 15 feet above normal as it moves toward shore. 16. When is an ordinary tropical storm considered to be a hurricane? 46 A. When il begins in the Atlantic and Caribbean seas. B. When it hits the coastline. C. When its winds reach 75 miles per hour or even more. D. When the circling winds bring damages. 17. What is the worst thing about hurricanes? A. The destructive effects of water. B. The heat they release. C. That they last about nine days on the average. D. Their strong winds. 18. The counterclockwise swirling of the hurricane is brought about by ____. A. the low-pressure area in the center of the storm B. the force of waves of water C. the trade winds D. the increasing heat 19. Apparently the phrase "innocent circling disturbances" in Paragraph Two means ____. A. disturbances caused by innocent people B. disturbances people are innocent of C. damaging circling winds D. harmless circling winds 20. Which of the following best summarizes the passage? A. The Hurricane-Its Harms and Benefits. B. The Benefits of Hurricanes. C. The Hurricane and Its Great Energy. D. The Disaster Caused by Hurricanes. Part? Vocabulary and Structure(40,,20minutes) Directions: There are 40 incomplete sentences in this part. For each sentence there are four choices marked A, B, C and D. Choose the ONE answer that best completes the sentence. Then mark the corresponding letter on the Answer Sheet with a single line through the center. 21. ____, I am afraid I can't go with you. A. With so much work to do B. With so much work doing C. For so much work to do D. To do so much work 22. He believes that happiness ____being easily pleased or satisfied. A. consists of B. consists with C. consists by D. consists in 23. They will ____the applications and pick out the best. A. look into B. look after C. look at D. look through 24. I can't ____books like those-they just make me sleepy. A. carry B. suffer C. appreciate D. take 25. Jean worked just so much ____. A. like what she was told B. as she was told to C. as to what she tried to do D. like she was told to 47 26. I don't remember the boiler ____during these years. A. being repaired B. having been repaired C. to be repaired D. to have been repaired 27. A successful business certainly sells its products ____. A. at last B. at best C. at any rate D. at a profit 28. The professor's speech ____at wide range of subjects from drug to women's rights. A. contained B. covered C. listed D. touched 29. In this factory the machines are not regulated ____but are jointly controlled by a central computer system. A. independently B. individually C. similarly D. irregularly 30. They did not sell a single car for a month and had to ____workers. A. lay down B. lay behind C. lay off D. lay out 31. There has been a great increase in retail sales, ____? A. does there B. hasn't there C. isn't there D. isn't it 32. Stormy applause broke forth ____the singer appeared on the stage. A. a moment B. the moment C. in a moment D. at the moment 33. Not until the game had begun ____at the sports ground. A. had he arrived B. would he have arrived C. did he arrive D. should he have arrived 3. ____evidence that language-acquiring ability must be stimulated. A. It being B. It is C. There is D. There being 35. Jenny is not in the ____for going to the party tonight. A. tempers B. condition C. mood D. health 36. Although Anne is happy with her success, she wonders ____will happen to her private life. A. that B. what C. it D. this 37. The old buildings blend with the new ones in perfect ____, so a walk along Huaihai Road is an enjoyable discovery. A. harmony B. order C. control D. action 38. ____he was not in uniform, he carried a pistol under his arm. A. Even when B. As long as C. As soon as D. Ever since 39. Not only I but also Tom and Mary ____fond of collecting stamps. A. am B. will C. are D. have 40. Marge's bedroom was in a ____, with books and papers covering every possible surface. A. order B. mess C. rubbish D. disorder 41. John, you are so lazy. This job ____hours ago. A. should finish B. must have finished C. could be finishing D. ought to have been finished 42. ____you are leaving tomorrow, we can eat dinner together tonight. A. For B. Since C. Before D. While 43. Why ____did you take the dog into the churchyard? A. on earth B. on the earth 48 C. in earth D. in the earth 44. My suggestion is that the experiment ____in another way. A. is done B. will be done C. has been done D. be done 45. Hangzhou is famous for the beautiful ____of West Lake. A. scenes B. views C. sight D. scenery 46. Thank you for your postcard; it was very ____of you to send it. A. pleased B. conscious C. responsible D. considerate 47. I really appreciate ____to help me, but I am sure that I can manage by myself. A. you to offer B. that you offer C. your offering D. that you are offering 48. The twins are so much ____that it is difficult to tell one from the other. A. similar B. alike C. same D. like 49. The revolutionary government acts on ____of the masses and against the privileged few. A. benefit B. sake C. behalf D. advantage 50. The men will have to wait all day ____the doctor works faster. A. if B. whether C. unless D. that 51. Have you got access ____people who can help you get work? A. of B. to C. with D. among 52. "Did you hear the phone ring last night?" "I didn't hear anything. I was ____that I didn't wake up. " A. very tired B. such tired C. so tired D. enough tired 53. There is no ____in applying for that job, as you are not properly qualified. A. reason B. point C. result D. mean 54. They climbed to the top of the hill ____they could get a bird's-eye view of the city. A. for fear that B. in order that C. in case D. as a result 55. She had clearly no ____of doing any work, although she was very well paid. A. interest B. meaning C. intention D. willingness 56. A lorry ____Jane's cat and sped away. A. ran over B. ran through C. ran into D. ran down 57. His remarks left me ____about his real purpose. A. wondered B. wonder C. to wonder D. wondering 58. The ____crown jewels are kept in the Tower of London. A. valued B. valueless C. invaluable D. worthy 59. Kenya's Tsavo Game Park ____Susan of the wildlife park she had visited in New Jersey. A. reminded B. recalled C. received D. remembered 60. Professor Wang, ____for his informative lectures, was warmly received by his students. A. knowing B. known C. to be known D. having known Part III Cloze(20,,15minutes) Directions: There are 20 blanks in the following passage. For each blank there are four choices marked A, B, C and D. You should choose the ONE that best fits into the passage. Then mark the corresponding letter on the Answer Sheet with a single line through the center. 49 The native Americans, the people we call the "Indians", had been in America for many thousands of years before Christopher Columbus arrived 61 1492. The Indians were kind to the early 62 . They taught them about the local 63 like sweet potatoes, com and peanuts. But soon the settlers wanted bigger farms and 64 land for themselves and their families. So the Europeans started to 65 the land from the Indians. Naturally, 66 the whites started taking all the Indians' land, the Indians started 67 back. But the whites were stronger and 68 . Slowly they pushed the Indians 69 those parts of the continent that the whites didn't want-the parts 70 it was too cold or too dry or too mountainous to live. By 1875, the Indians were living in a(n) 71 place called "reservation", 72 even here the whites took land from them-perhaps the whites wanted the wood, or perhaps 73 had important minerals in it, 74 the whites even wanted to make national parks there. So 75 on their reservations the Indians were not safe from 76 . There are many Hollywood films about the fight 77 the Indians and the whites. 78 in these films the Indians are bad and the whites are good and 79 . But was it 80 like that? 61. A. at B. in C. for D. since 62. A. residents B. builders C. emigrants D. settlers 63. A. plants B. animals C. crops D. grains 64. A. more B. many C. much D. mere 65. A. take B. carry C. fetch D. get 66. A. before B. if C. when D. however 67. A. complaining B. fighting C. struggling D. moving 68. A. firmer B. larger C. taller D. cleverer 69. A. hack B. in C. off D. into 70. A. which B. that C. where D. when 71. A. special B. strange C. standard D. odd 72. A. For B. But C. As D. Just 73. A. the land B. they C. the wood D. the reservations 74. A. for B. and C. or D. because 75. A. ever B. just C. only D. even 76. A. dangers B. the whites C. reservations D. animals 77. A. between B. with C. for D. against 78. A. In fact B. Likely C. Usually D. In time 79. A. humorous B. brave C. careful D. cruel 80. A. probably B. readily C. possibly D. really 第二卷(共50分) Part ? Translation (35,,25minutes) Section A (20,) Directions: Translate the following sentences into Chinese. You may refer to the corresponding passages in Part I. 1. These discoveries have grown into a whole discipline of color psychology. 2. So fire engines and ambulances in some advanced countries are now rushing around bright yellow colors that stop traffic dead. 3. For every social situation, there is a permissible time that you can hold 8 person's gaze without being intimate, rude, or aggressive. 4. Cyclo—cross races are usually held from. September to March adding winter weather hazards to the challenge. 5. In the process,the moisture in this warm air produces rain. and with it the heal is converted to 50 energy in the form of strong winds. Section B (15,) Directions: Translate the following sentences into English. 1. 学习语言最好的方法是尽可能常练习说。 2. 我们本来可以不犯这样的错误的,只是我们没有接受他的忠告。 3. 只要我们不灰心,我们就能找到克服困难的方法。 4. 委员会没能就一项政策达成一致意见,所以决定于下个月再开会。 5. 我们应该努力学习,因为不努力学习的人是不能取得伟大成就的。 Part ? Writing (15,,25minutes) Directions: For this part, you are given 25 minutes to write a letter to call on university students to help those people in need. You should write no less than 120 words and base your composition on the out line (given in Chinese) below: 1. 倡议书涉及的具体帮助对象; 2. 实施帮助的理由及意义; 3. 具体 措施 《全国民用建筑工程设计技术措施》规划•建筑•景观全国民用建筑工程设计技术措施》规划•建筑•景观软件质量保证措施下载工地伤害及预防措施下载关于贯彻落实的具体措施 或安排活动。 2005年江苏省普通高校“专转本”统一考试 大学英语答案及解析 Part I Reading Comprehension 1. 参考答案:A 详细解答: A。根据―People,when exposed to bright red, show an increase in breathing rate, heart beat and blood pressure.‖可知。 2. 参考答案:D 详细解答: D。根据第二句―Manufacturers have discovered by trial and error that...‖ 可知,生产商是在不断的试验中积累起经验来的。 3. 参考答案:B 详细解答: B。由―…that blue foods are considered by consumers as tasteless,…‖可知B项错误。 4. 参考答案:C 详细解答: C。由―Some of our preferences are clearly psychological(‖可知C项正确。而且,前文作者所举例子均在说明人们对颜色的心理态度。 5. 参考答案:D 详细解答: D。本文旨在通过说明颜色对人直接的生理影响阐述颜色为什么会影响到人们的情绪和行为。 6. 参考答案:A 详细解答: A。由前四句话可知,每一个眼神都有其特定的含义。 7. 参考答案:B 详细解答: B。由第一节倒数两句可知。 8. 参考答案:C 详细解答: C。与前文的―cut off eve contact‖同义。 9. 参考答案:C 详细解答: C。由第一节最后一句可知。 10. 参考答案:D 详细解答: D。本文主要讨论人与人之间眼神交汇的作用。 11. 参考答案:D 详细解答: D。据―rough country‖和―carry their bicycles‖可以推断出―下车‖的意思。 12. 参考答案:C 详细解答: C。本题可采用排除法。 13. 参考答案:A 详细解答: A。由―…was prevalent in the 1920s.…‖可知。 14. 参考答案:B 详细解答: B。由文中第三节最后一句话可知。 15. 参考答案:A 详细解答: A。可从文中所述的障碍可知,也可采用排除法。 16. 参考答案:C 详细解答: C。由第一节第二句可知。 17. 参考答案:A 详细解答: A。由第三节第三句可知,风暴带来的水是造成死亡和破坏的主要原因。 18. 参考答案:D 详细解答: D。由第二节最后一句―As the heat increases,…‖可知。 19. 参考答案:D 详细解答: D。由文意可推知,袭击海岸的大风暴始于―没有破坏力的气旋‖。 20. 参考答案:C 详细解答: C。本文主要是讨论风暴给人类带来的灾难。 51 Part? Vocabulary and Structure 21. 参考答案:A 详细解答: A。伴随状语表明原因,排除C和D。to do表尚未完成。 22. 参考答案:D 详细解答: D。consist of由……组成;consist with与……一致;consist in在于。 23. 参考答案:D 详细解答: D。look into调查;look after照顾;look at看;look through翻阅,细审。 24. 参考答案:C 详细解答: C。由题意可推知作者对那些书的态度——无法―欣赏‖。 25. 参考答案:B详细解答: B。as引导的定义从句。 26. 参考答案:B 详细解答: B。remember doing sth(表记得过去的事情,所以排除C和D。而A项表示这几年一直。在维修,不合逻辑。 27. 参考答案:D 详细解答: D。at last最后;at best最好;at any rate不管以什么样的速度;at a profit盈利地。 28. 参考答案:B 详细解答: B。cover包含,涵盖。 29. 参考答案:B 详细解答: B。与jointly形成对比,选择B。 30. 参考答案:C 详细解答: C。lay down放下;lay behind落后于,在……背后;lay out陈设,展开。 31. 参考答案:C 详细解答: C。反意疑问句。主句的实义动词是be,所以选择C。 32. 参考答案:B 详细解答: B。由the moment引导时间状语。 33. 参考答案:C详细解答: C。Not until引导时间状语开头,主句倒装。没有完成时态和虚拟语气,所以选择C。 34. 参考答案:C详细解答: C。evidence后接一个同位语从句,因此选用there be句型的主句用法。 35. 参考答案:C详细解答: C。in the mood for sth有做某事的心情。 36. 参考答案:B 详细解答: B。宾语从句中what作主语。 37. 参考答案:A 详细解答: A。in harmony协调;in order有序;in control得到控制;in action在行动。 38. 参考答案:A 详细解答: A。as long as只要;as soon as一……(就……);ever since自从;选择A表示即便当……的时候。 39. 参考答案:C 详细解答: C。采取就近原则。 40. 参考答案:B 详细解答: B。In a mess乱糟糟。 41. 参考答案:D 详细解答: D。情态动词取―应该‖之意,主谓符合被动语态。 42. 参考答案:B 详细解答: B。for因为;Since既然;before在……之前;while当……的时候,或表示对比。 43. 参考答案:A 详细解答: A。on earth到底,究竟。 44. 参考答案:D详细解答: D。suggestion后面的从句用虚拟语气should do sth(,should可以省略。 45. 参考答案:D 详细解答: D。scene场景;view观点,眼界;sight视力,目光;scenery景色。 46. 参考答案:D 详细解答: D。pleased高兴的;conscious明白的;responsible负责任的;considerate考虑周到的。 47. 参考答案:C 详细解答: C。appreciate后接名词短语或宾语从句,B和D时态不对。 48. 参考答案:D 详细解答: D。形容两个人相像时用alike,而不用similar( 49. 参考答案:C 详细解答: C。on behalf of代表。 50. 参考答案:C 详细解答: C。unless除非。 51. 参考答案:B 详细解答: B。get access to有机会接近。 52. 参考答案:C 详细解答: C。so…that…太……以致于…… 53. 参考答案:B 详细解答: B。point目的,理由。 54. 参考答案:B 详细解答: B。in order that引导目的状语。 55. 参考答案:C 详细解答: C。have no intention of doing sth(没有打算做某事。 56. 参考答案:A 详细解答: A。run over撞到并轧到其身体的一部分;run through匆匆浏览;run into碰撞;run down破败,跨掉,疲乏。 52 57. 参考答案:D 详细解答: D。leave sb(/sth(doing sth(让某人,某物做某事。 58. 参考答案:C 详细解答: C。valued被珍视的;valueless毫无价值的;invaluable珍贵的;worthy值得的。 59. 参考答案:A 详细解答: A。remind sb(of sth(使某人想起某事。 60. 参考答案:B 详细解答: B。分词短语作状语,动词和主语是被动关系。 Part III Cloze 61.参考答案: B 详细解答: B。表示年份用in。 62.参考答案: D 详细解答: D。下一句有settlers。 63.参考答案: C 详细解答: C。所举例子说明是指庄稼。 64.参考答案: A 详细解答: A。和前面的bigger并列+应该也是比较级。 65.参考答案: A 详细解答: A。take夺取。 66.参考答案: C 详细解答: C。白人抢地和印第安人反抗应该是同步的。 67.参考答案: B 详细解答: B。fight back抗争,反抗。 68.参考答案: D 详细解答: D。赢得战争不仅需要更健壮的体格,还需要更灵活的脑子。 69.参考答案: D 详细解答: D。push...into_把……推,逼进…… 70.参考答案: C 详细解答: C。定语从句,连接词在从句中作地点状语。 71.参考答案: A 详细解答: A。―保留地reservation‖应是―特别的‖,而不是奇怪的,或 标准 excel标准偏差excel标准偏差函数exl标准差函数国标检验抽样标准表免费下载红头文件格式标准下载 的。 72.参考答案: B 详细解答: B。even here暗示了前后的转折关系。 73.参考答案: A 详细解答: A。根据上文―the whites took land from them‖可知。 74.参考答案: C 详细解答: C。和上一分句并列。 75.参考答案: D 详细解答: D。较原来的土地上,印第安人的保留地意思更进一步。 76.参考答案: B 详细解答: B。威胁印第安人的是whites。 77.参考答案: A 详细解答: A。between连接fight涉及的双方。 78.参考答案: C 详细解答: C。根据事实,好莱坞的电影中―通常‖白人是好人,印第安人是坏人。 79.参考答案: B 详细解答: B。正面描述白人抢夺印第安人地盘的词。 80.参考答案: D 详细解答: D。作者怀疑好莱坞电影的立场。 Part ? Translation Section A 1.这些发现已经发展成一门完整的学科——色彩心理学。 2.所以,在一些发达国家,消防车和救护车的颜色为鲜黄色。当他们呼啸而过时,其他车辆都要为之让行。 3.对于每一种社交场合,你盯着对方凝视的时间不能超过一定的限度,否则你的举止会显得过于亲密、粗鲁或具有攻击性。 4.通常,自行车越野赛在9月至次年3月间举行。冬天恶劣的气候使比赛更具挑战性。 5. 在这一过程中,暖空气中的水汽变成了雨,热量随之转化成另一种形式的能——强风。 Section B 1. The best wav to learn a language is to practice speaking it as often as possible. 2. We could have avoided making such a mistake,but we didn't follow his advice. 3. So long as we don't lose heart,we'll find a way to overcome the difficulty. 4. The committee was unable to agree on a policy,so it decided to meet again next month. 5. We should study hard,because no man who does not study hard can achieve great success. Part ? Writing Dear fellow students. I'm writing on behalf of the people in need to ask for help. As a sensitive human being, I feel very bad every 53 time l see a child begging m the street; as a sensible university student, I strongly feel that we university students should do something for the kids who are not receiving education like us and are living a miserable life in our city. More and more people are coming to live in cities from the country. They are not living a good life, however. They are still very poor compared with the local people. Their children are unable to get education and to get a good job later. Therefore, this may make our society unstable and make their life harder. If we help the children get education just as we do, I believe all the people will find themselves in a better and better life. To help those kids, I suggest we establish a foundation with donations from all sources. We talk to the parents of those children, persuade them to have their children go back to school, and provide some financial aid for them. I believe we will have all the children in school if we try. Yours sincerely. 2006年江苏省普通高校“专转本”统一考试试卷 大学英语 第一卷(共100分) 注意事项: 1(答第1卷前,考生务必按规定要求填涂答题卡上的姓名、准考证号等项目。 2(用铅笔把答题卡上相应题号中正确答案的标号涂黑。答案不涂写在答题卡上,成绩无效。 Part I ReadingComprehension(共20小题,每题2分,共40分) Directions:In thispart there are fourpassages(Each passage is followed by four comprehension questions(Read the passage and answer the questions(Then mark your answer On the answer sheet( Passage One Questions 1 to 5 are based on the following passage: Vegetable gardening is the relaxing art and science of turning a love for growing plants into a meaningful activity. Having vegetables in the backyard makes home gardening appealing for many people. In addition, vegetable gardening provides exercise and fun for both urban and suburban families. Vegetable gardeners agree that many home-grown vegetables are superior to thosepurchased from markets. From spring through late fall, a well-planned and well-kept garden can provide a supply of fresh vegetables, thus increasing the nutrition of the family diet. Freezers make it possible to keep some of the vegetables to be enjoyed at a later date. Other vegetables can be stored for a few months in a cool area. Although the money spent for a garden may be little, one cannot escape the fact thatgardening requires hard work and time. Many of the gardening tasks must be performed at times that are most inconvenient. Not doing jobs that should be done on a regular basis may result in failure and a negative feeling toward gardening. One should not plant a garden that is too large for him to care for. A small, well-kept garden is more enjoyable and profitable than a large neglected one. Vegetables do well in full sunlight and need at least five or six hours of sun during the middle of the day. Too much shading results in poor plants and few vegetables. If possible, the garden should be near the house, so the gardener can work in it whenever they are free. Soils for vegetables should be easily broken up and porous (多孔的 ) for quick water drainage and good aeration (透气). Usually the home owner has little choice in the soil type he can choose. Fortunately, many vegetables can be grown on poor soils if the soils are properly prepared. 1. Many people find home gardening____. A. expensive and boring B. interesting and enjoyable C. time-consuming and inconvenient D. neither interesting nor enjoyable 54 2. A well-planned and well-kept garden can provide a supply of fresh vegetables____. A. all year round B. whenever you need nutritious food C. for a greater part of the year D. throughout spring 3. The amount of money you spend on your garden ____. A. can be large B. must be large C. can not be large D. must not be small 4. Which of the following statements is true? A. Vegetables need to have as much shading as possible: B. A small garden is much better than a large one. C. Homeowners can freely choose the soi for their vegetable garden. D. Whenever possible, the vegetable garden should be close to your home. 5. From the passage we can conclude that____. A. vegetables need little care while growing B. vegetables can be grown on almost any kind of soil C. vegetable gardening is an indispensable part of life for urban and suburban people D. vegetable gardening is a good pastime for retired people Passage Two Questions 6 to 10 are based on the following passage: In old days, when a glimpse of stocking was looked upon as something so shocking as to distract the serious work of an office, secretaries were men. Then came the First World War and the male secretaries were replaced by women. A man's secretary became his personal servant, in charge of remembering his wife's birthday and buying her presents; taking his suits to the dry-cleaners; telling lies on the telephone to keep away people he did not wish to speak to; and, of course, typing and filing and taking shorthand. Now all this may be changing again. The microchip (芯片 ) and high technology is sweeping the British office, taking with it much of the routine clerical (文书的) work that secretaries did. "Once office technology takes over generally, the status of the job will rise again because it will involve the high-tech work-and then men will want to do it again." That was said by one of the executives (male) of one of the biggest secretarial agencies in this country. What he has predicted is already under way in the U. S. Once high technology has made the job of secretary less routine (泛味的 ), will there be a male takeover? Men should be careful of thinking that they can walk right into the better jobs. There are a lot of women secretaries who will do the job as well as men--not just because they can buy negligees (妇女长睡衣) for the boss's wife, but because they are as efficient and well-trained to cope with word processors and computers as men. 6. Before 1914 female secretaries were rare because they____. A. were less efficient and less trained than men B. were looked down upon by men C. would have disturbed the other office workers D. wore stockings and were not as serious as men 7. A female secretary has been expected, besides other duties, to____. A. be her boss's memory B. do everything her boss asks her to do C. clean her boss's clothes D. telephone the boss's wife 8. Secretaries, until recently, had to do a lot of work now done by____. A. machines B. other staff C. servants D. wives 9. A secretary in the future will ____. 55 A. have more work to do B. be better paid C. have higher status D. have less work to do 10. The writer believes that before long ____. A. both men and women can act as secretaries B. men will be better than machines C. men will take over women's jobs as secretaries D. women will operate most office machines Passage Three Questions 11 to 15 are based on the following passage: Humanity is a new experiment on planet Earth. For most of its history, life on Earth was restrictedto the sea. Living things began to populate the land slightly over 400 million years ago, and humans have existed for no more than 3 million years. One way to represent the evolution of life is to compress the 4.6-billion-year history of Earth into a 1-year-long film. In such a film, Earth forms as the film begins on January 1, and through all of January and February it cools and is eratered (变成坑状) and the first oceans form. But those oceans remain lifeless until sometime in March or early April, when the first living things develop. The 4-billion-year history of Precambrian (前寒武纪) evolution lasts until the film reaches mid-November, when primitive ocean life begins to evolve into complex organisms such as trilobites (三叶虫 ). If we examine the land instead of the oceans, we find a lifeless waste. But once our film shows plant and animal life on the land, about November 28, evolution proceeds rapidly. Dinosaurs, for example, appear about December 12 and vanish by Christmas Eve, as mammals(哺乳动物) and birds flourish. Throughout the 1-year-run of our film there are no humans, and even during the last days of the year as the mammals rise and dominate the landscape, there are no people. In the early evening of December 31, vaguely human forms move through the grasslands, and by late evening they begin making stone tools. The Stone Age lasts until about 11 ~ 45 pm, and the first signs of civilization, towns and cities, do not appear until 11 ~ 54 pm. The Christian era begins only 14 seconds before the New Year, and the Declaration of Independence is signed with 1 second to spare. 11. In comparing all of Earth's geological evolution to one calendar year, the author shows____. A. just how recently humanity has arrived on the scene B. just how simple it is to understand the history of Earth C. just how early humanity appears on planet Earth D. just how difficult it is to understand the history of Earth 12. According to the passage, primitive ocean life begins to evolve into complex organisms____. A. in the middle of December B. when Earth is cratered C. at the beginning of November D. when the land is still a lifeless waste 13. It is indicated in the passage that____. A. Earth is cool at first B. Earth is hot at the beginning C. there are oceans at the beginning D. there is life in the ocean in late February 14. In the one-year-long film dinosaurs____. A. die out on December 25 B. appear after December 25 C. die out on December 24 D. appear on December 24 15. In the compressed history of Earth the Declaration of Independence is signed____. A. a long time before the New Year B. immediately after the New Year's Day C. just one second before the New Year D. right on the New Year's Day Passage Four Questions 16 to 20 are based on the following passage: 56 Internet-addicted job seekers may be partly the cause of the fact that it is taking the unemployed 23% longer to find a new position than it took during the last recession (经济萧条 ),when the "benefits" of online job searching were unavailable. "A growing number of unemployed Americans waste time browsing ( 浏览 ) the estimated 4,000 to 5,000 online job sites, filling them with resumes, and then waiting for replies. It is common for long-time joblessness,‖ argues Professor John A. Challenger. Although the Internet has the potential to be very useful for job seekers and it has become the primary tool for many, Challenger believes that it should be considered secondary to the traditional technique of meeting would-be employers in person. In addition to slowing job search efforts, the Internet is making the hiring process longer for the employer. In a survey of 5,000 hiring managers by an online r~sum~ site, 72 % said that a majority of the resumes they received in response to an online job posting did not match the position's description. "The more unrelated resumes managers have to go through in order to select the few to bring in for interviews, the longer it takes to fill the position," points out Challenger. "One result of this has been the increased use of screening software by employers. This will make it even more difficult for job seekers to get their resumes in front of the hiring executive for an interview." "All of this is not to say that the Internet has not revolutionized job hunting. It has certainly made it easier for someone in San Francisco, for example, to search for job openings in Miami. In addition, the ability to conduct keyword searches has reduced the amount of time it takes to find the type of position a person is seeking." "Job seekers must learn how to use the Internet as a tool, rather than just relying on it as a means for submitting electronic resumes," concludes Challenger. 16. According to the passage, the coming of online job searching brings A. longer waiting time for the unemployed B. longer computer technology training for the job seekers C. more unemployment throughout the U. S. A. D. more job opportunities in the Internet world 17. Professor Challenger thinks should be considered more important by the unemployed. A. browsing the Internet job sites B. sending resumes by email C. going to a traditional training center D. meeting the potential employers in person 18. The Internet is making the hiring process longer for the employer because A. the speed of the Internet is not fast enough B. there is a lot of false information on the Net C. the hiring position cannot be fully described on the Net D. it has drawn many unfitting resumes 19. It can be inferred that "screening software" in Paragraph 3 is used to____. A. draw more job seekers to the websites B. get rid of some unrelated resumes C. serve as a bridge between the job seekers and the employers D. provide the job seekers with more chances to visit the employers in person 20. According to the passage-the Internet has revolutionized job hunting by____. A. changing the recession cycle B. opening more chances for job seekers C. improving the reemployment situation D. reducing the interview time Part ? Vocabulary and Structure ( 共40小题,每题1分,共40分) Directions: In this part there are forty incomplete sentences. Each sentence is followed by four choices. Choose the one that best completes the sentence and .then mark your answer on the answer sheet. 21. The earth revolves a little more rapidly____ it is closer to the sun. 57 A. whether B. whereas C. although D. when 22. The number of members in the club ____to two hundred. A. were limited B. limits C. was limited D. limited 23. On close examination, we found the signature not____. A. realistic B. accurate C. exact D. genuine 24. As a highly ____young designer, she has a promising future in her field. A. original B. occasional C. optional D. obedient 25. Let me give you a ____of how the computer works. A. demonstration B. difference C. deduction D. distinction 26. An author must not be too ____to criticism. A. sensible B. sensitive C. senseless D. insensible 27. Fu Lei is known for his ____of music and knowledge on philosophy. A. explanation B. interpretation C. composition D. interaction 28. The judge dismissed the case because there was not ____evidence. A. adequate B. excessive C. many D. plenty 29. Much to my surprise, they went away without telling us their ____address. A. forever B. everlasting C. permanent D. eternal 30. Instead of going into details about his project, he spoke____. A. in short B. in general C. in particular D. in common 31. My book is ____finished; I have only a few changes to make in the writing. A. virtually B. violently C. vertically D. visually 32. The amount of water used in the manufacturing process is enormous, but the amount wasted is____. A. the greatest B. more greater C. greatest D. even greater 33. The ____was conducted to find out how many people prefer butter. A. examination B. inspection C. survey D. analysis 34. Many university courses are not really ____to the needs of students or their future employers. A. associated B. adopted C. geared D. qualified 35. ____David's expression, we'd say he is not in a good mood today. A. To judge by B. Judged by C. To be judged by D. Judging by 36. It is not considered ____to litter in public. A. respectful B. respective C. respected D. respectable 37. He was ____of his political rights because of the bribery. A. deprived B. acquired C. acquainted D. accused 38. His description of the ghost was so ____that his little daughter screamed when he came to the part of killing. A. moderate B. vivid C. active D. Passive 39. I thought his speech would be interesting, but it turned out that the more he talked, ____. A. the more bored became I B. the more I became bored C. the more bored I became D. I became the more bored 40. You have to take the ____examination before an interview can be considered. A. precious B. preliminary C. prior D. potential 41. Many writers ____their childhood memories for the materials of most of their stories. A. draw on B. draw up C. draw out D. draw off 42. Unless you have a good map, this place is very difficult to ____. 58 A. preserve B. dwell C. dislocate D. locate 43. In Scotland, as in the rest of the United Kingdom, ____schooling begins at age 5 and ends at age 16. A. compelling B. compulsory C. obliged D. compulsive 44. ____the rain, we should have had a pleasant trip to the countryside. A. Because of B. Due to C. Thanks to D. But for 45. When confronted with such an urgency, my mind goes ____,and I can hardly remember my own date of birth. A. dim B. blank C. faint D. vain 46. Horseback riding ____both the skill of handling a horse and mastery of diverse riding styles. A. fosters B. solves C. involves D. exaggerates 47. I was ____the chance of going to university when my parents' business collapsed and they lost everything they owned. A. ignored B. refused C. neglected D. denied 48. The fried fish we ate at the restaurant yesterday is marvelous. I'd like to have it again even if it costs _. A. as twice much B. twice as much C. much as twice D. as much twice 49. There has been a ____for industry to be constructed in a single region, instead of being scattered evenly over the whole country. A. trend B, signal C. vision D. currency 50. The final document was supposed to mend the damage ____upon the world by the war. A. impressed B. compromised C. imposed D. condensed 51. If you ____your friends, you may lose them. A. conceal B. deceive C. receive D. conceive 52. I don't know whether he is ____to special treatment just because of his rank. A. right B. bound C. due D. entitled 53. ____too much to do, they have to keep themselves busy all day long. A. Having B. Have C. Had D. Being 54. The bank refused to ____him any money, so he had to postpone buying a house. A. credit B. borrow C. loan D. lease 55. Air pollution, together with overpopulation, ____many problems in big cities today. A. are causing B. is causing C. are caused D. is caused 56. You can hire a bicycle in many places. Usually you'll have to pay a ____. A. fare B. fund C. deal D. deposit 57. She has no idea of what the book is about. She ____have read it very carefully. A. mustn't B. can't C. shouldn't D. needn't 58. It may be necessary to stop ____in the learning process and go back to the difficult points in the lessons A. at a distance B. at ease C. at intervals D. at length 59. I think I ____the movie we went to last night even more if I had read the book. A. would enjoy B. would have enjoyed C. will enjoy D. enjoyed 60. No sooner ____begun to speak than some noise arose from the audience. A. he had B. had he C. he has D. did he Part ? Cloze (共20小题,每题1分,共20分) Directions: There are twenty blanks in the following passage. For each blank there are four choices. Choose the one that best fits into the passage and then mark your answer on the answer sheet. 59 We got up early this morning and 61 a long walk after breakfast. We walked through the business62 of the city. The city 63 larger than I thought it would be. Well, the business section is smaller than I thought it would be. I suppose that's 64 Washington is a special kind of city. 65 of the people in Washington work for the government. About 9:30 we went to the White House. It's 66 the public from 10 67 12, and there was a long line of people 68 to get in. We didn't have to wait very long, because the line moved 69 quickly. The White House is really white. It is painted every year. And it seems very white, because it's got beautiful lawns 70 around it, with many trees and shrubs. The grounds 71 about four square blocks. I mean, they're about two blocks long 72 each side. Of course, we didn't see the whole building. The part 73 the President lives and works is not open to the public. But the part we saw was beautiful. We went 74 five of the main rooms. One of them was the library on the ground floor. On the next floor, there are three rooms named 75 the colors that are used in them: the Red Room, the Blue Room, and the Green Room. The walls are covered with silk76 . There are 77 of old furniture from the time 78 the White House was 79 built. And everywhere there are paintings and'statues of former presidents and 80 famous people from history. 61. A. made B. took C. went D. set 62. A. position B. place C. section D. circle 63. A. was B. is C. has been D. should be 64. A. reason B. for C. since D. because 65. A. Much B. More C. Few D. Most 66. A. opened to B. open to C. opening for D. opened for 67. A. near B. since C. towards D. Till 68. A. waited B. wait C. waiting D. to wait 69. A. somewhat B. pretty C. slightly D. completely 70. A. all B. whole C. every D. each 71. A. take B. make C. cover D. possess 72. A. about B. on C. in D. for 73. A. that B. where C. what D. which 74. A. across B. for C. through D. along 75. A. after B. by C. with D. for 76. A. clothes B. cloth C. clothing D. dressing 77. A. pieces B. bits C. blocks D. sheets 78. A. which B. since C. when D. where 79. A. firstly B. early C. at first D. first 80. A. the other B. other C. others D. Another 第二卷(共50分) Part ? Translation (共35分) Section A (共5小题,每题4分,共20分) Directions: Translate the following sentences into Chinese. You may refer to the corresponding passages in Part I. 81. Vegetable gardeners agree that many home-grown vegetables are superior to those purchased from markets. (Line 1, Paragraph 2, Passage 1) 82. One should not plant a garden that is too large for him to care for. ( Line 1, Paragraph 5, Passage 1 ) 83. There are a lot of women secretaries who will do the job as well as men. (Line 2, Last Paragraph, Passage2) 84. But those oceans remain lifeless until sometime in March or early April, when the first living things develop. 60 (Line 3, Paragraph 2, Passage 3) 85. Job seekers must learn how to use the Internet as a tool, rather than just relying on it as a means for submitting electronic resumes. ( Line I, Last Paragraph, Passage 4) Section B (共5小题,每题3分,共15分) Directions: Translate the following sentences into English 86(据说这幅画是毕加索(Picasso)的名作。 87(我记得以前在电视上见过他,但我不能肯定。 88(我们的冰箱坏了,必须找人修一下 89(面对这样复杂的情形,他努力抑制着自己的情感。 90(我期待着再次见到他,因为我们已整整十年没见面了。 Part V Writing (共15分) Directions: In this part you are required to write an essay about how your daily schedule has changed since you became a college student. You should write at least 120 words and base your composition on the outline (given in Chinese) below. 1(课堂学习日程 2(业余生活的安排 3(和中学有什么不同 2006年普通高校专转本统一考试英语试卷参考答案 Part I 1(B。由第一节可知,很多人享受于家庭园艺。 2(C。第二节中―从春季到深秋‖即大半年的时间。 3(A。由第四节第一句可知,人们花在花园上的钱可多可少。 4(D。由第五节最后一句可知。 5(B。由最后一节最后一句可知。 6(C。由本文第一句可知,以前女性不出门工作,人们思想较为保守,女秘书会干扰到职员们的工作。 7(B。由第二节可知,女秘书们需要为上司做所有的事情。 8(A。由第三节可知,现代社会的芯片和高科技可以取代秘书做很多事情。 9(C。由第三节和第四节可知,秘书这一职位会因为科技含量的提高而提高社会地位,这在美国已经成为了事实。 10(A。在最后一节中,作者认为男人有实力竞争这一职业。 11(A。全篇意在说明与全球的生物进化相比,人类历史只有很短的时间。 12(D。由第二节最后一句可知。 13(B。根据第二节第二句可知,地球在二月份温度降低,变成坑状,形成最初的海洋。由此可以推断,最初的地球温度很高。 14(C。由第三节最后一句可知。 15(C。由本文最后一句可知,《独立宣言》的签订比新年早一分钟。 16(A。第一节主要是讨论网络求职给求职者带来了更长的等待回复的时间。 17(D。本题可采用排除法。Challenger教授陈述了网络求职成功率的低下,认为网络不应该仅仅用来投电子简历,建议求职者与招聘方面对面。 18(D。由第二节可知,很多不符合岗位要求的简历增加了求职者等待回复的时间。 19(B。因为不相关简历的增加,所以降低求职者简历到达招聘人面前几率的―过滤软件‖也出现了。 20(B。由倒数第二节可知,网络确实是职场的一场革命,给求职者带来了更多的机会。 Part? 21(D。―当距离太阳更近时‖,地球转得更快。 61 22(C。主语中心词是number,因此谓语用单数形式。 23(D。realistic义为―现实的‖,accurate义为―准确的‖,exact义为―精确的‖,genuine义为―真实的‖。 24(A。original义为―创新的‖,occasional义为―偶尔的‖,optional义为―可选的‖,obedient义为―顺从的‖。 25(A。demonstration义为―展示‖,difference义为―差别‖,―deduction‖义为―扣除,推理, 结论‖,distinction义为―差别,特征,卓越‖。 26(B。sensible义为―理性的‖,senSitive义为―敏感的‖,senseless义为―无知觉的,愚蠢的,不醒人世的‖,insensible义为―无知觉的,无疼痛感觉的‖。 27(B。interpretation of music义为―对音乐的解读‖。 28(A。excessive义为―过量的,丰富的‖,adequate和plenty都是―足够的‖之义,但后者用于肯定句。 29(C。forever义为―永远的‖,一般不作定语,everlasting义为―持久的‖,permanent义为―固定的‖,eternal义为―永恒的,永久的‖。 30(B。与―going into details‖相对的是―In general(概要地)‖。 31(A。virtually义为‖事实上‖,violently义为―猛烈地,暴力地‖,vertically义为―垂直地‖,visually义为―视觉地‖。 32(D。本题考察比较级的格式。 33(C。examinatlon义为―考试,检查‖,inspection义为―督察‖,survey义为―调查‖(analysis义为―分析‖。 34(C。associate义为―关联‖,adopt义为―采用‖.gear义为―配备‖,qualified义为―合格的‖。 35(D。固定短语(judging by义为―从……判断‖。 36(D。respectful义为―恭敬的‖,respective义为―各自的‖,respected义为―被尊敬的‖,respectable义为―值得尊敬的,有风度的‖。 37(A。固定短语be deprived of义为―被剥夺‖。 38(B。moderate义为―温和的,适度的‖,vivid义为―逼真的‖,active义为―主动的‖, passlve义为―被动的‖。 39(C。―the more…,the more…‖的结构,句子中主谓不倒装。 40(B。面试之前的考试是―预备的‖,而不足―珍贵的‖,―优先的‖或是―潜在的‖。 41(A。draw on义为―借用‖,draw up义为―草拟,拆除‖,draw out义为―鼓励(某人说话 等)‖,draw off义为―拉走,撤退,转移‖。 42(D。―找到‖一个地方的位置用―locate‖。 43(B。compulsory education义为―义务教育‖。 44(D。虚拟句。 45(B。dim义为―(灯光)暗淡的((轮廓)模糊不清的‖(blank义为―空白的‖(faint义为―昏眩的,无力的‖,valn义为―自负的,徒劳的‖。 46(C。foster义为―加强,巩固‖(solve义为―解决‖,involve义为―需要,使某事物成为必要条件,涉及‖,exaggerate义为―夸张‖。 47(D。ignore义为―无视‖,refuse义为―拒绝‖,neglect义为―忽略‖,deny义为―拒不给 与‖。 48(B。本题考察的是倍数的格式,倍数后接as much/many。 49(A。现代工业的―趋势‖是建立单个的工业区。 50(C。impress义为―留下印象‖,compromise义为―妥协‖,impose义为―强加‖,condense义为―浓缩‖。 51(B。conceal义为―隐藏‖,deceive义为―欺骗‖,receive义为―迎接‖,conceive义为―构想,设想‖。 52(D。固定短语be entitled to do sth义为―享有做某事的权利‖。 53(A。分词短语作伴随状语。 62 54(C。loan义为―借出‖,credit义为―相信,存入‖,lease义为―租借‖。 55(B。考察主谓一致的问题。―together with‖后接的名词不算作中心词一部分。 56(D。fare义为―票价‖,fund义为―专款,现款‖,deal义为―交易‖,deposit义为―押金‖。 57(B。表示否定的推测,用can't(不可能)。 58(C。at a dlstance义为―一定距离之外‖,at ease义为―轻松地‖,at intervals义为―间 歇地‖,at length义为―详尽地,最后,终于‖。 59(B。虚拟语气。 60(B。No sooner后接过去完成时的倒装式。 Part ? 61(B。to take a walk义为―散步‖。 62(C。business section义为―商业区‖。 63(B。与后面一句的时态并列相同。 64(D。这一想象的―原因‖是华盛顿是一个特别的城市。 65(D。商业区较小,因为―大多数‖人都在政府部门工作。 66(B。be open to sb(义为―对某人开放‖。 67(D。白宫从十点―直到‖十二点向游人开放。 68(C。分词短语作伴随状语,与主语的关系是主动,因此用现在分词。 69(B。somewhat义为―某种程度上‖,pretty义为―相肖地‖,slightly义为―轻轻 地‖,completely义为―完全地‖。 70(A。all around义为―周围‖。 71(C。那些土地―覆盖‖了四个长方形的面积。 72(B。 73(B。定语从句,所修饰的成分在从句中作状语。 74(C。go through义为―穿过‖。 75(A。named after义为―以……命名‖。 76(B。丝绸的布:silk cloth。 77(A。a piece of turniture义为―一件家具‖。 78(C。定语从句。 79(D。firstly义为―首先‖,early义为―早的‖,at first义为―开始‖(first义为―刚开始时‖。 80(B。其他的:other。 Part ? Section A 1(菜园种植者一致认为,许多自家种的蔬菜要比市场上买来的好。 2(菜园不应种得太大,使人无法照料。 3(有许多女秘书能够像男人一样做好这项工作。 4(在三月的某个时候或四月初第一批生物刚刚形成,但是在此之前海洋中一直没有生命。 5(求职者得学会如何把因特网作为工具使用,而不是仅仅靠它来提交电子简历。 Section B 1. In is said that the painting ls a masterpiece by Picasso( 2. I remember seeing him on TV before,but I anm not sure( 3. 0ur refrigerator is out of order and we must have it repaired( 4. Faced with such a complicated situation,he tried to hold back his emotions. 5. I'm looking forward to seeing him again,for it is ten years to the day since we last met. Part V 63 My Daily Schedule Has Changed Since I became a college student,l have changed my daily schedule for both study and entertainment In college, most of the classes are taken in the morning, with the whole afternoon free. In most cases,I spare the afternoon for after-class work and activities. Therefore,I focus on my study in the evening.I go to the library where I can refer to a lot of great works to review,preview and do some extra research. College students go to bed much later than high school students, so l could study for a long time in the evening. College students do not have the sole aim of passing a national exam as high school students do. After-class activities are equally important to improve one's working ability.I joined three clubs, where I usually work and help organize some activities in the afternoon. At weekends,I usually do part-time work to earn money for my tuition fee. College life ls very different from the life at high school. High school Iife is mainly about preparing for the College Entrance Examination,while college life teaches you how to deal with difficulties in life by yourself. 2007年江苏省普通高校“专转本”统一考试试卷 大学英语 第一卷(共100分) 注意事项: 1(答第?卷前,考生务必按规定要求填涂答题卡上的姓名、准考证号等项目。 2(用铅笔把答题卡上相应题号中正确答案的标号涂黑。答案不涂写在答题卡上,成绩无效。 Part ? Reading Comprehension(共20小题,每小题2分,共40分) Directions: In this part there are four passages. Each passage is followed by four comprehension questions. Read the passage and answer the questions. Then mark your answer on the answer sheet. Passage One Questions1,5 are besed on the following passage. Under proper conditions, sound waves will be reflected from a hillside or other such obstruction. Sound travels at the rate of about one-fifth of a mile per sccond. If the hill is eleven hundred feet away, it takes two seconds for the sound to travel to the hill and back. Thus, by timing the interval between a sound and its reflection (the echo), you can estimate the distance to an obstruction. During World War ? the British used a practical application of this principle to detect German planes on their way to bomb London long before the enemy was near the target. They used radio waves instead of sound waves, since radio wavcs can penetrate fog and clouds. The outnumbered Royal Air Force( RAF) always seemed to the puzzled Germans to by lying in wait at the right time and never to be surprised. It was radio echoes more than anything else that won the Battle of Britain. Since the radio waves were used to tell the direction in which to send the RAF planes and the distance to send them (their range of flight, in other words), the device was called―radio directing and ranging‖, and from the initials the word ―radar‖was coined. 1. Sound waves reflected from a hill can be used to estimate the ____. A. height of the hill B. speed of sound C. distance to the hill D. intensity of sound 2. Practical application of this principle resulted in ____. A. new electronic instruments for planes B. a radio directing and ranging device C. new radio sets for RAF bombers 64 D. an electronic detecting device 3. Radar enabled the English to ____. A. detect German planes on their way to London B. direct the outnumbered RAF planes effectively C. confuse German bomber pilots D. both A and B 4. It can be inferred from the passage that ____. A. Light waves could be used in a device similar to radar B. Radar was a practical application of a well-known principle C. Radar greatly increased the effectiveness of the Royal Air Force D. Sound waves are reflected from a hillside under all conditions 5. The author of this passage probably intended to explain ____. A. exactly how radar works B. why the British used radio waves in their device C. how radar (word and device) came into being D. how radar helped the British win the Battle of Britain Passage Two Questions 6,10 are based on the following passage. In general, the ancient Romans were a practicai people. They cared less about philosophy and pure mathematics than the Greeks did. The Romans were the best of the ancient engineers and architects. They were brilliant students of Greek geometry and trigonometry(三角学), and they applied their knowledge to the construction of fine bridges, roads, aqueducts(渠道) , and public buildings. They knew about the smelting of iron ore, but they used the iron they produced mainly for spears, swords, and shields, rather than for building construction. While the Romans accomplished practical wonders,they did very little theoretical scientific thinking. Because of their frequent wars, many of their inventions were no more than improvements in the design of Greek weapons with which they were familiar. There are two reasons for the Roman neglect of philosophy and pure mathematics.First,they were apparently too busy conquering nearby nations and forming them into a rapidly growing empire to waste much time on abstract thinking. Secondly, they were handicapped(妨碍) by the rigidity of their numerical system. (Try to multiply XI by LVII or to divide CXLIII by IX;it simply cannot be done with pencil and paper.)The Romans did all of their arithmetic on an abacus, the ancient counterpart of the modern computing machine.Their number system discouraged the study of pure mathematics. 6. The author calls the Romans "practicat" because they ____. A. cared little about philosophy B. applied their knowledge to construction C. saw the necessity for developing theoretical science D. studied the past and learned from it 7. The ancient Romans did not concern themselves with ____. A. theoretical or abstract thought B. applied mathematics C. studying D. either A or C 8. In construction,the Romans made extensive use of ____. A. iron ore B. geometry and trigonometry C. Greek mathematical thought D. both B and C 65 9. Many Roman inventions were based on Greek weapons because the ____. A.Greeks had superior weapons B. Romans were involved in frequent wars C. Romans were not familiar with many types of weapons D. Greeks proved themselves inspired inventors 10. The author of this passage probably wanted to explain why ____. A. he considers the Romans a practical people B. Roman architects and engineers were the best in ancient times C. theoretical thinking is necessary for the growth of a nation D. the Romans contributed little to philosophical thought Passage Three Questions 11,15 are based on the following passage. Professor Barry Wellman of the University of Toronto in Canada has invented a term to describe the way many North Americans interact these days. The term is ―networked individualism.‖This concept is not easy to understand because the words seem to have opposite meanings. How can we be individuals and be networked at the same time? You need other people for networks. Here is what Professor Wellman means. Before the invention of the Internet and e-mail, our social networks involved live interactions with relatives, neighbors, and colleagues at work. Some of the interaction was by phone, but it was still voice to voice, person to person, in real time. A recent research study by the Pew Internet and American Life Project showed that for a lot of people, electronic interaction through the computer has replaced this person-to-person interaction. However, a lot of people interviewed for the Pew study say that's a good thing. Why? In the past,many people were worried that the Internet isolated us and caused us to spend too much time in the imaginary world of the computer. But the Pew study discovered that the opposite is true. The Internet connects us with MORE real people than expected-helpful people who can give advice on careers,medical problems,raising children,and choosing a school or college.About 60 million Americans told Pew that the Internet plays an important role in helping them make major life decisions. Thanks to the computer,―networked individuals‖are able to be alone and together with other people-at the same time! 11. The Pew study was conducted in ____. A. The United States B. Canada C. The U. S. and Canada D. Europe 12. In this article,a network is a group of connected ____. A. radio or TV stations B. people C. computers D. roads 13. Before the invention of the Internet,our connections with people took place mainly ____. A. in person B. by phone C. by letter D. by email 14. Which of the following has happened since the invcntion of the Internet and email? A. People are talking on the phone more than ever. B. Interaction through the computer has replaced a lot of person to person interaction. C. Americans are turning into hermits. D. Sixty million Americans have bought computers. 15. Which of the following was NOT one of the discoveries of the Pew study? A. The Internet has put us in touch with more people than expected. B. People use the Internet to get advice on careers, medical problems, and other questions. C. For many Americans,the Internet plays an important role in helping them make important decisions. D.―Internet addiction‖is a growing problem among people who use computers. 66 Passage Four (非英语类学生必做,16,20小题。) Questions 16,20 are based on the following passage. Summers with father were always enjoyable. Swimming, hiking, boating, fishing-the days were not long enough to contain all of our activities. There never seemed to be enough time to go to church, which disturbed some friends and relations. Accused of neglecting this part of our education, my father instituted a summer school for my brother and me. However, his summer course included ancient history, which Papa felt our schools neglected, and navigation, in which we first had a formal examination in the dining room, part of which consisted of tying several knots in a given time limit. Then we were each separately sent on what was grandly referred to as a cruise in my father's 18-foot knockabout, spending the night on board, and loaded down, according to my mother, with enough food for a week. I remember that on my cruise I was required to formally plot our course, using the tide table, even though our goal was an island I could see quite clearly across the water in the distance. 16. What was the original reason for holding the summer school? A. The father wanted the children to learn more about religion. B. The children got poor grades in their regular school. C. The regular school teachers neglected the children. D. Friends and relatives thought the children should learn religion. 17. The purpose of the cruise mentioned in the passage was to ____. A. have fun B. test the author's sailing ability C. reward the author for completing summer school D. get to the island 18. Why did the author have to plot the course of her cruise? A. She had to demonstrate her ability to do so. B. She was afraid of getting lost. C. The coast was dangerous. D. The tides were strong. 19. How long did the author's cruise last? A. All summer. B. Overnight. C. A week. D. One day, morning till night. 20. Apparently a knockabout is ____. A. a seaman's knot B. a cruise C. an island D. a boat (英语类学生必做,16,20小题。) Questions 16,20 are based on the following passage. The three main types of secondary education in the United States have been provided by the Latin grammar school, the academy, and the public high school. The first of these was a colonial institution. It began in New England with the establishment in 1635 0f the Boston Free Latin School.The curriculum consisted mainly of the classical languages, and the purpose of this kind of school was the preparation of boys for college, where most of them would be fitted for the ministry. The academy began in the early 1750's with Benjamin Franklin's school in Philadelphia, Which laterbecame the University of Pennsylvania. It extended generally to about the middle of the nineteenth century, except in the southern states where the public high school was late in developing and where the academy continucd a principal means of secondary education even after 1900. The academy was open to girls as well as to boys, and it provided a wider curriculum than the Latin grammar school had furnished. It was designed not only as a preparation for college but also for practical life in commercial and business activities. Although its wide educational values were evident and are recognized as important contributions to secondary education in this country, the academy was never considered a public institution as the public high school has come to be. The public high school had its origin in Massachusetts in 1821 when the English Classical School was established in Boston. In 1827, that state enacted the first state-wide public high-school law in the Untied States. 67 By 1840, there were perhaps a dozen public high schools in Massachusetts and many in other eastern states;by 1850, they were also to be found in many other states. Just as the curriculum of the academy grew out of that of the Latin grammar school, so the curriculum of the public high school developed out of that of the academy. The public high school in the United States is a rejection of the aristocratic(贵族) and selective principle of the European educational tradition. Since 1890, enrollments in secondary schools, mainly public high schools, have practically doubled in this country every ten years. 16. According to the author, the main types of secondary education in the United States have been provided by ____. A. New England establishments B. the Latin grammar school and the academy C. the public high school D. both B and C 17. According to the passage, which of the following sequences indicates the order in which the school developed? A. Latin grammar school, public high school, academy. B. Latin grammar school, academy, public high school. C. public high school,Latin grammar school, academy. D. public high school, academy, Latin grammar school. 18. Since 1890, secondary-school enrollments have ____. A. almost doubled every ten years B. practically doubled C. declined in enrollment D. increased tenfold 19. In the southern states, the principal means of secondary education even after 1900 continued to be the ____. A. Latin grammar school B. academy C. classical language school D. church school 20. One can properly infer from this article that―Latin grammar school‖refers to ____. A. the Boston Free Latin School B. all elementray schools in the United States C. schools which taught Latin exclusive of all other subjects D. a number of grammar schools which developed in New England Part ? Vocabulary and Structure(共40小题,每小题1分,共 40分) Directions: In this part there are 40 incomplete sentences. Each sentence is followed by four choices. Choose the one that best completes the sentence and then mark your answer on the answer sheet. 21. Mary is one of the brightest students who ____from New York University. A. graduated B. have graduated C. had graduated D. has graduated 22. My friend's son, who is a soldier, was delighted when he was ____only a few miles from home. A. camped B. situated C. placed D. stationed 23. She has bought some lovely ____to make herself a dress. A. stuff B. clothing C. material D. pattern 24. She is pleased with what you have given her husband and ____you have told him. A. that B. which C. all what D. all that 25. I passed my exams but it was a long time ____my friends about it. A. that I didn't tell B. before I told C. after I told D. since I told 26. The stone statue in the city square was put up ____the fallen heroes. A. in memory of B. in search of C. in terms of D. in view of 27. When I applied for my passport to be renewed, I had to send a ____photo. A. fresh B. familiar C. recent D. late 68 28. The Americans and the British not only speak the same language but also ____many social customs. A. join B. share C. create D. maintain 29. ____from the hill-top, the lake scenery is beyond description. A. Seen B. Seeing C. To see D. Having seen 30. In many schools, students don't have easy access ____computers. A. of B. into C. for D. to 31. His success was due to ____he had been working hard all the time. A. that B. the fact which C. the fact that D. the fact what 32. He didn't want to go to the cinema but they begged ____so hard that he finally and went with them. A. gave off B. gave away C. gave out D. gave in 33. The toy boat turned over and sank to the ____of the pool. A. hot tom B. ground C. floor D. base 34. You mustn't be ____, or else you will make a lot of mistakes. A. quick B. hasty C. rapid D. fast 35. Do not disturb me. I ____letters all morning and have written six so far. A. write B. am writing C. was writing D. have been writing 36. Teachers always tell their students that it is no good ____today's work for tomorrow. A. to leave B. leaving C. left D. leave 37. I have really got angry with John because ____I suggest, he always disagrees. A. whatever B. anything C. what D. everything 38. George had great difficulty in swimming across the lake, but he finally succeeded on his fourth____. A. process B. attempt C. display D. intention 39. After much thought, the engineer found a very ____solution to the problem. A. logical B. necessary C. clever D. ordinary 40. Edison tested more than one thousand materials to see if they could ____electric current and glow. A. bring B. make C. carry D. produce 41. If we had known that she had planned to arrive today, we ____her at the bus station. A. will have met B. might meet C. had met D. might have met 42. When a machine is ____, suitable materials must be chosen for its parts. A. building B. build C. to build D. to be built 43. His wide ____of the newspaper world enabled him to become a successful editor. A. career B. experience C. experiment D. information 44. Scientists have recently ____the theory that eating too much fat is bad for the heart. A. put forward B. put along C. put out D. put up 45. When we reached the station, the train had not arrived yet; so we ____. A. needed not to hurry B. needn't have hurried C. didn't need to hurry D. had not needed to hurry 46. Many things ____impossible in the past are quite common today. A. considering B. being considered C. to be considered D. considered 47. We believe that the young generation will prove ____of our trust. A. worth B. worthy C. worthwhile D. worthless 48. The two elements ____water is made up are the gases-oxygen and hydrogen. A. that B. which C. of which D. with which 49. So fast ____that it is difficult for us to imagine its speed. 69 A. light travels B. travels light C. does light travels D. does light travel 50. Output is now six times ____it was before liberation. A. that B. which C. what D. of which 51. Most foreign businessmen are ____the government's new policy on foreign investment. A. in relation to B. in possession of C. in contrast of D. in favor of 52. He made such a ____contribution to the university that they are naming one of the new buildings after him. A. modest B. generous C. real D. adequate 53. It has always been the ____of our firm to encourage workers to take part in social activities. A. campaign B. procedure C. policy D. rule 54. We arranged to meet at the theatre at seven but she didn't ____. A. turn up B. turn down C. turn off D. turn round 55. The passengers ____out of the exit when we arrived at the airport. A. were just coming B. just came C. are just coming D. just come 56. As soon as World War ? ended, Einstein urged that atomic energy ____to peaceful use. A. is put B. be put C. would be put D. will be put 57. Surveys have revealed that quite a lot of people watch TV only to ____time. A. waste B. spend C. kill D. take 58. Neither the teacher not her students ____to attend the meeting by the headmaster. A. has been asked B. has asked C. have asked D. have been asked 59. The photographs of Mars taken from satellites are ____than those taken from the earth. A. much clearer B. the clearest C. clearest D. more clearer 60. Although I hadn't seen him for years, I ____his voice on the telephone at once. A. realized B. recognized C. heard D. discovered Part ? Cloze(共20小题,每小题1分,共20分) Directions: There are 20 blanks in the following passage. For each blank there are four choices. Choose the one that best fits into the passage and then mark your answer on the answer sheet. (非英语类学生必做,61,80小题。) I arrived in the United States 61 February 6, 1986, but I remember my first day here very 62 . My friend was waiting for me when my 63 landed at Kennedy Airport at three o'clock in the afternoon. The weather was very 64 and it was snowing, but I was 65 excited to mind. From the airport, my friend and I 66 a taxi to my hotel. On the way, I saw the skyline of Manhattan for the first time and I stared in 67 at the famous World Trade Center, the tallest building in the world.My friend helped me 68 at the hotel and then left because he had to go 69 to work. He promised to return the next day. Shortly after he left, I went to a 70 near the hotel to get something to eat. As I couldn't speak 71 of English, I couldn't tell the 72 what I wanted. I was very upset and started to make some gestures, but the waiter didn't 73 me. Finally, I ordered the same thing the man at the next table was eating. After dinner, I started to walk along Broad-way 74 I came to Times Square with its movie theatres, neon lights, and huge crowds of people. I did not feel tired, so I 75 walking around the city. I wanted to see everything on my first day. I knew it was 76 , but I wanted to try. When I returned to the hotel, I was exhausted, but I 77 sleep because I kept hearing the fire and police sirens during the night. I lay 78 and thought about New York. It was a very big and 79 city with many tall buildings and big cars, and full of noise and busy people. I also decided right then that I had to learn to speak 80 . 61. A. in B. at C. on D. for 70 62. A. clearly B. quickly C. eagerly D. badly 63. A. car B. plane C. bus D. helicopter 64. A. hot B. cold C. mild D. changeable 65. A. so B. very C. much D. too 66. A. took B. made C. paid D. reached 67. A. sadness B. horror C. disappointment D. astonishment 68, A. prepare B. stay C. unpack D. pack 69. A. back B. forward C. over D. across 70. A. store B. supermarket C. restaurant D. baker's 71. A. a little B. a word C. much D. any 72. A. baker B. manager C. assistant D. waiter 73. A. understand B. obey C. pay attention to D. take care of 74. A. when B. after C. until D. as 75. A. began B. stopped C. intended D. continued 76. A. important B. impossible C. exciting D. interesting 77. A. could B. couldn't C. would D. wouldn't 78. A. awake B. asleep C. waiting D. confused 79. A. polluted B. polluting C. interested D. interesting 80. A. French B. Spanish C. English D. Chinese (英语类学生必做,61,80小题。) Sleep is important to us because it helps restore organs and tissues in our body. But how much sleep do we actually need? For most of us, eight hours seems to be about the right amount. Yet we know that there are many people who get 61 perfectly with less sleep, and some who may need 62 . A great deal depends on the 63 we live. But a good general rule 64 is to sleep as long as we have to in order to feel happy and be able to work 65 our best when we awaken. There are actually different 66 of sleep. There is a deep sleep and a shallow sleep. In a shallow sleep our body does not get the same kind of rest it gets in a 67 sleep, 68 after eight hours of a shallow sleep we may still feel tired. But a short,deep sleep can be very 69 . Alexander the Great was able to get a deep sleep_70 he needed it. Once, during the night before an important battle, he remained 71 longer than anyone else. 72 he wrapped himself in a cloak and73 down on the earth. He slept so 74 that his generals had to wake him three times for him to give the75 to attack! Normally when we go to sleep, our―sleep center‖blocks off nerves so that 76 our brain and body go to sleep. 77 prevents us from wanting to do anything, and the other 78 our internal organs and limbs go to sleep. 79 sometimes only one goes to sleep and the other does not. A very tired soldier can sometimes 80 asleep(brain sleep) and keep on marching, because his body is not asleep. 61. A. across B. away C. along D. over 62. A. more B. much C. least D. little 63. A. style B. way C. method D. manner 64. A, follow B. to follow C. followed D. following 65. A. to B. in C. for D. at 66. A. levels B. standards C. heights D. varieties 67. A. daytime sleep B. nighttime sleep C. shallow D. deep 68. A. now that B. in order that C. so that D. for fear that 71 69. A. restful B. restless C. useful D. useless 70. A. wherever B. however C. whenever D. whichever 71. A. active B. awake C. asleep D. aware 72. A. Instead B. Still C. Moreover D. Then 73. A. lay B. laid C. lied D. lain 74. A. deep B. sound C. deeply D. lightly 75. A. conference B. consent C. confidence D. command 76. A. either B. both C. neither D. not only 77. A. It B. One C. This D. That 78. A. makes B. forces C. causes D. leads 79. A. So B. Thus C. Or D. But 80. A. turn B. go C. fall D. become 第?卷(共50分) Part ? Translation (共35分) Section A (共5小题,每小题4分,共20分) Directions: Translate the following sentences into Chinese. You may refer to the corresponding passages in Part ? . 1. They used radio waves instead of sound waves, since radio waves can penetrate fog and clouds.(Passage One) 2. Because of their frequent wars, many of their inventions were no more than improvements in the design of Greek weapons with which they were familiar.(Passage Two) 3. In the past, many people were worried that the Iniernet isolated us and caused us to spend too much time in the imaginary world of the computer. (Passage Three) 4.(非英语类学生必做) Summers with father were always enjoyable. Swimming, hiking, boating, fishing the days were not long enough to contain all of our activities.(Passage Four) 5.(非英语类学生必做) There never seemed to be enough time to go to church, which disturbed some friends and relations. (Passage Four) 4. (英语类学生必做) The curriculum consisted mainly of the classical languages, and the purpose of this kind of school was the preparation of boys for college, where most of them would be fitted for the ministry. (Passage Four) 5.(英语类学生必做) Just as the currichlum of the academy grew out of that of the Latin grammar school, so the curriculum of the public high school developed out of that of the academy. ( Passage Four) SectionB(共5小题,每小题3分,共15分) Directions: Translate the following sentences into English. 1(使他吃惊的是,她不仅没有表扬他反而批评了他。 2(陈大夫整个晚上都在为伤员做手术。 3(我们提议对计划作修改。 4(他那么细心不会不注意到这一点的。 5(很难说这两个人中谁年龄大些,看上去他们年龄差不多。 Part ? Writing(共15分) Directions: In this part you are required to write an essay about making the most of our school days.You should write at least 120 words and base your composition on the outline (given in Chinese) below. 1(生命是短暂的,时间是宝贵的; 72 2(在校学习的时间更短暂; 3(应该充分利用在校时光。 2007年普通高校专转本统一考试英语试卷参考答案 Part I 1(C。第一节的主要内容。 2(B。第二节和第三节主要是介绍一句这一原理而产生的雷达仪器。 3(D。由第三节可知。 4(C。由第二节最后一句可知。 5(C。本文意在介绍雷达这一仪器及其名称的由来。 6(B。根据第一节可知,古罗马人是最好的工程师和建筑家。 7(A。根据第三节可知,古罗马人很少进行理论的或抽象的思考,而主要集中精力发展实用的武器。 8(D。由第一节可知。 9(B。由第三节第二句可知。 10(A。本文中心句为开头第一句。 11(A。细节题。答案可在第四节中找到。 12(C。因特网是指一些互相联系的计算机。 13(A。根据第二节可知,以前人们的交互主要人对人的交流。 14(B。由第三节第一句可知。 15(D。D项在文中没有提到。 16(D。由―Accused of neglecting this part of our education‖可知。 17(B。这是父亲所设航海课程的内容。 18(A。考查其能力的内容之一。 19(B。由―spending the night on board‖可知。 20(D。 Part? 21(B。就近原则。 22(D。station义为―驻扎‖。 23(C。stuff为―东西‖之义,clothing为―衣物‖之义,material为―衣料‖之义,pattern为―样式‖之义。 24(D。定语从句,all that为固定搭配连接词。 25(B。根据句义,我通过考试很长时间之后才告诉我的朋友。在英语中,此句需转换一下思维:在我 告诉朋友们之前已经过了很长时间。 26(A。1n memory of为―纪念,怀念‖之义,in search of为―寻找‖之义,in terms of为―就某事而言‖, 1n view of为―由于,鉴于‖之义。 27(C。fresh为―新鲜的‖之义,familiar为―熟悉的‖之义,recent为―最近的‖之义,late为―晚期的,迟 的,已故的‖之义。 28(B。J01n为―加入‖之义,share为―共同拥有‖之义,create为―创造‖之义,maintain为―维持‖之义。 29(A。主语是the lake scenery,因此应该用过去分词表被动。 30(D。固定短语have access to,义为―可以使用,可以接触到‖。 31(C。同位语从句用that连接。 32(D。give off为―散发出‖之义,give away为―赠送,泄露,告发,丧失,抛弃‖之义,glve out为―分 发,公布,耗尽,用完‖之义,give ln为―妥协‖之义。 33(A。池塘的底部用―bottom‖表示。 34(B。hasty比其余三个多了―赶紧,匆忙‖之义。 35(D。用现在完成时的进行时态表示该动作一直在持续。 73 36(B。固定句型:It is no good doing sth( 37(A。 38(B。他最后终于在第四次―尝试‖的时候成功了。 39(C。clever为―聪明的‖之义。 40(C。此处carry为―承载‖之义。 41(D。过去时态的虚拟。 42(D。本题可采用排除法。只有―is to be built‖格式正确。 43(B。他在报界广泛的―经验‖使他成为了一名成功的编辑。 44(A。put forward为―提出‖之义,put out为―伸出,生产,出版,熄灭‖之义。 45(B。过去时态的虚拟。 46(D。分词作状语,与主语是被动的关系。 47(B。worth为―价值‖之义,worthy为―值得的‖之义,worthwhile为―有价值的‖,worthless为―无价值 的,没有用处的‖之义。 48(C。定语从句,从句中包含―be made up of‖这一短语。 49(D。以so开头,主谓倒装。 50(C。宾语从句,主句宾语在从句中作宾语。 51(D。in relation to为―和某事物有关系‖,in possession of为―拥有‖之义,in contrast of为―与某事物 形成对比‖之义,1n favor of为―赞成‖之义。 52(B。他为学校做了如此―慷慨‖的贡献以致于学校将以他的名字名字一幢新楼。 53(C。campaign为―运动‖之义,procedure为―程序‖之义,policy为―政策‖之义,rule为―规则‖之义。 54(A。turn up为―出现‖之义,turn down为―调低(音量等)‖,turn off为―关掉‖之义, turn round为―转身,曲解,改变意见‖之义。 55(A。在when引导的从句中,主句动词为可持续动词用进行时态。 56(B。虚拟态,用(should)do sth( 57(C。to kill time为―消磨时间‖之义。 58(D。就近原则。 59(A。考察比较级的格式。 60(B。―辨认‖出人的声音用―recognize‖。 Part ? 61(C。表不在具体的某一天用―on‖。 62(A。虽然已经是1986年的事了,但我还是能―清楚地‖记得我第一天到这儿的情景。 63(B。―飞机‖降落在肯尼迪机场。 64(B。正在下雪,天气很―冷‖。 65(D。too…to…结构。 66(A。打的:to take a taxi。 67(D。第一次看到曼哈顿,我―惊讶地‖注视着著名的世贸中心。 68(C。我入住旅馆,朋友帮我―打开行囊‖。 69(A。 70(C。根据下文―waiter‖提不,作者应该是在―restaurant‖。 71(B。因为英文―一个字‖都不会说,所以我无法告诉―服务员‖我想要什么。 72(D。 73(A。我很沮丧,开始打一些手势,可是服务员还是不能―明白我的意思‖。 74(C。晚饭后,我沿着百老汇―一直走到‖时代广场。 75(D。我不觉得累,所以我―继续‖在城市里转悠。 74 76(B。我知道这是―不可能的‖,但是我还是要尝试。 77(B。回到旅馆的时候,我累坏了,但是我―无法入睡‖。 78(A。我躺着,―一直没睡着‖,想着纽约。 79(D。这是一个又大又―有意思‖的城市,有很多高楼和大汽车。 80(C。我决定开始学习―英语‖。 (英语类学生必做,16-20小题。) 16(D。由本文第一句可知。 17(B。第一节主要是关于拉丁语法学校,第二节主要是关于学院的介绍,第三节主要是公立高中的介绍。 18(A。由文章最后一句可知。practically与almost同义。 19(B。由第二节―except in southern states…where the academy continued a principal means of secondary education even after 1900‖可知。 20(D。由第一节可知,拉丁语法学校是最早的从新英格兰发展起来的语法学校。(英语类学生必做,61-80小题。) 61(C。get along为―前进;离去,走开;渐晚,上年纪;继续生活;活下去;进展,进行;相处融洽‖之义。 62(A。有些人需要很少的睡眠,而有些人则需要―更多‖。 63(B。很大程度上取决于我们生活的―方式‖。 64(C。分词短语作后置定语。动词与主语是被动关系。 65(D。at one's best为―处在最佳状态‖之义。 66(A。睡眠有不同―层次‖——shallow sleep和deep sleep。 67(D。在shallow sleep中人们得到的休息和―deep‖sleep不一样。 68(C。前面事实导致后面结果,为因果关系。 69(A。restful为―让人得到休息的‖之义。 70(C。―每当‖他需要的时候,亚历山大大帝总能睡熟。 71(B。大战前一晚,他一定几乎没睡觉,所以后来他才睡那么熟。 72(D。他很久都没睡,―然后‖就把自己裹起来睡觉。 73(A。躺下:lie down,过去式为lav down。 74(C。用副词修饰动词sleep。 75(D。叫了三次才把他叫醒下―命令‖进攻。 76(B。句中动词用的是复数形式,因此主语用both修饰。 77(B。与下半句的the other呼应,因此用one。 78(A。固定短语,make sb. /sth(do sth( 79(D。转折关系。 80(C。fall asleep为―入睡‖之义。 Part ? Section A 1(他们用的是无线电波而不是声波,因为电波能穿透云雾。 2(因为战争频繁,其中许多发明仅仅是对他们所熟悉的希腊武器的设计作些改进。 3(过去,许多人担心,因特网把我们同他人隔绝开来,使我们把太多的时间花在了电脑的虚拟世界里。 4(暑期,和父亲在一起总是非常快乐。游泳、远足、划船、垂钓——这些日子还不够从事所有这些活动。(非英语类学生必做) 5(似乎总没有足够时间去做礼拜,这使我们一些朋友和亲戚感到不安。(非英语类学生必做) 4(课程主要有古典语言,这种学校的目的就是为男孩上大学作准备,其中大多数人将被培养适应从75 事神职工作。(英语类学生必做) 5(正如学院的课程是基于拉丁语法学校课程一样,公立中学的课程是在学院课程的基础上发展起来 的。(英语类学生必做) SectionB 1. To his surprise,she criticized him instead of praising him. 2. Dr. Chen went on operating on wounded soldiers throughout the night. 3. We propose that some changes( should) be made in the plan. 4. He ls too careful to not have noticed it. 5. It's hard to tell which of the two is older.They look about the same age. Part V Life is so short that people have been always feeling sad for the flying time. To avoid the sadness, it is important to make the most of time and to make your life your more interesting. People who are still remembered after their life are those who spent their time pursuing their noble dream or their original career. Life on campus is much shorter. Most people miss their school time when they are able to study hard for their later life. After they have gone to work, they feel that they have no time to study.Therefore, young people are always told to treasure their school time and study hard. As people say, time iS like the water in a sponge. When you want some, you can get some if you try hard. It is also true for studying at school. Time easilv flies away when you are playing cards with friends or playing computer games through night. As school life is a very important part for you to prepare for the life on society, young people should make most of their school time to prepare for a successful life. 2008年江苏省普通高校“专转本”统一考试试卷 大学英语 第一卷(共100分) 注意事项: 1(答第?卷前,考生务必按规定要求填涂答题卡上的姓名、准考证号等项目。 2(用铅笔把答题卡上相应题号中正确答案的标号涂黑。答案不涂写在答题卡上,成绩无效。 Part ? Reading Comprehension (共20小题,每小题2分,共40分) Directions: In this part there are four passages. Each passage is followed by four comprehension questions. Read the passage and answer the questions. Then mark your answer on the answer sheet. Passage One Questions 1,5 are based on the following passage. Sometimes, people simply do not realize they are being ill mannered. Take Ted, for example. He prides himself on speaking his mind, and has something to say on everything. But his frankness is often extremely embarrassing. He is incapable of saying, ―I thought that last advertising campaign had a lot of good ideas in it, but perhaps next time we could give the copy more vitality (活力) .‖ Instead, he could say, ―That campaign was a disaster. A child of three could have done better!‖ The fact that he is often right does not help. Other employees dislike his manner even more, he is too sensitive to notice. Another character among the list of ill-mannered employees is Sally, who seems to regard just being at work as a severe punishment. Everything is done unwillingly. Asking her to do a task beyond her basic job description is often not worth the trouble. It will be done, but half-heartedly. 76 Fergus is just the opposite. He shows an over-familiarity to his boss. When an important visitor is shown into the manager’s office, Fergus cannot take the hint and leave. Instead he will attempt to take part in the conversation, declaring, ―You can talk in font of me. Henry and I don’t have many secrets, do we? ‖ Over the years Perguse has fallen behind his former equals. But he seeks to maintain the same close relationship that he imagines existed in their younger days. 1(Which of the following words describes Ted best? A. Cold B. Tactless C. Stupid D. Warm-hearted 2(It can be inferred from the passage that Ted ____. A. is well-known for his honestly B. tends to blame others rather than himself C. often gives the right idea in the wrong way D. is treated unfairly by the management 3(Which of the following is true about Sally? A. She thinks it’s unfair to have so much work to do. B. She is unhappy to help others. C. She hates being ordered about. D. She does everything half-heartedly. 4(Form the passage, we can infer that Fergus ____. A. was once close to all his colleagues B. has remained in the same position for years C. doesn’t know what a hint is D. knows everything that happens in the office 5(The writer is taking the point of view of ____. A. a colleague B. people like Ted C. an employee D. a manager Passage Two Questions 6,10 are based on the following passage. A lot of real businesses are focusing on the Internet as a way of building sales, but some people with nothing to sell are creating their own Web sites to announce wedding plans, show off pictures of their kids or just have communication. It’s not as hard as you think and it can be a lot of fun. Carol Masser has some tips on building you own Web site. ―Like my living room, or my study, it’s where I put, mementos (纪念品), collectibles (收藏物), books and paintings and things that I would like to show to my friends and relatives.‖ Renne Tubegeihan uses his own Web site to invite people into his virtual home. While John Mara shod designed his own Web site to attract other parents of disabled children. ―I’d say the goal of this site is to basically link parents in the same situation with disabled children and exchange ideas with one another.‖ Renne and John are part of a growing number of surfers who have established their own Web site. If you want to weave a Web yourself, it’s easier than you think. The first step is to get your own Web address. They’re sold by Web service providers like Microsoft, AT&T, and other local companies. You are expected to pay between 15 and 25 dollars every month to rent space for your site. But some companies, like Tripod and Geocities, issue the site for free if you agree to allow their advertising on your page. Before you get into setting up your own Web site, you should go around and do some surfing and check out other sites. Once you know what you want to do, it’s time to build. If you just want the basics, you can use Microsoft’s free Internet Assistant; or if you want to get a little fancier, you can by a software program like Adobe Page Mill for 100 dollars or Microsoft’s FrontPage Editor for 150 dollars. 6(According to the passage, some people create their own Web sites in order to ____. A. sell their personal things B. set up family business 77 C. communications with others D. show off their Web-site building skills 7(What is the main goal of John Marashod’s Web site? A. To contact parents who like to share their ideas on raising children. B. To link parents who offer free service to disabled children. C. To send information to disabled children. D. To link parents with disabled children so as to communicate with each other. 8(Some companies offer free Web address provided that ____. A. their advertisements are allowed to be put on the Web site B. the Web service providers agree to put their advertisement on their Web sites C. Microsoft and other service providers pay the monthly rent D. 25 dollars are to Microsoft, AT&T or other local companies 9(What do you need to do after you get your Web site address according to the passage? A. You need to contact a local company for free advertising. B. You need to surf some web sites so as to decide what will be included in your Web site. C. You need to learn the basic skills of creating a Web site. D. You need to buy a software program called Adobe Page Mill. 10(What does the author think about creating one’s own Web site? A. It’s very complicated. B. It’s expensive by American standard. C. It’s much easier than we think. D. It’s just too simple. Passage Three Questions 11,15 are based on the following passage. Moods, say the experts, are emotions that tend to became fixed, influencing one’s outlook for hours, days or even weeks. That’s great if your mood is a pleasant one, but a problem if you are sad, anxious, angry or simple lonely. Perhaps the best way to deal with such moods is to talk them out: sometimes, though, there is no one to listen. Modern pharmacology (药物学) offers a lot of tranquilizers (镇静剂), antidepressants and anti-anxiety drugs. What many people don’t realize, however, is that scientists have discovered the effectiveness of several non-drug approaches to make you free from an unwanted mood. These can be just as useful as drugs, and have the added benefit of being non-poisonous. So next time you feel out of sorts, don’t head for the drug-store, try the following approach. Of all the mood-altering self-help techniques, aerobic (增氧健体的) exercise seems to be the most efficient cure for a bad mood. ―If you could keep the exercise, you’d be in high spirits.‖ Says Kathryn Lance, author of Running for Health and Beauty. Researchers have explained biochemical and various other changes that make exercise compared favorably to drugs as a mood-raiser. Physical exertion such as housework, however, does little. The key is aerobic exercise—running, cycling, walking, swimming or other repetitive and tamed activities that boost the heart rate., increase circulation (血液循环) and improve the body’s utilization (利用) of oxygen. Do them for at least 20 minutes a session, three of five times a week. 11(What id the main subject of the passage? A. How to beat a bad mood. B. How to do physical exercise. C. How to talk bad moods out. D. How to get involved in aerobic exercise. 12(It can be inferred from the passage that ____. A. the best way to overcome a bad mood is to talk to oneself B. when one is in a bad mood, he or she may not work very efficiently 78 C. some drugs are more effective than physical exercise D. a person can make a big profit if he is in a good mood 13(―Feel out of sorts‖ in the second paragraph could be best replaced by ― ____". A. put things in order B. be in bad mood C. search for tranquilizers D. need a mood-raiser 14(Which of the following is NOT true according to the passage? A. Doing housework can also help you overcome bad mood. B. Aerobic exercise can make you in high spirits. C. Swimming, walking, or running is an efficient cure for a bad mood. D. Taking anti-anxiety drugs might do harm to people’s health. 15(Which of the following best describes the organization of this passage? A. A procedure is explained and its significance is emphasized. B. A problem is pointed out and solutions are given. C . Two contrasting views of a problem are presented. D. Recent pharmacological advancements are outlined in order of importance. Passage Four (非英语类学生必做,英语类学生不做,16—20小题。) The average person swallows about half-a-ton of food a year—not counting drink—and though the body is remarkably efficient at extracting (摄取) just what it needs form this huge mixture, it can only cope up to a point. If you go on eating too much of some things and not enough of others, you’ll eventually get out of condition and your health will suffer. So think before you start eating. It may look good. It may taste good. Fine! But how much good is it really doing you? What you eat and the way it affects your body depend very much on the kinds of person you are. For one thing, the genes you inherit form your parents can determine how your body metabolism (新陈代谢) copes with particular foods. The tendency to put on weight rather easily, for example, often runs in families—which means that they have to take particular care. And your parents many shape your future in another way. The way you are brought up shapes some basic attitudes to food—like whether you have a sweet tooth, nibble (吃零食)between meals, take big mountains or eat chips with everything. Eating habits, good or bad, tend to get passed on. And then there is your lifestyle. How much you spend on food ( time as well as money ), how much exercise you get—these can alter the balance between food and fitness. And finally, both your age and your sex may affect this balance. For example, you’re more likely to put on weight as get older, especially if you’re a woman. 16(Before eating something, we are advised to consider ____. A. how good it will taste B. whether we really need it C. why we want to eat it D. if we have eaten too much of it 17(We can infer from the passage that ____. A. in order to keep fit, we should spend as little time and money as possible on food B. nibbling between meals is a bad habit that will throw our body out C. if other members of our family get fat quite easily, we probably will too D. we can eat as much as we want and leave it to our body to select what it needs 18(f we love sweet things, it’s probably due to ____. A. our body’s needs B. our parent’s influence C. our basic instinct D. the taste of the food 79 19. According to the passage, which person is most likely to put on weight? A. A successful young businessman. B. A child who is picky about food. C. A man with a good appetite. D. A woman who has just retired. 20(What conclusion can we draw from this passage? A. People need a standard to guide their eating. B. Proper eating is the key to one’s health. C. Women should pay more attention to what they eat. D. The balance between food and fitness is hard to maintain. Passage Four (英语类学生必做,非英语类学生不做,16—20小题。) Mrs. P sneezes violently. Mrs. Q says, ―Bless you!‖ Mrs. P says, ―Thank you.‖ Again, this hardly seems to be a case of language being used to communicate ideas, but rather to maintain a comfortable relationship between people. Its function is to provide a means of avoiding a situation which both parties might otherwise find embarrassing. No factual content is involved. Similarly, the use of such phrases as ―Good morning‖ or ―Pleased to meet you‖, and ritual exchanges about health or the weather, do not communicate ideas in the usual sense. Sentences of this kind are usually automatically produced, and stereotyped in structure. They often state the obvious (e. g. Lovely day) or have no content at all (e. g. Hello). They certainly require a special kind of explanation, and this is found in the idea that language is here being used for the purpose of maintaining good relationship between people. The anthropologist (人类学家) Bronislaw Malinowski created the phrase ―phatic communication‖ to signal friendship—or, at least, lack of hatred. For someone to withhold these when they are expected, by staying silent, is a sure sign of distance, alienation (疏远) , or even danger. These illustrations apply to English and to many European languages. But cultures vary greatly in the topics which they permit as phatic communication. The weather is not as universal a conversation-filler as the English might like to think. For example, Rundi women (in Burendi, Central Africa), upon takingleave, are quite often heard to say, routinely and politely, ―I must go home now, or my husband will beat me.‖ Moreover, phatic communication itself is far from universal. Some culture say little, and prefect silence, as in the case of the Palicans of southern India, or the American of Colombia. 16(What is the subject of the passage? A. Language used to communication ideas. B. Difference between English of language use. C. A special social function of language use. D. Relationship between language and culture. 17(In Paragraph 1, the author implies that ____. A. people usually think language is only used to communicate ideas B. the exchange about health or the weather is a universal topic C. people will always say something when facing embarrassment D. the example of Mrs. P and Mrs. O is common in all cultures 18(The phrase ―stereotyped in structure‖ in Paragraph 2 probably means ____. A. sentence of this kind have a variety of meanings B. this kind of sentence has no content at all C. the structure of these three sentences is always changing D. sentence of this kind usually have a fixed structure 19(The phrase ―phatic communication‖ in Paragraph 3 can be explained as ____. A. exchange of ideas B. explanation of facts 80 C. exchanges about heath or the weather D. exchange for maintaining social relationship 20(In the last paragraph, the author concludes that ____. A. the weather is a universal topic for human communication B. phatic communication differs form culture to culture C. the English and people in may Europeans countries have common language for phatic communication D. every culture has its own way to communicate ideas Part ? Vocabulary and Structure(共40小题,每小题1分,共 40分) Directions: In this part there are 40 incomplete sentences. Each sentence is followed by four choices. Choose the one that best completes the sentence and then mark your answer on the answer sheet. 21. His salary as a bus driver is much higher than ____. A. that of a teacher B. those of a teacher C. these of a teacher D. this of a teacher 22. While Tim was walking in the street, he came ____an old classmate of his. A. into B. across C. onto D. up with 23. Hardly ____making the speech when the people stood up applauding. A. has the speaker finished B. the speaker had finished C. had the speaker finished D. had finished the speaker 24. Human behavior is mostly a product of learning, whereas the behavior of an animal depends mainly on ___. A. consciousness B. instinct C. impulse D.reaction 25. If you ____in taking this annoying attitude, we’ll have to ask you to leave. A. persist B. insist C. resist D. assist 26. There are not many teachers who are strong ____of traditional methods in English teaching. A. sponsors B. advocates C. contributors D. performers 27. Please see to it ____no one comes in without identification. A. that B. lest C. when D. which 28. We had ____enough Russian to be table to read the instructions. A. set up B. picked up C. taken up D. made up 29. Only those who are ____of their lagging behind are more likely to catch up. A. awake B. visual C. aware D. illusive 30. It is suggested that smoking ____in public places. A. will not be allowed B. was not allowed C. not be allowed D. is not allowed 31. In ____to the Party’s call, a great number of doctors and nurses went to the frontline to fight the flood. A. return B. admission C. order D. response 32. You could not persuade him to accept it, ____make him see the importance of it. A. no more than you could B. if only you could not C. nor could you D. or you could not 33. The 20-year-old tennis play’s dream is to ____China at 2008 Beijing Olympic Games. A. support B. present C. represent D. compete 34. Britain is proud of her great poets, just s Italy is proud of her painters, and Germany ____of her composers. A. / B. is C. proud D. as 35. It is no use just telling me to do it; suggest some ____ways to do it. A. active B. positive C. passive D. negative 81 36. One can’t see well through a through a telescope without correctly ____it to one’s sight. A. balancing B. adopting C. repairing D. adjusting 37. ____children have been taken out of school and taught by their parents at home. A. A number of B. A great deal of C. An amount of D. The number of 38. There is much ____can be done about the accidents from carelessness. A. which…arose B. that…arising C. which…arisen D. that…arise 39. Those of us who work in the coal mine should have ____lungs checked regularly. A. our B. their C. his D. my 40. To their disappointment, there are no textbooks ____for the students of Class Two. A. possible B. preferable C. considerable D. available 41. More and more people are beginning to earn English, ____can partly explain the booming of language training centers in China. A. which B. what C. that D. it 42. The higher a rocket flies, ____air it meets. A. the fewer B. the little C. the less D. the much 43. It is during his spare time ____Johnson has been studying a course in history. A. when B. which C. what D. that 44. The ____of modern life in bi cities is so quick that some old people feel it hard to keep up with it. A. way B. step C. pace D. routine 45. Some teachers devote too much time to helping their shower students and ____the brighter ones. A. neglect B. isolate C. frustrate D. corrupt 46. —Have you seen Mary today? —No, I think she ____away on her vacation. A. must be B. may have been C. must have been D. might have been 47. Notice a person’s reaction ____stress in these situations, and you will soon find a solution ____this problem. A. of…to B. of…of C. to…of D. to…to 48. All the books can be used to ____your reading comprehension. A. training B. train C. be trained D. being trained 49. She did not know whether to sell her books or ____. A. to keep them for reference B. keeping them for reference C. if she should keep them for reference D. to be kept for reference 50. She wears very ____ring that her boyfriend gave her as a birthday gift. A. gold new valuable B. valuable gold new C. valuable new gold D. valuable and gold 51. The headquarters of Mr. Zhang’s company was ____located in Tianjin, but now it is in Shanghai. A. formally B. firmly C. formerly D. fairly 52. We don’t mind a bit you bring your friends in for a dinner, but it is rather too much when sixteen people came ____for dinner. A. consequently B. expectedly C. excessively D. unexpectedly 53. Our point is that nuclear science should be developed to benefit people ____harm them. 82 A. more than B. other than C. rather than D. better than 54. It is generally agreed that textile industry used to ____greatly to the economy of our city. A. add B. contribute C. lead D. stimulate 55. ____more time, she would certainly have done it much better. A. Given B. To be given C. Giving D. To give 56. The car was running so fast that it crashed into the truck and the driver was killed ____the spot. A. on B. at C. by D. to 57. Fortunately, the government has taken some measures to bring down the rate of inflation to a (an) ____level. A. unpredictable B. manageable C. questionable D. sustainable 58. When I saw the mess my paper was in, it was obvious that someone ____it. A. was reading B. would have read C. had been reading D. had read 59. I am firmly ____that this plan would do much good to our company. A. recommended B. amazed C. convinced D. satisfied 60. He was hoping they would take his suggestion, but for some reason they ____. A. turned it over B. turned it off C. turned it down D. turned it up Part ? Cloze(共20小题,每小题1分,共20分) Directions: There are 20 blanks in the following passage. For each blank there are four choices. Choose the one that best fits into the passage and then mark your answer on the answer sheet. (非英语类学生必做,61,80小题。) A schoolboy’s life is in preparation for the real battle of life. It is also 61 of differences and interests. One of the 62 important parts of a schoolboy’s life is to get 63 knowledge and good mind-training as he can. His 64 business in school is to learn. He 65 read the book he is 66 in the classes. He has to do the homework set to him. 67 part that 68 a schoolboy’s life is the school discipline. At school there are 69 rules 70 . This strict discipline is very 71 for him when he 72 the society to 73 a living. It teaches him some very necessary virtues on the road to a 74 life. School is place for a schoolboy to learn what the social life is 75 . 76 in the classroom and the playground, he has to catch up with his fellows and not members of his family. He can not behave 77 he does in his home. He is no longer a spoiled child, and his school fellows will not give 78 to his wished. He soon gets his corners robbed off and learns the lesson of give-and-take, good manners, and thought for 79 . This is 80 the way when he has to carry himself in society. 61. A. sure B. fond C. full D. short 62. A. larger B. greater C. least D. most 63. A. as many B. more C. as much D. little 64. A. mostly B. main C. almost D. partly 65. A. likes B. obliges C. requires D. has 66. A. learner B. studied C. Taught D. examined 67. A. Other B. Another C. Else D. Rest 68. A. contains B. forms C. consists of D. makes up 69. A. loose B. hard C. strict D. kind 70. A. kept B. being kept C. strict D. kind 71. A. useful B. harmful C. useless D. harmless 83 72. A. leaves B. comes C. enters D. organizes 73. A. produce B. make C. get D. take 74. A. famous B. successful C. social D. ideal 75. A. like B. on C. up D. alike 76. A. Since B. As C. For D. In that 77. A. until B. as C. by D. with 78. A. in B. up C. off D. out 79. A. himself B. teachers C. others D. parents 80. A. too B. as well C. either D. also (英语类学生必做,61,80小题。) Have you ever seen a movie in which a building was burned down or a bridge was 61 ? Have you seen a film in which a train crashed or a ship sank 62 the ocean? If so, you 63 how these things could happen without 64 the people in the film. The person 65 knows the answer is the ―special-effect‖ person. He has one of the most important hobs in the film 66 . In a 67 for one movie there was a big glass bowl filled 68 water in which small fish were swimming. The director of the movie wanted the fish to stop swimming suddenly and 69 they seemed to stare at an actor. Then the director wanted the fish to stop 70 and swim away. But fish can’t be ordered to do anything. It was quite a 71 . The special effects person thought 72 this problem for a long time. The 73 was an idea for controlling the fish with a 74 use of electricity. First, he applied electricity 75 the fish bowl, causing thefish to be absolutely still. Then he rapidly reduced the 76 of electricity, allowing the fish to swim away.77 he got the humorous effect that the director wanted.78 special effects require training, skill, and experience. It also adds a great deal 79 the expense of producing the film. It helps explain 80 so many movies are expensive to make. 61. A. destroyed B. collapsed C. jammed D. decreased 62. A. to B. into C. down D. back to 63. A. can wonder B. can have wondered C. wonder D. may have wondered 64. A. breaking B. damaging C. harming D. disturbing 65. A. which B. who C. whoever D. whichever 66. A. industry B. job C. performance D. growth 67. A. view B. sight C. scene D. situation 68. A. over B. of C. in D. with 69. A. since B. for C. while D. till 70. A. to stare B. to stare at C. staring D. staring at 71. A. problem B. question C. fault D. mistake 72. A. out B. of C. about D. with 73. A. effect B. influence C. result D. end 74. A. harmless B. useless C. careless D. helpless 75. A. at B. to C. in D. toward 76. A. amount B. number C. distance D. strength 77. A. Nevertheless B. Because C. Anyway D. Thus 78. A. The all B. All the C. Most the D. Almost the 79. A. in B. with C. on D. to 80. A. what B. why C. which D. when 84 第?卷(共50分) Part ? Translation(共35分) Section A(共5小题,每小题4分,共20分) Directions: Translate the following sentences into Chinese. You may refer to the corresponding passages in Part ? . 81. Another character among the list of ill-mannered employees is Sally, who seems to regard just being at work as severe punishment. (Passage One) 82. …some people with nothing to sell are creating their own Web sites to announce wedding plans, show off pictures of their kids or just have communication. (Passage Two) 83. What many people don’t realize, however, is that scientists have discovered the effectiveness of several non-drug approaches to make you free from an unwanted mood. (Passage Three) 84. (非英语类学生必做,英语类学生不做)What you eat and the way it affects your body depend very much on the kinds of person you are. (Passage Four) 85. (非英语类学生必做,英语类学生不做)For one thing, the genes you inherit form your parents can determine how your body metabolism (新陈代谢) copes with particular foods. (Passage Four) 84. (英语类学生必做,非英语类学生不做)Its function is to provide a means of avoiding a situation which both parties might otherwise find embarrassing. (Passage Four) 85.(英语类学生必做,非英语类学生不做)Similarly, the use of such phrases as ―Good morning‖ or ―Pleased to meet you‖, and ritual exchanges about health or the weather, do not communicate ideasin the usual sense. (Passage Four) SectionB(共5小题,每小题3分,共15分) Directions: Translate the following sentences into English. 86. 汤姆失业了,不是因为他年岁过大,而是因为他懒惰。 87. 玛丽连伙食费都不够,更不用说娱乐了。 88. 她一听见脚步声就不唱了。 89. 据说李先生多次到过澳大利亚。 90. 学好外语必须大量练习,正向成语所说―熟能生巧‖。 Part V Writing (共15分) Directions: In this part you are required to write an essay about the sports you like. You should write at least 120 words and base your composition on the outline given below. 1. What kinds of sports do you like? 2. Why do you like them? 3. When and how do you play sports? 2008年普通高校专转本统一考试英语试卷参考答案 Part I Reading comprehension Passage One 本文谈论的是言行举止。有时人们不会意识到不良的言行举止,并举三例加以说明。 1(B。Tactless是―缺乏策略‖之义,符合Ted说话太直白的个性。 2(C。文中―The fact that he is often right does not help(‖与此选项对应。 3(B。文章第四段结尾两句点明答案。 4(B。最后一段―Over the years Fergus has fallen behind his former equal,‖暗含选项B。 5(D。文章多处用employee一词,全文语气有一种从上观下之感。 85 Passage Two 本文谈论个人网页的口趋流行以及制作个人网页的步骤和一些软件工具。 6(C。第一段前三行点明了答案。 7(D。第二段John Marashod的原话与选项D内容一致。 8(A。第三段最后一句与此选项内容一致。 9(B。第四段开头两句点明了答案。 10(C。第一、第三段中有两处点明了答案。 Passage Three 本文谈论情绪,以及消除不良情绪的诸多良方。 11(A。beat在此是―应付、处理‖之义。 12(B。文章第一段暗含此意。 13(B。第三段介绍了应对不良情绪的方法,由此可知feel out of sorts即是be in a bad mood。 14(A。第四段中...home work...does little否定了A项的正确性。 15(B。细读文章可知,作者先提出问题bad moods,然后提出各种solutions。 Passage Four(非英语类学生必做) 本文谈论饮食与健康之间的关系,以及个性之间的差异。 16(B。第一段指出,偏食不利健康。D项概括不全。 17(C。第二段最后一句中runs in family是―家族遗传‖之义。 18(B。第三段第一、第二句说明,爱吃甜食,是受父母的影响。 19(D。第五段说随着年龄的增长,人容易发胖,尤其是老年妇女。 20(B。此选项紧扣文章主旨。 Passage Four(英语类学生必做) 本文谈论习惯用语的社会功用。 16(C。本文谈论的是习惯用语的社会功用。 17(A。从另一面印证文章的主旨。 18(D。stereotyped是―固定模式‖之义。 19(D。第三段第一句点明了答案。 20(B。最后一段第二句点明了答案。 Part? Vocabulary and Structure 21(A。salary为单数,用that替代。 22(B。come across的意思是―偶然遇见‖。 23(C。倒装句,主句一般用过去完成时态。 24(B。1nstlnct是―本能‖之义。 25(A。persist in doing sth(是―坚持做某事‖之义。 26(B。advocate义为―倡导者‖。 27(A。see to it that…为固定结构,义为―确保‖。 28(B。pick up此处义为―学习‖,―习得‖。 29(C。be aware of…固定短语,义为―意识到……‖ 30(C。suggest后跟宾语从句,用虚拟语气。此处省略了―should‖。 31(D。in response to…义为―响应……‖ 32(C。第二句承接第一句为否定,此处用倒装。 33(C。represent义为―代表‖。 34(A。承前句子结构,此处省略了is proud两词。 35(B。positive way义为―行之有效的方法‖。 86 36(D。调整望远镜方能远观,故选。 37(B。children为可数名词,故选。 38(B。much后的定语从句要用that引导。about后接的不是句子,故用arising。 39(A。主语为those of us,故选用our相配。 40(D。available是常考词汇,义为―提供给‖。 41(A。非限制性定语从句,用which引导。 42(C。air为不可数名词,故不选A项。 43(D。强调句用that引导。 44(C。生活节奏用pace一词。 45(A。neglect义为―忽略‖。 46(C。表示肯定的猜测。 47(D / C。reaction后接介词to。solution后可接3个介词:of, for, to. 48(B。can be used to是被动语态结构,to后接动词原形。 49(A。whether后接to引导的不定式,or后同样接to引导的不定式,使结构对称。 50(C。名词前形容词的顺序。 51(C。forerly表示以前,与now一词相呼应。 52(D。 53(C。rather than义为―而不是……‖ 54(B。contribute…to…义为―对……作出贡献‖。 55(A。事情已经发生,故不能选用B项。 56(A。on the spot义为―当场‖。 57(D。sustainable义为―可承受的‖。 58(D。表示已经发生过的事。 59(C。I am firmly convinced…义为―我坚信……‖ 60(C。turn down在此处是―拒绝‖之义。 Part ? Cloze(非英语类学生必做) 61(C。be full of为―充满‖之义。 62(D。构成最高级。 63(C。as much…as he can构成固定句式。 64(B。business是名词,应用形容词main修饰。 65(D。与下一句形成平行句式。 66(C。此句是被动语态,故选taught。 67(B。part为单数,故选此项。 68(D。make up义为―由不同部分构成某个整体‖。 69(C。strict rules是―严格的校规‖之义。与下一句呼应。 70(D。用不定式被动语态。 71(A。此处是肯定语气,故选此项。 72(C。进入社会。 73(B。make a living义为―谋生‖。 74(B。成功人生。 75(A。义为―社会的真实性‖。 76(C。for引导表示原因的从句。 77(B。作连词,义为―像……一样‖。 78(A。give in to his wishes义为―听随某人所愿‖。 87 79(C。为他人着想。 80(D。此句为类比,故选also。 (英语类学生必做) 61(B。collapse多指房屋或桥梁倒塌。 62(B。沉入。 63(D。用完成时态,表示此想法由来已久。 64(C。伤害之义。 65(B。与person呼应。 66(A。film industry为―电影产业‖之义。 67(C。某一场境。 68(D。be filled with固定词组。 69(C。表示两个动作同时进行。 70(C。stop后接动名词,表示停止做正在做的事。 71(A。让鱼听话,可是个难题。 72(C。think about为―考虑‖之义。 73(C。考虑所得出的结果。 74(A。用电量不能伤及池鱼。 75(B。apply…to…固定搭配。 76(A。电量是不可数名词,用amount。 77(D。以此方法之义。 78(B。此题是语法题―all the+名词‖是习惯用法。 79(D。add…to…固定词组。 80(B。电影成本高之原因。 Part ? Translation Section A 81(行为不端的雇员中的另一个人是Sally。她似乎把干活当作一种惩罚。(Passage One) 82(有些无所售卖的人建立自己的网站是为了发布结婚讯息,展示自己孩子的相片,或者只是为了建立联系。( Passage Two) 83(然而,许多人没有意识到的是,科学家已经发现了几种有效的非药物方法,使你摆脱不良情 绪。(Passage Three) 84((非英语类学生必做)你吃下的食物以及这些食物时你具体的影响在很大程度上因人而异。 (Passage Four) 85((非英语类学生必做)一方面,从父母身上遗传的基因能够决定你的新陈代谢将如何处理某些特殊的食物。( Passage Four) 84((英语类学生必做)其作用是提供一种方式,以避免使双方都尴尬的情形。( PassageFour) 85((英语类学生必做)同样,―早上好‖或者―很高兴见到你‖这类词句以及关于健康或天气的寒喧都不传达通常意义上的思想。( Passage Four) Section B 86. Tom was out of job,not because of his old age,but of his laziness. 87. Mary cannot afford her accommodation, say nothing of her entertainment. 88. She stopped singing as soon as she heard footsteps. 89. It is said that Mr. Li has been to Australia several times. 90. To have a good command of English needs a lot of practices, just as the proverb goes ―Practice makes perfect. " 88 Part ? Writing In recent years,more and more people are recognizing the importance of health. Doing sports is said to be one of the best ways to keep fit. You can see people of all ages doing sports in public gardens, in gyms and on the sports fields.I like many kinds of sports,of which I like basketball most. There are many reasons for my preference.The most important reasons are the following ones. First of all, playing basketball is of great fun. It is a good opportunity to communicate with your friends. In addition, playing basketball is a team job. The players have to learn to cooperate if they do not want to be defeated. Last but not Ieast,playing basketball needs a lot of energy,which makes me physically and mentally strong enough to cope with the stress in my study and my life. After school and at weekends, I often play basketball with my classmates or friends.I am practicing hard. I have confidence to be a member in the school team before long. Love basketball,love yourself. 2009年江苏省普通高校“专转本”统一考试试卷 大学英语 第?卷(共100分) 注意事项: 1(答第?卷前,考生务必按规定要求填涂答题卡上的姓名、准考证号等项目。 2(用铅笔把答题卡上相应题号中正确答案的标号涂黑。答案不涂写在答题卡上,成绩无效。 Part ? Reading Comprehension(共20小题,每小题2分,共40分) Directions: In this part there are four passages. Each passage is followed by four comprehension questions. Read the passage and answer the questions. Then mark your answer on the answer sheet. Passage One Questions 1,5 are besed on the following passage. Young Koreans are beginning to do it alone when it comes to finding a partner, though matchmaking is still the most common way for boys to meet girls. Professional matchmakers can make thousands of American dollars by introducing suitable marriage partners to each other, but partners also play a role in the process during which young Koreans meet. In Confucius Korea, where marriage is regarded as more of a business contract than a sacred thing, the scene of the first meeting is repeated hundreds of times a day in coffee shops in the main hotels around Seoul. The business of continuing the family lineage (血统)and keeping the blooding pure is often too important to be left to romance and chance encounters. Often, the girl will work out a system of secret signals with her mother, from which her parents can tell if she is interested. For example, if the girl orders a coffee it might mean that she wants her parents to leave her alone with the boy, while a milk shows that she wants them to stay. Sometimes the matchmaking is not always so formal, with the introduction being made by friends. But whether through friends or families, there is hardly a Korean man in the country who has not gone through this process --- sometimes six or seven times. 1. The word ―matchmaking‖ in the passage means ____. A. a very formal ritual(仪式)attended by boys and girls B. introducing boys and girls to know each other for the purpose of marriage C. producing matches to make a fire or light a cigarette D. arranging games between men and women 89 2. The fact that the first meeting is repeated again and again in coffee shops in the main hotels suggests that ____. A. Koreans like drinking coffee in coffee shops very much B. men and women want to meet as many times as possible C. they are busy with communicating with different people D. marriage is seen as a business contract instead of something sacred 3. Why does the matching still exist in Korea? A. Because most people want to keep the bloodlines pure and chance encounters romantic. B. Because most people try to avoid romance and keep the bloodlines pure. C. Because most people regard marriage as continuing family lineage and romance. D. Because most people try to break family lineage and bloodlines. 4. Which of the following is true according to the passage? A. Almost every man or woman in Korea has to go through the matchmaking. B. From the signal the boy’s parents know whether their son is interested in the girl or not. C. At present young Koreans no longer need matchmaking to help them find partners. D. Only a girl and a boy will appear in the first arranged meeting. 5. The passage is mainly about ____. A. the practice of matchmaking in Korea B. the importance of a business contract C. the work of professional matchmakers D. the first meeting between men and women Passage Two Questions 6,10 are based on the following passage. In the university Jim was a fast-tracker. He made good grades with little effort, and his classmates thought of him ―most likely to succeed‖. After graduation, he joined a large company and at first did well. However, he switched to several smaller companies where the same pattern developed time and again: well-liked, regarded as a fast-tracker. People wonder why he isn’t doing better. Then there was Tom who was always regarded as ―average‖. However, he set his goals high, thenfound a way to achieve his goal. Today he owns a million-dollar company. Researchers have found that school performance is little related to job performance. Qualities like ―steady and dependable‖ and ―practical and organized‖ are more important. ―You don’t need talent to succeed‖, insist some experts. ―All you need is a big pot of glue (胶水). You put some on your chair, you sit down, and you stick to every project until you’ve done the best you can do.‖ Average achievers stay glued to their chairs and postpone pleasure so they can receive future benefits. Many fast-trackers, on the other hand, expect too much too soon. When rewards don’t materialize instantly, they may become disappointed and unhappy. 6. A fast-tracker in this passage refers to a person who ____. A. feels happy with everything B. makes others disappointed and unhappy C. is an excellent student D. learns new things quickly 7. According to the passage, which of the following is true about a fast-tracker? A. He is very talented in his work. B. He is not likely to get rich. C. He may not stick to his work. D. He is dependable and practical. 8. The word ―materialize‖ in the last paragraph means ―____". A. matter B. appear C. match D. attract 90 9. It is implied in the passage that ____. A. one has no pleasure until he succeeds B. glue can help one focus on one’s work. C. success has nothing to do with special talent D. success is closely related to patience and steadiness 10. A proper title for the passage can be ____. A. The magic of Glue B. The Way to Success C. Why Talented People Often Succeed Quickly D. How average Achievers Do Better than Others Passage Three Questions 11,15 are based on the following passage. From good reading we can derive pleasure, companionship, experience, and instruction. A good book may absorb our attention so completely that for the time being we forget our surroundings and even our identity. Reading good books is one of the greatest pleasures in life. It increases our contentment when we are cheerful, and lessens our troubles when we are sad. Whatever may be our main purpose in reading, our contact with good books should never fail to give us enjoyment and satisfaction. With a good book in hand we need never be lonely. Whether the characters portrayed are taken from real life or are purely imaginary, they may become our companions and friends. In the pages of books we can walk with the wise and the good of all lands and all times. The people we meet in books may delight us either because they resemble human friends whom we hold dear or because they represent unfamiliar types whom we are glad to welcome as new acquaintances. Our human friends sometimes may bore us, but friends we make in books need never weary us with their company. By turning the pages we can dismiss them without any fear of hurting their feelings. When human friends desert us, good books are always ready to give us friendship, sympathy and encouragement. One of the most valuable gifts bestowed by books is experience. Few of us travel far from home or have a wide range of experiences, but all of us can lead varied lives through the pages of books. Whether we wish to escape from the seemingly dull realities of everyday life or whether we long to visit some far-off place, a book will help us when nothing else can. To travel by book we need no bank account to pay for our way, no airship or ocean liner or streamlined train to transport us, no passport to enter the land of our heart’s desire. Through books we may get the thrill of hazardous adventure without danger. We can climb lofty mountains or cross the scorching sands of the desert, all without hardship. Indeed, through books the whole world is ours for the asking. The possibilities of our literary experiences are almost unlimited. The beauties of nature, the enjoyment of music, the treasures of art, the triumphs of architecture, the marvels of engineering, are all open to the wonder and enjoyment of those who read. 11. The main idea in the first paragraph is ____. A. pleasure can only be derived from reading good books B. a good book may distract our attention C. people can read good books in different ways D. enjoyment and satisfaction can be derived from reading good books 12. The main reason that people like their acquaintances in books is ____. A. they are like human friends exactly B. they never bore us in comparison with our human friends C. they never hurt our feelings 91 D. they give human beings friendship, sympathy and encouragement 13. We forget our surroundings and even our identity because ____. A. the reading environment is very good B. the book is extremely interesting and attractive C. we are alone and no one interrupts us D. we are asked to return the book the second day 14. Which of the following is NOT true? A. All of us can lead varied lives through the pages of books. B. We can travel by book free of charge to our heart’s content. C. We should rely on reading to escape from the seemingly dull realities of everyday life. D. Valuable experiences can be obtained from reading good books. 15. ―The whole world is ours for the asking‖ implies that ____. A. the world is more accessible in books than in reality B. the world is easy to travel around as long as we walk C. the world belongs to us whenever we ask D. round-world trips can be made by everyone Passage Four(非英语专业做) Questions 16 to 20 are based on the following passage. Increasingly over the past ten years, people --- especially young people --- have become aware of the need to change their eating habits, because much of the food they eat, particularly processed foods, is not good for health. Consequently, there has been a growing interest in natural foods: foods which do not contain chemical additives and which have not been affected by chemical fertilizers widely used in farming today. Natural foods, for example, include vegetables, fruit and grain which have been grown in soil that is rich in organic matter. In simple terms, this means that the soil has been nourished by unused vegetable matter, which provides it with essential vitamins and minerals. This in itself is a natural process compared with the use of chemicals and fertilizers, the main purpose of which is to increase the amount --- but not the quality --- of foods grown in commercial farming areas. Natural foods also include animals which have been allowed to feed and move freely in healthy pastures. Compare this with what happens in the mass production of poultry: there are battery farms, for example, where thousands of chickens live crowded together in one building and are fed on food which is little better than rubbish. Chickens kept in this way are not only tasteless as food; they also produce eggs which lack important vitamins. It is significant that nowadays fiber is considered to be an important part of a healthy diet. In white bread, for example, the fiber has been removed. But it is present in unrefined flour and of course in vegetables. It is interesting to note that in countries where the national diet contains large quantities of unrefined flour and vegetables, certain diseases are comparatively rare. Hence the emphasis placed on the eating of whole meal bread and more vegetables by modern experts on ―healthy eating‖. 16. People have become more interested in natural foods because ____. A. they like changing their diet from time to time B. they want to eat food that is more delicious C. much of the food they eat is no longer considered to be healthy D. they want to be fashionable 17. Soil that is rich in organic matter ____. A. has nothing nutritious added to it B. is essential for producing fruit and vegetables 92 C. contains unused vegetable matter D. has had chemicals and fertilizers added to it 18. According to the author, the use of chemicals and fertilizers ____. A. will not necessarily increase the production of foods B. is a good way to improve the quality of foods C. promotes the foods both in quality and in quantity D. is no good for health 19. Chickens can be called ―natural foods‖ only when ____. A. they are delicious B. they are fed on food little better than rubbish C. they lay eggs rich in vitamins every day D. they are allowed to move about and eat freely 20. Which of the following is considered good for healthy eating? A. Unrefined flour and vegetables. B. White bread and honey. C. Canned beans and fruits. D. Mass-produced chickens and eggs Passage Four(英语专业必做) Questions 16,20 are based on the followlng passage. Drunken driving-sometimes called America's socially accepted form of murderhas become a national epidemic. Every hour of every day about three Americans on average are killed by drunken drivers, adding up to an incredible 250,000 over the past decade. Heavy drinking used to be an acceptable part of the American macho(男子汉)image and judges were lenient in most courts, but the drunken slaughter has recently caused so many well-publicized tragedies, especially involving your children, that public opinion is no longer so tolerant. Twenty states have raised the legal drinking age to 21, reversing a trend in the 1960s to reduce it to 18. After New Jersey lowered it to 18, the number of people killed by 18-20-year-old drivers more than doubled, so the state recently upped it back to 21. Reformers, however, fear raising the drinking age will have little effect unless accompanied by educational programs to help young people to develop―responsible attitude‖about drinking and teach them to resist peer pleasure to drink. New laws have led to increased arrests and tests in many areas already, to a marked decline in fatalities. Some states are also penalizing bars for serving customers too many drinks. A tavern(酒栈) in Massachusetts was fined for serving six or more brandies to a customer who was―obviously intoxicated‖and later drove off the road, killing a nine-year-old boy. As the fatalities continue to occur daily in every state, some Americans are even beginning to think well of the 13 years of national prohibition of alcohol that began in 1919, which President Hoover called the "noble experiment". They forgot that legal prohibition didn't stop drinking, but encouraged political corruption and organized crime. As with the becoming drug trade, there is no easy solution. 16. Drunken driving has become a serious problem in America because ____. A. most drunken drivers drive their cars at top speed B. most drivers regard heavy drinking are part of the American macho image C. fatalities caused by drunken driving have been greatly increased D. about 25,000 people on average are killed every year by drunken drivers 17. Why have the public changed their opinion about drunken driving? A. Because detailed statistics on drunken slaughter are now available. 93 B. Because they are no longer tolerant of the drunken slaughter. C. Because judges usually give more severe sentences to drunken divers. D. Because drunken drivers are conscious of their image. 18. The statistics issued in New Jersey suggested that ____. A. many drunken drivers were not of legal age B. many young people were in no way qualified drivers C. raising the legal drinking age would have little effect on the reduction of tragedies D. raising the legal drinking age would reduce the fatalities caused by drunken drivers 19. Laws recently introduced in some states have ____. A. reduced the number of convictions B. resulted in fewer fatalities of traffic accidents C. prevented bars from serving drunken drivers D. specified the amount drivers can drink 20. What's the author's attitude towards the solution to drunken driving in the United States? A. IronicaL B. Causal. C. Optimistic. D. Realistic. Part ? Vocabulary and Structure(共40小题,每小题1分,共 40分) Directions: In this part there are 40 incomplete sentences. Each sentence is followed by four choices. Choose the one that best completes the sentence and then mark your answer on the answer sheet. 21. Keep two extra pencils ____ while taking the examination. A. at ease B. at length C. at hand D. in turn 22. The competition from experienced staff members, some of whom are higher in rank, ____ to my disadvantage. A. works B. working C. work D. worked 23. I don’t know what ____ Alfred to read your letter. ____ he thought it was addressed to him. A. made…Supposedly B. induced…Presumably C. stimulated…Possibly D. introduced…Probably 24. Never before ____ available for quick and easy access in so many different fields of study. A. so much free information were B. were so much free information C. has so much information been D. so much free information has been 25. It’s true that not having a car imposes some ____, but owning one also complicates life in many ways. A. constructions B. introductions C. instructions D. restrictions 26. Word got around ____ he had resigned his position as executive secretary of the committee. A. what B. that C. whatever D. which 27. If you invest so heavily in the stock market, you’re ____ risks. A. turning in B. heading off C. turning on D. heading for 28. A further ____ into the cause of the disaster revealed that the director was partly to blame because he had not insisted on adequate precautions. A. entry B. request C. inquiry D. conquest 29. It is only in the most difficult circumstances ____ a man’s abilities are fully tested. A. when B. that C. where D. so that 30. Adults tend to interpret the action of children ____ their own experience. A. in terms of B. in the habit of C. in words of D. in the account of 94 31. In fact, the Iraq war was fairly ____ across the West. A. conversational B. contradictory C. contrary D. controversial 32. ―You missed a golden opportunity.‖ "Yes, I ____ that job when it was offered.‖ A. must have taken B. should have taken C. might take D. ought to take 33. The use of military force faces strong opposition among key US allies where opinion polls show ____ majorities of the populations support a peaceful solution. A. substantial B. enormous C. considerable D. overwhelming 34. There is every sign that most of the Asian Americans have been ____ the nation’s culture. A. assimilated into B. involved into C. associated with D. connected with 35. Professor Wu told us that by the end of the years he ____ here for three years. A. will have worked B. will have been working C. would have been working D. has been working 36. We cannot choose whether we will pay income tax or not, because payment of income tax is ____. A. compulsive B. impulsive C. comprehensive D. compulsory 37. You’re the manager’s relative, but that doesn’t ____ you to a free meal in our restaurant. A. qualify B. entitle C. adapt D. allow 38. Believe it or not, Matt earns ____ his brother, who has a better position in a big company. A. much as twice as B. as twice much as C. twice as much as D. as much twice as 39. Please keep the numbered cards in ____; don’t mix them up. A. series B. succession C. sequence D. procedure 40. The whole world is now clearly aware of the ____ to which global warming has affected the earth. A. extent B. range C. grade D. limit 41. What would happen if students were ____ of books? A. refused B. declined C. denied D. deprived 42. There are two major problems linked with atomic power plants, ____ the first concerns nuclear waste. A. of which B. of the two C. of them D. of the plants 43. It seems that there is no resolution to this racial ____. A. convention B. compromise C. conflict D. conservation 44. There’s no doubt that your achievement ____ your hard work. A. contributed to B. is attributed to C. is led to D. resulted in 45. Look at the terrible situation I am in now! If only I ____ your advice. A. follow B. had followed C. would follow D. have followed 46. The policeman on duty became ____ of the youth and asked him why he was standing there. A. doubtful B. suspicious C. suspected D. hesitated 47. Americans are trained to think themselves as separate individuals, ____ most Chinese consider themselves as members of a group. A. whereas B. as C. since D. like 48. It’s no good ____. You just have to put up with all these. A. complained B. complaining C. to be complaining D. to complain 49. As we are unlikely to find another hotel before dark, the ____ action is to stay here for the night. 95 A. senseless B. sensitive C. sensational D. sensible 50. He hopes that when he comes back in five years’ time all the old buildings ____ down. A. will have been pulled B. will be pulling C. will have pulled D. will be pulled 51. It has never ____ to her that it was her son who committed such a silly mistake. A. occurred B. happened C. arisen D. seemed 52. Quite a number of NBA players have the experience of ____ from one team to another. A. transforming B. transferring C. transplanting D. transmitting 53. The atmosphere is as much a part of the earth as ____ its soils and the water of its lakes, rivers and oceans. A. is B. has C. are D. have 54. Trying to ____ wild life is a job that concerns all of us. A. deserve B. detect C. protect D. reserve 55. ____, the next problem was how to make a good plan. A. Having made the decision B. The decision having been made C. Has the decision been made D. The decision has been made 56. Steven hawking is now confined ____ the hospital by illness. A. in B. to C. with D. at 57. The rapid development China has achieved in the last decade ____ that the future is likely to be very exciting. A. make it clear B. it has made clear C. it makes clear D. has made it clear 58. Football players are ____ males, though there are some females. A. most B. mostly C. much D. all 59. Tom has been most helpful to us; in fact I don’t know what we ____ without him. A. have done B. will have done C. had done D. would have done 60. I don’t have any ____ plan for the weekend ---- I was thinking of spending some time with my family or perhaps getting together with some friends. A. confident B. conservative C. concrete D. considerate Part ? Cloze (共20小题,每小题1分,共20分) (非英语类学生必做,61,80小题。) Directions: There are 20 blanks in the following passage. For each blank there are four choices. Choose the one that best fits into the passage and then mark your answer on the answer sheet. There was a time when parents who wanted an educational present for their children would buy a typewriter, a globe or an encyclopedia set. Now those 61 seem hopelessly old-fashioned: this Christmas, there were a lot of 62 computers under the tree. 63 that computers are the key to success, parents insist that children 64 taught to use them in school as early as possible. The problem for schools is that when it 65 computers, parents do not always know best and are eagerly urging the schools to offer computer courses as soon as possible. Many schools are 66 parental impatience and are purchasing hardware hastily 67 good educational planning, saying ―Ok, we’ve moved into the computer age.‖ Teachers 68 themselves caught in the middle of the problem between parental pressure and 69 educational decisions. Educators do not even agree 70 how computers should be used. A lot of money is going for computerized educational materials 71 can be taught 72 with pencil and paper. Even those who believe that all children should 73 to computer warn that there are potential 74 to the very young. The temptation of the computer is so strong that 96 young children who quickly 75 themselves to it are never aware how much time they have 76 on it. There are some other problems. 77 every school can afford to go into computing, and there is a division 78 the haves and have-nots. Very few parents ask 79 computer instruction in poor school districts, 80 there may be barely enough money to pay the reading teacher. 61. A. items B. toys C. sets D. series 62. A. private B. children C. school D. personal 63. A. Giving B. Providing C. Convinced D. Believed 64. A. was B. be C. are being D. were 65. A. talks about B. comes to C. turns to D. mentions to 66. A. ignoring B. blaming C. yielding to D. turning a deaf ear to 67. A. without B. with C. through D. for 68. A. rely on B. relax C. free D. find 69. A. wise B. clever C. slow D. enough 70. A. on B. with C. to D. about 71. A. however B. where C. what D. which 72. A. equally B. in the same way C. just as well D. not as well 73. A. be open B. have access C. look D. turn 74. A. approaches B. exposures C. problems D. means 75. A. adopt B. keep C. adapt D. devote 76. A. cost B. spent C. taken D. paid 77. A. Nor B. Not C. No D. Any 78. A. among B. amid C. amidst D. between 79. A. for B. against C. to buy D. to use 80. A. due to that B. in any case C. although D. where Part ? Cloze (共20小题,每题1分,共20分) (英语专业必做) Directions:There are 20 blanks in the following passage. For each blank there are four choices. Choose the one that best fits into the passage and then mark your answer on the answer sheet. No one should be forced to wear a uniform under any circumstance. Uniforms are 61 to the human spirit and totally 62 in a democratic society. Uniforms tell the world that the person who wears one has no value as 63 but only lives to function as a part of words. Those 64 uniforms lose all their self worth. There are those who say that wearing a uniform gives a person sense of 65 with a larger, more important concept. What could be more important than the individual himself? If an organization is so weak that it must 66 cloth and buttons to inspire its members, that organization has 67 reason to continue its existence. Others say that the 68 of making people wear uniforms, 69 in a school, removes all envy and competition in the matter of dress 70 that a poor person who cannot 71 good quality clothing need not be belittled(轻视) by a 72 person who wears expensive,quality clothing. Those in favor of wearing uniforms tend to ignore 73 critical concepts as freedom of choice, motivation, and individuality. Uniforms also 74 the economy. Right now, billions of dollars are spent 75 the fashion industry every year. Thousands of persons are 76 in designing,creating, and marketing different types of clothing. The wearing of uniforms would destroy the fashion industry, which 77 would have an unfavorable effect on such industries as advertising and marketing. 78 advertising, mass media such as newspapers, magazines and television would not be able to 79 in business. Our entire information and entertainment industries would 80 . 61. A. entertaining B. demanding C. charming D. penetrating 97 62. A. unnecessary B. illogical C. irresponsible D. independent 63. A. an employee B. a staff C. a person D. an individual 64. A. with B. under C. in D. on 65. A. possession B. identification C. involvement D. participation 66. A. rely on B. turn out C. take off D. cope with 67. A. entire B. ever C. none D. no 68. A. behavior B. conduct C. practice D. habit 69. A. tell B. speak C. put D. say 70. A. hence B. so C. however D. otherwise 71. A. provide B. afford C. undergo D. show 72. A. witty B. powerful C. wealthy D. competitive 73. A. such B. many C. so D. all 74. A. bruise B. harm C. blow D. defeat 75. A. with B. for C. in D. over 76. A. applied B. adapted C. adopted D. employed 77. A. after all B. by contrast C. on the contrary D. in turn 78. A. Without B. Unless C. Except D. Against 79. A. represent B. occupy C. stay D. preserve 80. A. destroy B. fail C. conflict D. hit 第?卷(共50分) Part ? Translation (共35分) Section A (共5小题,每小题4分,共20分) Directions: Translate the following sentences into Chinese. You may refer to the corresponding passages in Part ? . 81. The business of continuing the family lineage (血统)and keeping the blooding pure is often too important to be left to romance and chance encounters. (Passage One) 82. Researchers have found that school performance is little related to job performance. Qualities like ―steady and dependable‖ and ―practical and organized‖ are more important. (Passage Two) 83. The people we meet in books may delight us either because they resemble human friends whom we hold dear or because they represent unfamiliar types whom we are glad to welcome as new acquaintances. (Passage Three) (以下2题为非英语类学生必做) 84. Increasingly over the past ten years, people --- especially young people --- have become aware of the need to change their eating habits, because much of the food they eat, particularly processed foods, is not good for health. (Passage Four) 85. It is interesting to note that in countries where the national diet contains large quantities of unrefined flour and vegetables, certain diseases are comparatively rare. (Passage Four) (以下2题为英语类学生必做) 84. Reformers, however, fear raising the drinking age will have little effect unless accompanied by educational programs to help young people to develop "responsible attitude" about drinking and teach them to resist peer pressure to drink. (Passage Four) 85. As the fatalities continue to occur daily in every states, some Americans are even beginning well of the 13 years of national prohibition of alcohol that began in 1919, which President Hoover called the "noble experiment". (Passage Four) 98 Section B (共5小题,每小题3分,共15分) Directions: Translate the following sentences into English. 86. 互联网的出现改变了人们的生活、工作与思维方式。 87. 对一个想找工作的学生来说,有没有硕士学位的确有很大影响。 88. 我们往往认识不到的是,看似无用的东西也许从长远来看大有裨益。 89. 至关重要的是,发达国家和发展中国家应通过更多的交流增进相互间的理解和尊重 90. 要在学习上取得进步,学生需要具备独立学习的技能,而不是事事都依靠老师。 Part V Writing (共15分) Directions: In this part you rare required to write an essay about the ways of finding information. You should write at least 120 words and base your composition on the outline given below. 1. There are different ways of finding information. 2. Which way(s) do you prefer? And Why? 2009年普通高校专转本统一考试英语试卷参考答案 Part I Reading Comprehension Passage One 1(B。细节题。见文章第二段:…by introducing suitable marriage partners to each other。 2(D。细节题。第三段第一行:―marriage is regarded as more of a business contract than a sacred thing.‖ 3(B。细节理解题。关键在于理解文章第四段的内容。可用排除法去除其他3个干扰项。 4(A。整体理解题。正确选项所指在第一段。 5(A。主旨题。通读全文可以得知,在韩国,matchmaking在婚姻中的重要性。 Passage Two 6(D。细节理解题。通过第一段和第四段可以得出结论。注意C选项的干扰。如果fast tracker是 excellent student,那他(她)在工作中也一定是非常优秀的。 7(C。整体理解题。A和D容易排除,文中所给例子及评论,只能得出C结论。B也许正确,但文章并没有明确指出或暗示这一点,况且该文与rich无甚大关联。 8(B。词义理解题。利用构词法就可以猜出该词含义,再根据其所在句子,即可得出正确选项。 9(D。整体理解题。glue在文中只是个比喻,文章第三段很明确指出它的含义。 10(B。主旨题。首先排除A和C,比较另外两个选项,明显文章重点讲述的是成功的条件。 Passage Three 11(D。段落理解题。仅仅从该段第一句即可得出答案。注意A和D说法的顺序区别。 12(B。整体理解题。该题难在学生很容易把眼光仅仅放在acquaintances这个单词上,实际上这个单词就相当于friends。理解到这点,这道题就简单了。 13(B。细节题。文章第一段A―good book may absorb our attention so completely that for the time being we forget our surroundings and even our identity‖ 14(C。理解题。该道题难度属于容易C选项明显属于常识性错误。 15(A。推断题。该题中的句子在文章中还有一个限制成分:through books。因此答案也就不言自明了。 Passage Four 16(C。细节理解题。第一段―There has heen a growing interest in natural fonds: foods which do not contain chemical additives and which have not been affected by chemical fertilizers…‖ 17(C。细节理解题。从第二段第一句即可得出结论―Natural foodS, for example, include 99 vegetables, fruit and grain which have been grown in soil that is rich in organic matter.‖ 18(D。常识判断题。从文章第一段划线后半部分和第二段最后一句,都可得出结论。 19(D。细节题。第三段第一行―Natural foods also include animals which have been allowed to feed and move freely in healthy pastures.‖animals当然包括chickcn。 20(A。整体理解题。从文章最后一段可以得出结论。 Part ? Vocabulary and Structure 21(C。词组辨析,at hand义为在手头。 22(A。句子结构分析。两个逗号之间部分为修饰成分,对整个句子谓语时态无影响,该句子主语为第三人称单数。 23(B。词汇考察,induce含义为―导致,促使,引诱‖。 24(C。倒装句+现在完成时态。根据整个句子含义,该句使用完成时态足最准确和切当的。 25(D。词汇考察,construction义为―建筑‖,introduction义为―介绍‖,instruction义为―指示‖,说明restriction义为―限制‖。 26(B。同位语从句用法。用that来引导。 27(D。词组辨析,head for义为―动身、前往、寻找‖。 28(C。单词辨析,根据特征词into,很容易选出正确选项。 29(B。强调句,用that来引导。 30(A。短语辨析,in terms of义为―根据、按照‖。 31(D。单词辨析,controversial义为―有争议的,有争论的‖。 32(B。虚拟语气,should have done义为―该做某事而未做‖。 33(D。形容词辨析。注意:被修饰词是majorities。 34(A。词义辨析+搭配,assimilate义为―吸收,同化‖。 35(C。纯时态题。整个句子时态为一般过去时。故应在将来完成时+进行时的基础上再加上一般过去时。 36(D。词义辨析,compulsory为―义务的,必需的‖之义。 37(B。动词辨析+搭配,entitle为―给予……权利(资格)‖之义。 38(C。倍数表达法,倍数+as+形容词(副词)+as。 39(C。名词词义辨析,sequence为―顺序‖之义。 40(A。名词含义辨析+固定搭配,extent为―程度,范围‖之义。 41(D。句意理解+搭配,deprive为―剥夺、丧失‖之义。 42(A。非限制性定语从句。 43(C。名词辨析,conflict为―冲突‖之义。 44(B。短语搭配+用法辨析,be attributed to为―归因于‖之义。 45(B。特殊结构的虚拟语气,if only义为―要是……多好‖。 46(B。形容词辨析十固定搭配,suspicious为―可疑的,怀疑的‖之义。 47(A。特殊连词用法,含有转折意义。 48(B。固定句型。 49(D。形容词辨析,sensible为―明智的‖之义。 50(A。一般讲来完成时+被动语态。该题可参考35题。 51(A。固定句型,It has never occurred to…为―从没想到‖之义。 52(B。动词辨析,transfer之义为―调动,转换‖。 53(C。句型结构,平行结构的同级比较,谓语相同。 54(C。动词辨析,protect为―保护‖之义。 55(B。独立主格结构,主句的主语与作修饰成分的现在分词短语的逻辑主语不一致。 100 56(B。固定搭配,be confined to为―(被)限制,封闭在……‖之义。 57(D。句子结构分析+时态,现在完成时+it作形式宾语。 58(B。副词位置,句子谓语部分含be动词,修饰词(副词)位置可前可后。 59(D。虚拟语气,根据前一分句确定具体时态。 60(C。形容词辨析,concrete为―具体的‖之义。 Part? Cloze 61(A。名词用法辨析,item为―条款,项目‖之义,可用来指代它前面的事物。 62(D。固定搭配,personal computer为―个人电脑‖之义。 63(C。上下文+动词辨析,convince为―确信,相信‖之义。 64(B。虚拟语气,should被省略。 65(B。固定句型,When lt comes to…为―当提及,当说到‖之义。 66(C。句意分析+固定搭配,yield to为―屈服,让步‖之义。 67(A。上下文衔接,―匆忙购买了硬件(电脑)而没有准备好的教学计划‖。 68(D。句意分析,―教师们发现自己陷于……的两难境地‖。 69(C。上下文,between…and连接的是两个含义相反的短语。 70(A。固定搭配,后接on表示就某一问题达成一致。 71(D。定语从句。 72(C。短语用法+上下文,as well为―同样,也‖之义。 73(B。固定用法+句意,have access to为―得进入,得使用‖之义。 74(C。句意一致,可参考最后一段的第一句。 75(C。固定搭配,adapt to为―适应‖之义。 76(B。句意+固定搭配。 77(B。部分否定,not every为―并不是每一个‖之义。 78(D。单词用法,两者之间用between,即使那两者都为复数。 79(A。句意及短语辨析,ask for为―请求,要‖之义。 80(D。非限制性从句,后接地点。 (英语专业学生必做) 16(C。第一节第二句的数据说明美国因酒后驾驶导致的死亡人数急剧增加。 17(B。由第二节后半句可知。 18(D。由第三节后半句可知,因为降低限制年龄,年轻的肇事司机几乎翻了一倍,所以义调回了原 来的限制年龄。 19(A。由―to a marked decline in fatalities‖可知。 20(D。从文末作者的担忧来看,作者的态度很实际。 (英语专业必做) 61(B。entertaining为―有趣的,让人愉快的‖之义,demanding为―需很大耐性或技巧的,苛求的‖,charming 为―有魅力的‖之义,penetrating为―敏锐的,洞察的‖之义。 62( A。在民主社会,制服是根本―不需要的‖。 63(D。制服告诉人们作为一个―个人‖没有任何价值。 64(C。那些―穿着‖制服的人失去了他们所有的个人价值。 65(B。possession为―拥有‖之义,identification为―认同‖之义,involvement为―参与,投入,沉迷‖之 义,participation为―参与‖之义。 66(A。rely on为―依赖‖之义,turn out为―制造,结果是‖之义,take off为―脱下‖之义, cope with为―应付‖之义。 67(D。如果一个组织弱小到依赖布衣来激励它的成员,那么它就―没有‖理由再继续存在。 101 68(C。behavior为―行为‖之义,conduct为―组织,管理‖之义,practice为―实践,练习,惯例,常规‖之义,habit为―习惯‖之义。 69(D。say有―比如‖之义。 70(B。固定短语so that,表示结果。 71(B。provide为―提供‖之义,afford为―买得起‖之义,undergo为―经受‖之义,show为―表明,说明‖之义。 72(C。与poor形成对比的是―wealthy‖。 73(A。固定短语―such…as…‖,表示列举。 74(B。制服也会对经济―造成损害‖。 75(C。―在‖时尚行业,每年要花去几十个亿。 76(D。applied为―应用,致力于‖之义,adapt为―调整‖之义,adopt为―采用‖之义,employ为―雇佣‖之义。 77( D。after all为―毕竟‖之义,by contrast为―形成对比的是‖之义,on the contrary为―相反的是‖之义,in turn为―从而‖之义。 78(A。―没有‖广告的话,传媒就无法做生意。 79(C。呆在生意场中。 80(B。destroy为―破坏‖之义,fail为―失败‖之义,conflict为―矛盾‖之义,hit为―击打‖之义。 Part ? Translation Section A 81(传宗接代、维系家族血统正往往极为重要,不能由浪漫和偶遇支配。 82(研究人员发现,学习成绩与工作能力几乎没有关系。有些品质,比如―稳定而可靠‖、―实际而有条不紊‖则更加重要。 83(我们在书中遇到的人们之所以是我们感到快乐,原因就是他们或者很像我们珍爱的朋友,或者代表了新类型的人,而且是我们乐于结识的。 84(在过去的十年里,人们尤其是年轻人——越来越清楚地意识到有必要改变饮食习 惯,因为人们所吃的很多食物,特别是加工过的食物,不利于健康。 85(有趣的是,我们注意到,在那些国民饮食中含有大量未经精加工的面粉和蔬菜的国家,某些疾病相对而言很少见。(以上2题为非英语类学生必做) 84(然而,改革者们担心提高饮酒的合法年龄收效甚微,除非同时实施教育计划来帮助年轻人培养对饮酒持―负责的态度‖,并且教育他们抵制同龄人劝酒的压力。 85(由于每个州每天都持续发生死亡事故,有些美国人开始赞誉在1919年开始,实行了长达13年的全国禁酒令,即胡佛总统称之为―高尚的实验‖的禁酒令。(以上2题为英语类学生必做) Section B 86. The appearance of the Internet has changed the way people live, work and think. 87. For a student who wants to get a job,a master's degree/whether he has a master's degree does make a difference. 88. What we don't always recognize is that what seems to be useless may, in the long run, proves( to be) beneficial. 89. It is crucial that developed countries and developing countries promote mutual understanding and respect through more communication. 90. In order to get ahead in their studies,students need to equip themselves with independent study skills instead of relying on their teachers for everything. Part ? With so many new inventions in modern society, people can find information via many different 102 ways. People can make use of older ways like radio, television and telephone. Young people prefer newer and faster means such as cell phone and internet. Among these ways, internet is my favorite. Internet is very conveneient. On internet,people can find information of different kinds from all sources. I can switch from political news to a job-hunting page just by one click, which saves a lot of time.I can even talk to my parents and friends on the same computer. Some people may say that there is too much false information on the internet, which can bring bad influence to you. For me, this is not a problem. I always save some popular, reliable and respectable websites and make good use of them and care little about the attached advertisement. Small kids can also learn a lot of things from the internet if their parents tell them how to tell good from bad. 2010年江苏省普通高校“专转本”统一考试试卷 大学英语 第?卷(共100分) 注意事项: 1(答第?卷前,考生务必将自己的姓名、准考证号、考试科目用铅笔涂写在答题卡上。 2(每小题选出答案后,用2B铅笔把答题卡上对应题目的答案标号涂黑。如需改动,用橡皮擦干净后,再 选涂其他答案标号。如果答案不涂写在答题卡上,成绩无效。 Part ? Reading Comprehension(共20小题,每小题2分,共40分) Directions: There are 4 passages in this part. Each passage is followed by some questions or unfinished statements. For each of them there are four choices marked A, B, C andD. You should decide on the best choice and mark the corresponding letter on the Answer Sheet with a single line through the center. Passage l Questions l to 5 are based on the following passage. Sometimes you'll hear people say that you can't love others until you love yourself. Sometimes you'll hear people say that you can't expect someone else to love you until you love yourself. Either way, you've got to love yourself first and this can be tricky. Sure we all know that we're the apple of our parents' eyes, and that our Grandmas think we're great talents and our Uncle Roberts think that we will go to the Olympics, but sometimes it's a lot harder to think such nice thoughts about ourselves. If you find that believing in yourself is a challenge, it is time you built a positive self-image and learnt to love yourself. Self-image is your own mind's picture of yourself. This image includes the way you look, the way you act, the way you talk and the way you think. Interestingly, our self-images are often quite different from the images others hold about us. Unfortunately, most of these images arc more negative than they should be. Thus changing the way you think about yourself is the key to changing your self-image and your whole world. The best way to defeat a passive self-image is to step back and decide to stress your successes. That is, make a list if you need to, but write down all of the great things you do every day. Don't allow doubts to occur in it. It very well might be that you are experiencing a negative self-image because you can't move past one flaw or weakness that you see about yourself. Well, roll up your sleeves and make a change of it as your primary task. If you think you're silly because you aren't good at math, find a tutor. If you think you're weak because you can't run a mile, get to the track and practice. If you think you're dull because you don't wear the latest trends, buy a few new clothes. 103 The best way to get rid of a negative self-image is to realize that your image is far from objective and to actively convince yourself of your positive qualities. Changing the way you think and working on those you need to improve will go a long way towards promoting a positive self-image. When you can pat yourself on the back, you'll know you're well on your way. 1. You need to build a positive self-image when you ____ . A. dare to challenge yourself B. feel it hard to change yourself C. are unconfident ahout yourself D. have a high opinion of yourself 2. According to the passage, our self-images ____ . A. have positive effects B. are probably untrue C. are often changeable D. have different functions 3. How should you change your self-image according to the passage? A. Keep a different image of others. B. Make your life successful. C. Understand your own world. D. Change the way you think. 4. What is the passage mainly about? A. How to prepare for your success. B. How to face challenges in your life. C. How to build a positive self-image. D. How to develop your good qualities. 5. Who are the intended readers of the passage? A. Parents. B. Adolescents. C. Educators. D. People in general. Passage 2 Questions 6 to 10 are based on the following passage. Do you want to live with a strong sense of peacefulness, happiness, goodness, and self-respect? The collection of happiness actions broadly categorized as―honor‖help you create this life of good feelings. Here's an example to show how honorable actions create happiness. Say a store clerk fails to charge us for an item. If we keep silent, and profit from the clerk's mistake, we would drive home with a sense of sneaky excitement. Later we might tell our family or friends about our good fortune. On the other hand, if we tell the clerk about the uncharged item,the clerk would be grateful and thank us for our honesty. We would leave the store with a quiet sense of honor that we might never share with another soul. Then, what is it to do with our sense of happiness? In the first case, where we don't tell the clerk, a couple of things would happen. Deep down inside we would know ourselves as a type of thief. In the process, We would lose some peace of mind and self-respect. We would also demonstrate that we cannot be trusted. since we advertise our dishonor by telling our family and friends. We damage our own reputations by telling others.In contrast, bringing the error to the clerk's attention causes different things to happen.Immediately the clerk knows us to be honorable. Upon leaving the store, we feel honorable and our self-respect is increased. Whenever we take honorable actions we gain the deep internal rewards of goodness and a sense of nobility. There is a beautiful positive cycle that is created by living a life of honorable actions.Honorable thoughts lead to honorable actions. Honorable actions lead us to a happier existence.And It's easy to think and act honorably again when we're happy. While the positive cycle can be difficult to start, once 104 it's started, it's easy to continue. Keeping on doing good deeds brings us peace of mind, which is important for our happines. 6. According to the passage, the positive action in the example contributes to our ____. A. self-respect B. financial rewards C. advertising ability D. friendly relationship 7. The author thinks that keeping silent about the uncharged item is equal to ____. A. lying B. stealing C. cheating D. advertising 8. The phrase―bringing the error to the clerk's attention‖(Para.5)means ____. A. telling the truth to the clerk B. offering advice to the clerk C. asking the clerk to be more attentive D. reminding the clerk of the charged item 9. How will we feel if we let the clerk know the mistake? A. We'll be very excited. B. We'll feel unfortunate. C. We'll have a sense of honor. D. We'll feel sorry for the clerk. 10. Which of the following can be the best title of this passage? A. How to Live Truthfully B. Importance of Peacefulness C. Ways of Gaining Self-respect D. Happiness through Honorable Actions Passage 3 Questions 11 to 15 are based on the following passage. "Sesame Street‖(芝麻街) has been called ―the longest street in the world‖. That is because the television program by that name can now be seen in so many parts of the world. That program became one of America's exports soon after it went on the air in the New York in 1969. In the United States more than six million children watch the program regularly. The viewers include more than half of the nation's pre-school children. Although some educators object to certain clements in the program, parents praise it highly. Many teachers also consider it a great help, though some teachers find that problems arise when first graders who have learned from―Sesame Street‖are in the same class with children who have not watched the program. The program uses songs, stories, jokes and pictures to give children a basic understanding of numbers, letters and human relationships. Tests have shown that children have benefited from watching ―Sesame Street‖. Those who watch five times a week learn more than the occasional viewers. In the United States the program is shown at different times during the week in order to increase the number of children who can watch it regularly. Why has―Sesame Street‖been so much more successful than other children's shows? Many reasons have been suggested. People mention the educational theories of its creators, the support by the government and private businesses, and the skillful use of a variety of TV tricks. Perhaps an equally important reason is that mothers watch―Sesame Street‖along with their children. This is partly because famous adult stars often appear on―Sesame Street‖. But the best reason for the success of the program may be that it makes every child watching it feel able to learn. The child finds himself learning, 105 and he wants to learn more. 11. Why has―Sesame Street‖been called―the longest street in the world‖? A. The program has been shown ever since 1969. B. The program became one of America's major exports soon after it appeared on TV. C. The program is now being watched in most parts of the world. D. The program is made in the longest street in New York. 12. Some educators are critical of the program because ____. A. they don't think it fit for children in every respect B. it takes the children too much time to watch it C. it causes problems between children who have watched it and those who have not D. some parents attach too much importance to it 13. So many children in the United States watch the program because ____ . A. it uses songs, stories and jokes to give them basic knowledge B. it is arranged for most children to watch it regularly C. tests have shown that it is beneficial to them D. both A and B 14. Mothers often watch the program along with their children because ____. A. they enjoy the program as much as their children B. they find their children have benefited from watching the program C. they are attracted by some famous adult stars on the show D. they can learn some educational theories from the program 15. What is the most important reason for the success of the program according to the author? A. The creators have good educational theories in making the program. B. The young viewers find they can learn something from it. C. Famous adult stars often appear in the program. D. It gets support from the government and private businesses. Passage 4 Questions 16 to 20 are based on the following passage. You will have no difficulty in making contact with the Agent. As you enter his office, you will be greeted immediately and politely asked what you are looking for. The Estate Agent's Negotiator(房地产 推销员)—as he is called—will probably check that you really know your financial position. No harm in that, but you can always tell him that you have confirmed the position with the XYZ Building Society. He will accept that. He will show you the details of a whole range of properties; many of them are not really what you are looking for at all. That does not matter. Far better turn them down than risk missing the right one. The printed details he will give you are called―particulars‖. Over the years, a whole language has grown up, solely for use in Agent's particulars. It is flowery, ornate (华丽的) and,providing you read it carefully and discount the adjectives, it can be very accurate and helpful. Since the passing of the Trades Description Act, any trader trying to sell something has had to be very careful as to what they say about it. Estate Agents have, by now, become very competent at going as far as they dare. For instance, it is quite acceptable to say―delightfully situated‖.That is an expression of his opinion. You may not agree, but he might like the idea of living next to the gasworks. If, on the other hand, he says that the house has five bedrooms when, in fact, it has only two, that is a 106 misstatement of fact and is an offence. This has made Estate Agents, and others for that matter, rather more careful. Basically, all that you need to know about a house is: how many bedrooms it has; anindication of their size; whether the house has a garage; whether there is a garden and whether it is at the back or the front of the house; whether it is semi-detached or terraced (联排的). 16. The Estate Agent's Negotiator will ____. A. want all the details of your financial circumstances B. want to satisfy himself that you understand the financial implications of buying a house C. check your financial position with the XYZ Building Society D. accept any statement you make about your financial position 17. The author believes ____. A. it is better to be given information about too many properties than too few B. you should only look at details of properties of the kind you have decided to buy C. the agent will only show you the details of properties you have in mind D. it doesn't matter if you miss a few properties you may be interested in 18. The adjectives in Agent's particulars are ____. A. accurate B. helpful C. both accurate and helpful D. safe to ignore 19. The Trades Description Act applies to ____. A. house agents only B. most estate agents C. any trader D. buyers of houses 20. The phrase―going as far as they dare‖(Para.4) probably means ____. A. covering as wide an area as possible B. selling houses as far from the estate agent's houses as possible C. telling lies about properties if nobody is likely to find out about it D. trying every possible means to make the description of houses sound attractive Part ? Vocabulary and Structure (共40小题,每小题1分,共40分) Directions: There are 40 incomplete sentences in, this part. For each sentence there are four choices marked A, B, C and D. Choose the ONE answer that best completes the sentence. Then mark the corresponding letter on the Answer Sheet with a single line through the center. 21. Scientists will have to ____new methods of increasing the world's food supply in order to solve the problem of famine in some places. A. come up for B. come down with C. come down to D. come up with 22. I'd like to rent a house, modern, comfortable and ____in a quiet and safe neighborhood. A. all in all B. above all C. after all D. over all 23. In order to expose corruption, they have decided to set up a special team to ____the company's accounts. A. search for B. work out C. look into D. sum up 24. Some people would like to do shopping on Sundays since they expect to pick up a lot of wonderful ____in the markets. A. batterics B. baskets C. barrels D. bargains 25. The residents living in these apartments have free ____to the swimming pool, the gym and 107 other facilities. A. excess B. excursion C. access D. recreation 26. Reporters and photographers alike took great ____at the rude way the actor behaved during the interview. A. annoyance B. offence C. resentment D. irritation 27. Nothing healthful should be omitted from the meal of a ____child because of a dislike. A. proved B. supposed C. considered D. related 28. I'd like to ____this old car for a new model but I'm afraid I can not afford it. A. exchange B. convert C. replace D. substitute 29. She said she liked dancing but was not in the ____for it just then when it was so noisy in the hall. A. manner B. intention C. mood D. desire 30. He was ____admittance to the concert hall for not being properly dressed. A. rejected B. denied C. withheld D. deprived 31. Most people tend to ____a pounding heart and sweating palms with the experience of emotion. A. classify B. identify C. satisfy D. modify 32. The company ____many fine promises to the engineer in order to get him to work for them. A. held up B. held on C. held out D. held onto 33. ____the tragic news about their president, they have cancelled the 4th of July celebration. A. In the course of B. In spite of C. In the event of D. In the light of 34. The workers' demands are ____; they are asking for only a small increase in their wages. A. particular B. moderate C. intermediate D. numerous 35. Our explanation seemed only to have ____his confusion. He was totally at a loss as to what to say. A. brought up B. added to C. worked out D. directed at 36. With a standard bulb, only 5% of the electricity is ____to light—the rest is wasted away as heat. A. compressed B. conformed C. converted D. confined 37. Modern technology has placed ____every kind of music, from virtually every period in history and every corner of the globe. A. at our request B. at our disposal C. in our presence D. in our sight 38. Much has been written on the virtues of natural childbirth,but little research has been done to ____these virtues. A. confirm B. consult C. confess D. convey 39. Plastic bags are useful for holding many kinds of food ____their cleanness, toughness and low cost. A. by virtue of B. at sight of C. by means of D. by way of 40. I'm trying to think of his name, but my mind goes completely ____. I must be slipping. A. bare B. blank C. hollow D. vacant 41. ____in the regulations that you should not tell other people the password of your e-mail account. 108 A. What is required B. What requires C. It is required D. It requires 42. It is generally believed that gardening is ____it is a science. A. an art much as B. much an art as C. as an art much as D. as much an art as 43. The indoor swimming pool seems to be a great deal more luxurious than ____. A. is necessary B. being necessary C. to be necessary D. it is necessary 44. He ____English for eight years by the time he graduates from the university next year. A. will learn B. will be learning C. will have learnt D. will have been learnt 45. It was not until the subprime loan crisis(次贷危机) ____great damage to the American financial system that Americans ____the severity of the situation. A. caused; realized B. had caused; realized C. caused; had realized D. was causing; had realized 46. —The police think your brother John stole the diamond in the museum yesterday evening. —Oh? But he stayed with me at home the whole evening; he ____the museum. A. must have been to B. needn't have been to C. should have been to D. couldn't have been to 47. ____the meeting himself gave his supporters a great deal of encouragement. A. The president will attend B. The president to attend C. The president attended D. The president's attending 48. Everything ____into consideration, the candidates ought to have another chance. A. is taken B. taken C. to be taken D. taking 49. ____from heart trouble for years, Professor White has to take some medicine with him wherever he goes. A. Suffered B. Suffering C. Having suffered D. Being suffered 50. The concert will be broadcast live to a worldwide television audience ____at one thousand million. A. estimating B. estimated C. estimates D. having estimated 51. About half of the students expected there ____more reviewing classes before the final exams. A. is B. being C. to be D. have been 52. ____made the school proud was more than 90% of the students had been admitted to key universities. A. What; because B. What; that C. That; what D. That; because 53. Information has been released ____more middle school graduates will be admitted into universities this year. A. while B. that C. when D. as 54. He is the only one of the students who ____a winner of scholarship for three years. A. is B. are C. have been D. has been 55. —What's that newly-built building? — ____the students have out-of-class activities, such as drawing and singing. 109 A. It is the building that B. That's where C. It is in which D. The building that 56. With a large amount of work ____, the chief manager couldn't spare time for a holiday. A. remained to be done B. remaining to do C. remained to do D. remaining to be done 57. Why! I have nothing to confess. ____you want me to say? A. What is it that B. What it is that C. How is it that D. How it is that 58. —David has made great progress recently. — ____, and____. A. So he has; so you have B. So he has; so have you C. So has he; so have you D. So has he; so you have 59. It is universally known that microscopes make small things appear larger than ____. A. really are B. are really C. are they really D. they really are 60. —Tom, ____, but your TV is going too loud. —Oh,I'm sorry, I'll turn it down right now. A. I'd like to talk with you B. I'm really tired of this C. I hate to say this D. I need your help Part ? Cloze (共20小题,每小题1分,共20分) Directions: There are 20 blanks in the following passage. For each blank there are four choices marked A, B, C and D. You should choose the ONE that best fits into the passage. Then mark the corresponding letter on, the Answer Sheet with a single line through the center. The eyes are the most important 61 of human body that is used to 62 information. Eye contact is crucial for establishing rapport(融洽关系) 63 others. The way we look at other people can 64 them know we are paying attention to 65 they are saying. We can also look at a person and give the 66 we are not hearing a word. Probably all of us have been 67 of looking directly at someone and 68 hearing a word while he or she was talking 69 we were thinking about something totally 70 to what was being said. Eye contact allows you to 71 up visual clues about the other person; 72 , the other person can pick up clues about you. Studies of the use of eye contact 73 communication indicate that we seek eye contact with others 74 we want to communicate with them, when we like them, when we are 75 toward them (as when two angry people 76 at each other) , and when we want feedback from them. 77 , we avoid eye contact when we want to 78 communication, when we dislike them, when we are 79 to deceive them, and when we are 80 in what they have to say. 61. A. unit B. part C. link D. section 62. A. transfer B. translate C. transmit D. transport 63. A. against B. with C. for D. to 64. A. forbid B. allow C. permit D. let 65. A. how B. which C. what D. that 66. A. impression B. expression C. suggestion D. attention 67. A. ignorant B. careless C. guilty D. innocent 110 68. A. nor B. so C. not D. neither 69. A. or B. unless C. why D. because 70. A. related B. relevant C. unrelated D. indifferent 71. A. tear B. pick C. size D. take 72. A. likewise B. moreover C. otherwise D. therefore 73. A. in B. about C. with D. of 74. A. why B. where C. when D. what 75. A. friendly B. hostile C. respectful D. mistrustful 76. A. glance B. glare C. gaze D. stare 77. A. Exactly B. Generally C. Conversely D. Interestingly 78. A. hold B. establish C. avoid D. direct 79. A. wanting B. tending C. forcing D. trying 80. A. uninformed B. unconcerned C. uninterested D. unheard 第 ? 卷(共50分) Part ? Translation (共35分) Section A(共5小题,每小题4分,共20分) Directions: Translate the following sentences into Chinese. You may refer to the corresponding passages in Part ?. 81. The best way to get rid of a negative self-image is to realize that your image is far from objective and to actively convince yourself of your positive qualities. (Passage 1) 82. There is a beautiful positive cycle that is created by living a life of honorable actions. Honorable thoughts lead to honorable actions. Honorable actions lead us to a happier existence. (Passage 2) 83. While the positive cycle can be difficult to start, once it's started, it's easy to continue. Keeping on doing good deeds brings us peace of mind, which is important for our happiness. (Passage 2) 84. That is because the television program by that name can now be seen in so many parts of the world. That program became one of America's exports soon after it went on the air in New York in 1969. (Passage 3) 85. For instance, it is quite acceptable to say―delightfully situated‖. That is an expression of his opinion. You may not agree, but he might like the idea of living next to the gasworks. (Passage 4) Section B (共5小题,每小题3分,共15分) Directions: Translate the following sentences into English. 86(一个公司应跟上市场的发展变化,这是很重要的。 87(相比较之下,西方的家长更有可能把孩子的成功归因于天赋。 88(一般来说,你准备得越充分,考试前就越不会紧张。 89(要不是因为你及时地帮助,我真不知道该如何完成这项任务。 111 90(在任何情况下,我们都不应该放弃对生活的希望。 Part ? Writing (共15分) Directions: For this part,you are allowed 30 minutes to write a speech in the name of Li Ming,the chairman of the Student Union, in order to introduce the study strategies at college to all the freshmen, in your university. You should write at least 120 words following the outline given below in Chinese. 1(问候听众、介绍自己、说明演讲的话题;(请勿使用真实姓名,否则答题无效) 2(介绍大学学习策略(如学习习惯、时间安排、自主与合作学习等); 3(表达祝愿。 Study Strategies at College 2010年江苏省普通高校专转本统一考试英语试卷答案与解析 Part ? Reading Comprehension Passage 1 【文章大意】本文讲的是如何建立一个积极的自我评价/形象和自爱的问题。有时你不可能去爱别人, 只有你先爱惜自己,而且只有你自己先爱惜自己,才能去爱别人。每个人都是长辈眼中的佼佼者,都被他 们寄予厚望,但是有时自己很难这么评价自己。如果你发觉信任自己有些困难,那么是该建立一个积极的 自我评价和自爱的时候了。自我评价是你自己怎么看待自己,包括你看的方式、行动方式、走路的姿态以 及思考的方式。令人感兴趣的是通常自我形象和别人对自己的评价不一样。自己对自己的评价往往更消极。 改变自我评价和你的内心世界的关键是改变你认识自己的方式。最好的消除消极自我评价的方法是列出每 天要做的所有事情,不要对它们产生怀疑。产生消极的自我评价很可能是因为你走不出过去的弱点。为此 我们可以有针对性地去弥补。比如数学较差我们可以请家教。不善于长跑,我们可以去操场多加锻炼。最 好的摆脱消极的自我评价的方法是要意识到你的自我评价远远不够客观并积极的说服自己来肯定自己的优 点。改变自己的想法并努力去提高那些你不大擅长的方面,有助于建立积极的自我评价。 1(【答案】 C 【解析】 文章第一段的最后一句话:If you find that believing in yourself is a challenge,it is time you built a positive self-image and learnt to love yourself(当你发觉你在信任自己这一点上有困难时,是该建立 积极的自我形象/评价以及学会爱自己的时候了)。challenge名词,挑战,怀疑,质问。A项敢于改变挑战 自己;B项感觉改变自己有困难;C项对自己没信心;D项对自己评价很高。结合题意故只有C项符合文 中的意思。 2(【答案】 B 【解析】 文章第二段话的第三、四句:Interestingly,our self-images are often quite different from the images others hold about us. Unfortunately, most of these images are more negative than they should be(令人感兴趣的是通常自我形象和别人对自己的评价不一样。自己对自己的评价往往更消极)。 故选择B。A项片面了,没有包括消极的作用。C项说很容易改变,和文章第五段相悖,实际上做起来还 是很麻烦的。 3(【答案】 D 【解析】 文章第二段的最后一句话:Thus changing the way you think about yourself is the key to changing your self-image and your whole world(改变自我评价和你的内心世界的关键是改变你思考的方 式)。A、B、C项文中均未涉及。故选择D。 4(【答案】 C 【解析】 这篇文章是讲述如何建立一个积极的自我评价/形象和自爱的问题。A、B、D项均偏离文章 的主题。故选择C。 112 5(【答案】 B 【解析】 文章的第一段提到父母爷爷奶奶以及叔叔对你的期望,结合文中倒数第二段提出请家教补习数学的事情,可判断作者所针对的读者是青少年。 Passage 2 【文章大意】本文介绍了如何通过高尚的行动获得快乐。所有的快乐行为都有高尚的特性。这些行为有助你快乐地生活。作者给出了如何通过值得高尚的行为来获得快乐,那就是当商店收银员忘记收取一件商品的钱时,你怎么做:是默不作声并且为之欣喜,还是告诉收银员,两种不同的行为带了不同的影响。前者让你得到小便宜,或许你会告诉你的亲人朋友,但是这样会让他们认为你的品质不好,而且这种行为等同于偷窃;相反当你告诉收银员她的失误时,她会很高兴也会很感谢你。这种值得高尚的行为也给你带来轻松快乐的心情。生活中充满高尚的行为会产生一种美好的积极的循环。高尚的想法产生高尚的行为;而高尚的行为又给我们带来愉快的生活。然而这种积极的循环开始很难,一旦开始就很容易持续。坚持做好事给我们的思想带来安宁,这对我们的快乐十分重要。 6(【答案】 A 【解析】 语见文章的第一段尾句The collection of happiness actions broadly categorized as―honor‖ help you create this life of good feelings(所有的快乐行为都有高尚的特性,这些行为有助你快乐地生活)和第二段Here's an example to show how honorable actions create happiness(例子说明了高尚的行为带来快乐)。B项经济的奖励;C项广告的能力;D项有好的关系,与文章要表达的主题不符合。 7(【答案】 B 【解析】 文章第五段第二句Deep down inside we would know ourselves as a type of thief,内心深处,我们知道这是一种偷窃的行为。故排除A、C、D三项,选B。 8(【答案】 A 【解析】 语见文章第五段第七句In contrast,bringing the error to the clerk's attention causes different things to happen和第八句Immediately the clerk knows us to be honorable。其中the error指代收银员没有收钱的商品,attention注意力,也就是说告诉收银员这件商品没有收钱。所以选择A。B项给收银员提建议;C项让收银员注意力更加集中;D项提醒收银员收过钱的商品;这些都不算是高尚的行为。 9(【答案】 C 【解析】 当让收银员知道有未收钱的商品时我们会是什么感受,文中第五段倒数第二句话可以找 到答案。Upon leaving the store,we feel honorable and our self-respect is increased我们一离开商店,我们就感到高尚和自尊在增长。故选择C。 10(【答案】 D 【解析】 文章的标题的确定,需要综合全文来考虑。综观全文,可知本文是介绍如何通过高尚的行动获得快乐。所有的快乐行为都有高尚的特性。这些行为有助你快乐地生活。作者给出了如何通过高尚的行为来获得快乐。A项如何真实的活着;B项宁静的重要性;C项获得自尊的途径;D项快乐来自于高尚行为。故选择D。 Passage3 【文章大意】文章介绍了―芝麻街‖被叫做世界上最长的街的原因。之所以这么说是因为―芝麻街‖这一电视节目现在在全世界都被关注。自从它于1969年在纽约开播以来就成为美国电视节目的焦点。在美国有超过六百万的孩子定期观看此节目,其中包括一半以上学龄前儿童。尽管很多教育专家反对这一电视节目里面的内容但是父母们对它评价很高。许多老师认为它对学生有好处。然而一部分老师发现看了节目和不看节目的学生在同一班级学习时会产生一些问题。这一节目用音乐、故事、笑话等给孩子们普及一些关于数字、字母和人际关系的基本知识。那些经常看此节目的人要比偶尔看的人学习的知识要多。除此之外,这一节目还采用在一周不同时间播放的方式来增加定期观看的孩子。最后作者阐述了―芝麻街‖如此受欢迎的原因。人们给出了很多种解释,如教育理论、政府和私人企业的支持、多样化的提高收视率的技巧等。让母亲们看此节目的重要原因是经常有著名的成人偶像出现在此节目中;但它成功的最主要的原因可能是113 它使得每一个看节目的孩子感觉到他们可以学会些东西。 11(【答案】 C 【解析】 ―芝麻街‖之所以被称为―世界上最长的街‖,语见文章第一段第二句话That is because the television program by that name can now be seen in so many parts of the world,之所以这么说是因为―芝麻街‖这一电视节目现在在全世界都被关注。 12(【答案】 C 【解析】 参考文章第二段的第三句话Although some educators object to certain elements in the program, parents praise it highly尽管很多教育专家反对这一电视节目里面的某些内容,但是父母们对它评价很高和第四句Many teachers also consider it a great help, though some teachers find that problems arise when first graders who have learned from―Sesame Street‖are in the same class with children who have not watched the program许多老师认为它对学生有好处。然而一部分老师发现看了节目和不看节目的学生在同一班级学习时会产生一些问题。certain elements in the program某些节目内容,一些教育者认为节目的每一方面都不适合孩子排除A项;B项文章没涉及;D项不是教育者反对的原因,因为紧接着后面就提到了在教学中产生的问题。 13(【答案】 D 【解析】 答案可以在文章的第三段找到。有如此多的孩子观看此节目的原因:其一首先是节目的内容让孩子受益颇多;其次是该节目播放的时间是一周里的不同时间段。这也有利于吸引更多的儿童观众。 14(【答案】 C 【解析】 母亲陪孩子看此节目的原因可以从文中第四段倒数第三句和第四句可找到答案。 Perhaps an equally important reason is that mothers watch―Sesame Street‖along with their children. This is partly because famous adult stars often appear on―Sesame Street‖,让母亲们看此节目的重要原因是经常有著名的成人偶像出现在此节目里,和C项表述一致。 15(【答案】 B 【解析】 ―芝麻街‖成功的原因文章列举了很多,但是最重要的原因是在文章中的第五段倒数第 二句话:But the best reason for the success of the program may be that it makes every child watching it feel able to learn但它成功的最主要的原因可能是它使得每一个看节目的孩子感觉到他们可以学会些东西。 A、C、D项均可通过第五段来排除。 Passage 4 【文章大意】本文写的是关于房地产商推销房子的情况。当你进入房地产销售部时,你和房地产推销员的沟通完全没问题。因为你一踏进门,他们就会立刻很有礼貌地问你想寻找什么样的房子。他们很可能是想判断你的经济地位。他们将很热情的给你介绍所有特征的细节,其中很多是你不关心的,但这无关紧要,拒绝它们远比错过一个合适的要好。所有华丽的词都被他们给用来介绍房子。你所要做的就是弄明白你想知道的东西。 16(【答案】 D 【解析】 文中第一段的第二句话:The Estate Agent's Negotiator(房地产推销员)—as he is called—will probably check that you really know your financial position他们很可能是想判断你的经济地位。A项想要你所有的详细的经济情况;B项让你明白你的经济状况可以买房而以此满足他们的期望;C项检查你和某建筑公司的经济地位;都不符合题意,因为房地产推销员之所以这么做是想根据你的经济情况来向你推销适当价位的房子。 17(【答案】 A 【解析】 根据文章的第二段的最后一句话即可找到此题的答案。Far better turn them down than risk missing the right one((拒绝它们远比错过一个合适的要好。)B你应该仅仅看你决定买的房子 的特征。C项和He will show you the details of a whole range of properties;many of them are not really 114 what you are looking for at all相矛盾。D项文章未提及。 18(【答案】 D 【解析】 参见文中第三段后面两句:Over the years,a whole language has grown up,solely for use in Agent's particulars. It is flowery,ornate(华丽的)and,providing you read it carefully and discount the adjectives,it can be very accurate and helpful房地产经销商的语言很华丽,当你在理解这些文字时要细心,你要对这些形容词打折,只有这样才能更精确更有帮助。所以D项正确。 19(【答案】 C 【解析】 参见文中第四段的第一句Since the passing of the Trades Description Act,any trader trying to sell something has had to be very careful as to what they say about it商业用词法的颁布后,所有商人都得尽力做到对商品的描述实事求是。 20(【答案】 C 【解析】 第四段说Estate Agents have,by now,become very competent at going as far as they dare房地产经销商很擅长夸大其词,而且是胆大包天,再结合后面的he says that the house has five bedrooms when, in fact, it has only two, that is a misstatement of fact and is an offence他说有5个卧室,实际上你一去考查只有两个,这种言论让人很气愤,可以判断本题选C。 Part ? Vocabulary and Structure 21(【答案】 D 【解析】come up for air喘口气,调节一下精神;come down with付钱,染上(病);come down to屈尊做某事,归结起来(为……);come up with追赶上,想出,提出。提出想法/建议come up with a plan / method( 22(【答案】 B 【解析】 ―我想租一套现代的、安静的房子,最重要的是周边环境要安静的,邻居友善。‖A项all in all总的来说;大体而言;B项above all首要的是;尤其;最重要的是,符合题意;C项after all毕竟;究竟;归根结底;D项over all遍及。 23(【答案】 C 【解析】 search for搜索,搜寻;work out解决,解答;计算出;look into调查;研究;sum up总结,概括;是……的总和(数);合计。―为了揭露腐败,他们决定设立特别小组来调查该公司的财务‖。故选C。 24(【答案】 D 【解析】 batteries电池;baskets篮子;barrels桶;bargains廉价货。 25(【答案】 C 【解析】 固定结构,have access to有机会/权利做某事。excess超重的,过量的,额外的,超额的,附加的,过度的;excursion远足,短途旅行;access入门;接近(取得)……的方法(权利等);接近(或进入,享用)的机会;接近(或进入,享用)权;recreation娱乐(方式);消遣(方式)。 26(【答案】 A 【解析】 ―记者和摄影师好像对演员在采访过程中的粗鲁表示恼怒。‖对……感到恼怒have / take annoyance at...。A项annoyance vt. 恼怒;烦恼;打扰;n(使人烦恼的事;讨厌的东西(或人)。 例:I felt annoyance at being teased 我恼恨别人取笑我。B项offence犯规;违法行为;罪行;冒犯;无礼。 例: She felt she had committed an offence against the right of others她觉得自己侵犯了他人的权利。 C项resentment(因受虐待而)愤恨,不满,怨恨;愤恨;D项irritation激怒;恼怒;生气;令人恼火的 事。例:I felt a sudden irritation against Darren我突然对达伦感到气恼。 27(【答案】 B 【解析】 ―任何健康的食物都不应该因为假设的不喜欢而从孩子的饮食中剔除。‖B项suppose假定的,符合题意,故选B。 115 28(【答案】 D 【解析】―我想用一辆新车来换掉这辆旧车,可是恐怕我负担不起。‖由后面的半句排除A项交换和B项转换。C项 replace A with B,用A代替/替换B;D项substitute A for B,用A替换B,结合题意故选择D。 29(【答案】 C 【解析】 ―她说她喜欢跳舞,但是大厅太吵了,那会使她没有跳舞的心情。‖be in the mood for (doing) sth.,没有心情做某事;have desire for sth(想做某事。 30(【答案】 C 【解析】 ―他因衣着不当而不让进音乐厅。‖deprive sb. of(doing)sth. ; deny sb. sth. ; reject sb. to do sth. 拒绝某人做某事。结合题意选C。 31(【答案】 B 【解析】 ―许多人把心跳加速和手掌出汗等同于紧张。‖identify使等同于;把……看做一致(常与with连用)。 32(【答案】 C 【解析】 ―为了留住这位优秀的工程师,公司给他提出了许多很好的承诺。‖从意思上来判断只有C项正确。hold up举起,抬起,拿起;支持,支撑,承受住,支持住,承受住,支撑得住;hold on抓紧不放;(在困境或危险中)坚持住,挺住;hold out伸出,张开;拿出,提出,提供;hold onto紧紧抓住;不卖出;坚持住。 33(【答案】 D 【解析】 ―鉴于有关总统的坏消息,他们取消了7月4号的庆祝。‖A项In the course of在……期 间,在……过程中;B项In spite of不管;虽然;尽管;C项In the event of万一,倘若;如果发生; D项In the light of鉴于,由于,按照。故选择D。 34(【答案】 B 【解析】 由后面的一句―他们仅仅要求工资的小幅度增长‖,故可推断出正确答案为B选项,―工人们的要求很适度。‖particular adj.特定的,某一的;特殊的,特别的,特有的;moderate adj.中等的,适度的,不很大的;温和的;有节制的;中庸的,稳健的,不走极端的;intermediate形容词adj.中间的;中级的;中级的;中等的;适合中等程度者的;numerous形容词adj.很多的,许多的。 35(【答案】 B 【解析】 ―结果我们的解释似乎增加了他的迷惑,他对我所说的完全不理解。‖brought up教 育,养育(孩子);提及……;added to增加,加强;worked out解决,解答;计算出;产生结果; 发展;directed at旨在于;瞄准……目标。 36(【答案】 C 【解析】 电转换成光,用convert into / to。 37(【答案】 B 【解析】 ―现代技术让我们尽情享受音乐……‖B项at our disposal处置……的自由;有处理…… 的权利,符合题意,故选A。at our request应我们的请求;in our presence在我们面前;in our sight在 我们眼里/视线之内。 38(【答案】 A 【解析】 ―自然生产的好处报道了很多,但是关于这些优点很少有人去研究并证明。‖A项 confirm vi. 遵守,符合,遵从,服从;顺应,一致,顺从,随潮流;B项consult vt.,vi(商议,商量; 请教,咨询;找(医生)诊治翻阅,查阅;顾及,考虑;C项confess vt. ,vi. 承认,供认(错误或罪 行);D项convey vt. 运输;运送;输送;表达,转达,传递。结合题意,A项符合,故选A。 39(【答案】 A 【解析】 ―鉴于塑料袋具有干净、结实、廉价的特性,用来包裹食物很有好处。‖A项by 116 virtue of凭借……的力量;由于[因为];B项at sight of副词adv(一看见;C项by means of用,依靠;D 项by way of路经,途经;意在,为了。结合题意,A项符合,故选A。 40(【答案】 B 【解析】 固定用法,大脑一片空白mind is/goes blank。blank adj.空白的,空着的;茫然的,无表情的;A项bare adj(光秃秃的,无遮蔽的;赤裸的;(房间、柜子等)空的;C项hollow adj.空的;空洞的;D项vacant adj.未被占用的,空的;(指职位)空缺的。 41(【答案】 C 【解析】 ―规定里要求你的邮箱账号不可告诉其他人。‖句型it is required that…,要求…… 42(【答案】 D 【解析】 ―总体来说,园艺和科学一样,都是一门艺术。‖as much…as,同……一样。 43(【答案】 C 【解析】 ―室内游泳池看起来似乎是奢侈有余,没有存在必要。‖a great deal做状语,修饰 luxurious,后面所填的词是和luxurious并列的词。more…than比较级结构。 44(【答案】 D 【解析】 ―截止到明年他大学毕业,他学习英语八年了。‖By , 一段时间,主句的谓语部分需要用完成时态,此处为将来时间,故用将来完成时。 45(【答案】 B 【解析】 考核时态的掌握。次贷危机发生在先,人们意识到金融体制在后,而且句子的时态是一般过去时故选择B。 46(【答案】 D 【解析】 回答的前句说―他整个晚上都和我待在家里‖,因此他不可能在博物馆。Couldn't have done不可能做了某事,或表示对过去情况的虚拟,不能做某事而做了;needn't have done没有必要做某事;must have done是对过去情况进行较为肯定的推测;should have done本应该做某事,而实际上没有做,表示虚拟。 47(【答案】 D 【解析】 句子缺少主语,故A、C项排除;后面的时态是一般过去式,故B不对,to do表示将要发生的事。 48(【答案】 B 【解析】 take sth(into consideration考虑某事,宾语everything提前,应该是everything is taken into consideration,但是整个句子缺少连词,逗号不可能来连接两个句子,故A项排除,只可以选择 B。过去分词表示被动和完成。独立主格结构做状语。 49(【答案】 B 【解析】 ―由于多年患有心脏病,怀特教授随身带着药。‖动词ing在句子中做状语,可表示原因,结果,方式,目的状语,在此处是表示原因状语。 50(【答案】 B 【解析】 首先音乐会还没被直播,故其观众的估计也不可能发生了,所以排除D项。句子不可缺少谓语,故C项排除。观众人数的估计是被动,故只可选择B。estimate vt.,vi(,n(估计;评价,评估。 51(【答案】 B 【解析】 ―大约一半的学生希望考前有复习课。‖句子的谓语是expected故C项排除;而且后面 是复数classes,故A项排除。D项的时态和句子的时态一般过去时的时间先后上有冲突,故D项排除。排除法,故选择B。 52(【答案】 B 【解析】 主语从句和表语从句。―让我们学校感到骄傲的是我们学校超过90%的学生都被重点大学录取。‖主语从句缺乏宾语成分,故不选择that,而用what;表语从句是完整的陈述句时用that引导。故选B。 117 53(【答案】 B 【解析】 同位语从句。有些词的同位语从句必须用that引导,而且that不可省略,如:belief, thought, idea, fact, news, information等等。 54(【答案】 D 【解析】 首先the only one of the students做主语,句子的谓语用单数;only one of the students做主语,句子的谓语用复数;故排除B、C项;其次是for , 一段时间,句子是时态是完成时A项排除。故选择D。 55(【答案】 B 【解析】 ―这栋楼是学生进行课外活动的楼。‖where相当于in which。A项不对,后面的是地点状语从句,不用that引导。C项缺少先行词。 56(【答案】 D 【解析】 with复合结构。remain留待,尚待,不用于被动语态。remain to do…尚待去做。故选择D。 57(【答案】 B 【解析】 强调句型It is/was , 被强调的部分 , that/who , 句子的其他成分。该题用强调句型 强调say的宾语what。 58(【答案】 B 【解析】 当说话人说某种情况,某人和前者所说的情况一样时,肯定句中,我们用so it is with…或 者so , 助动词 , 主语;否定句中我们用neither/nor , 助动词 , 主语。当说话人肯定前者所说的内容时,即―的确如此‖,我们用so , 主语 , 助动词。从句子意思来判断,选择B。 59(【答案】 D 【解析】 ―众所周知,显微镜使得小物体看起来比它们实际情况要大得多。‖ 60(【答案】 C 【解析】 日常交际用语。―汤姆,虽然我很不想这么说,但是你把电视机的声音开得太大了。‖ Part ? Cloze 【文章大意】这篇文章讲述了眼睛交流在交流中的重要性和眼睛所能表达出的各种信息。眼睛可以流露出乐意和人建立融洽关系的信息,也可以传达拒绝交流的信息。前者通过眼神关注别人所讲的东西;后者是通过眼神流露出来对别人所讲事情毫不关心。当你想交流和不想交流,所表现出来的眼神会完全不一样。 61(【答案】 B 【解析】 unit单位;part部分;link连接;section章节,科,部门。眼睛是身体的一部分,故选择B。 62(【答案】 C 【解析】 transfer转移;迁移;translate翻译;传达;transmit(用无线电波等)传递,发射,播送,广播;transpon(用交通工具)运输,运送。此处说眼睛被用来传递信息,故选C。 63(【答案】 B 【解析】 和……建立关系,固定结构,establish relationship with sb(。 64(【答案】 D 【解析】 A项forbid禁止,妨碍,阻碍,阻止,forbid sb(to do sth. ;B项allow允许,许可(做 某事),allow sh. to do sth. ;C项permit允许,准许;允许,permit sb. to do sth. ;D项let让, let sb. do sth. 65(【答案】 C 【解析】 句子没有先行词此处不可选择which和that。what做say的宾语。故选C。 66(【答案】 A 【解析】 impression印象,外观,感想;expression表达,表情,表现,感情;suggestion建议,意见;暗示;联想;attention注意,专心,留心,殷勤,特别照料(或行动、处理)。句意为―我们同样可以盯着 118 一个人看,同时给人的印象却是一个字也没听。‖故选A。 67(【答案】 A 【解析】 很可能当别人在讲话时,因为我们在想些和他所说的完全无关的东西,我们是以一种很傻的表情盯着某人看。ignorant无知的,愚昧的;很无礼的,十分不懂规矩的;careless大意的;guilty形容词内疚的;有罪的;innocent清白的,无罪的,无辜的;无害的,没有恶意的;涉世不深的;天真的,单纯的。故选择A。 68(【答案】 C 【解析】 注意and连接的前后文的并列关系。 69(【答案】 D 【解析】 因为我们在想着别的与此毫不相关的东西,所以会有很呆的表情。故选D。 70(【答案】 C 【解析】 related与……有亲戚关系的;有关系的;relevant,有关的,切题的;有价值的;有意义的;确切的;中肯的;适当的;unrelated无关的,不相关的;无关联的;无亲戚关系的;indifferent不关心的,不感兴趣的,冷淡的;中立的;较差的,平庸的,不好的,一般的。与……不相关, be unrelated to sth(结合语境,应选C。 71(【答案】 B 【解析】 ―眼神的接触让你捕捉到了解一个人的视觉线索。‖A项tear up撕成碎片;挖开(地面 等),揭开,掀起;B项pick up拾起,捡起;抬起;理解;懂得;意识到;注意到;使搭乘;搭救; C项size up估量,估计;迅速对……作出判断评价;D项take up拿起;抱起占去;继续。故选择B。 72(【答案】 A 【解析】 ―同样,别人也能从你的眼神中了解你‖,表示和前面的情况相同。故选择A。likewise同样地,照样地;moreover再者,此外;otherwise用别的方法,不同地;要不然,否则;therefore因此,为此,所以。 73(【答案】 A 【解析】 在交流中in communication。 74(【答案】 C 【解析】 when表示―当……时候‖,和后面的句子结构保持一致。 75(【答案】 A 【解析】 在想交流的情况下,B项hostile怀有敌意的,不友善的和D项mistrustful不信任的,不合常理,故排除;C项respectful有礼貌的;恭敬的,也不能选择,因为后面的76题空所要表达的意思:就像当两个生气的人彼此看着对方一样,限定了此处只可选择A,友好的。 76.【答案】 B 【解析】 ―就像当两个生气的人彼此看着对方一样‖。glance at快速朝……看了一眼;glare at用愤怒的目光注视;gaze at仔细地看,打量;stare at盯,凝视(因吃惊,惊奇或喜欢而目瞪口呆)。结合题意故选B。 77(【答案】 C 【解析】 因为后面讲述的是―当你避免和人有眼神的交流时是在哪些情况下,比如:当我们不喜欢他们时;当我们力图欺骗他们时;当我们对他们所说的东西不感兴趣时。‖和前面的情况正好相反,故选择C项Conversely相反的,颠倒的,反过来。Exactly精确的,准确的;Generally总体的,大体的; Interestingly有趣的。 78(【答案】 C 【解析】 ―当我们避免交流时我们也会避免和人有眼神的交流‖,前面想和某人交流时是我们乐意和别人有眼神的交流,此处不想有眼神的交流故是想避免交流,avoid communication。B项establish建立,意思反了。A、D项不存在。 119 79(【答案】 D 【解析】 ―当我们力图欺骗别人时,我们不愿意眼神接触。‖want to do sth(想做某事;tend to do sth. 易于做某事,有做……的倾向;force to do sth(强迫做某事;try to do sth(努力做某事; 尽力做某事。用的是现在进行时,故排除A项和B项。C项强迫做某事,是force sb. to do sth(,故此 处若选择force项则该用被动语态be forced,而且force在此处意思也不符合文章的意思。故选择D。 80(【答案】 C 【解析】 对……不感兴趣用be uninterested in sth(。 Part ? Translation Section A 81(【答案】 最好的摆脱消极的自我评价的方法是要意识到你的自我评价远远不够客观并积极的 说服自己来肯定自己的优点。 【解析】Get rid of排除,摆脱;a negative self-image消极的自我评价,形象;be far from远 离……;convince sb(of sth(说服某人相信某事。 82(【答案】 生活中充满高尚的行为会产生一种美好的积极的循环。高尚的想法产生高尚的行为;而高尚的行为又给我们带来愉快的生活。 【解析】 Positive circle积极循环,周期;live a life of...过着……的生活;lead to导致,使产生。 83(【答案】 然而这种积极的循环比较难开始,一旦开始就很容易坚持。坚持做好事给我们的心境带来安宁,这对我们的快乐十分重要。 【解析】 Once曾经;bring sb. sth(给某人带来某东西;be important for对……很重要。 84(【答案】 这么说是因为叫此名字的电视节目现在在世界各地广受关注。自从它于1969年在纽约播出后的不久,它就成为美国电视节目的焦点。 【解析】 Go on the air开始播放。 85(【答案】 例如,说―非常难得的位置‖是可以接受的,这就是他的观点。你可能不同意,但他可能喜欢生活在煤气厂旁边的想法。 【解析】 Acceptable可接受的;delightfully大喜地,欣然地。 Section B 86(【答案】 It is important that one company should catch up with the market changes. 【解析】 使用固定句型:It is important that…; catch up with追上,赶上。 87.【答案】 Comparatively, western parents are more likely to attribute their children's success to their talent. 【解析】 相比较之下comparatively / comparatively speaking;更有可能be more likely to; 归因于 ascribe / attribute to / impute / put down to。 88(【答案】 Generally speaking, the better you prepare for the coming exam, the less nervous you become before it( 【解析】 一般来说generally speaking,固定表达;越……越……:the , 比较级,the , 比较级;为……做准备prepare for sth.。 89.【答案】 But for your timely help,I really didn't know how to accomplish this task. 【解析】 要不是but for,固定表达;及时地immediately/timely。 90(【答案】 In any case,we should not give the hope of life. 【解析】 在任何情况下in any case;放弃give up; abandon。 Part ? Writing Study Strategies at College 120 Good morning everyone. My name is Li Ming. I am glad to stand here to give a speech about my study strategies at colleges to all freshmen. I am the chairman of the Student Union. Firstly, let's welcome all freshmen here. When I entered our college, I was also a little puzzled about the study strategies because they are quite different from that of our high school. Therefore,I think it is necessary to introduce it.In college, we need to form a good habit of study, and adopt different methods to study different subjects. Besides,it is equally important to arrange our time well, as there are so many activities waiting us to take part in. But what we should remember is that obtaining knowledge is our main job in college. Meanwhile, we also need to get enough rest and learn to build relationship with others. Then, during the study period, we also have to combine studying by ourselves with cooperation with others, which is important for college students. At last,I send my best wishes to you and wish you a colorful and splendid college life. And I also help you can make great progress in your study. That's all. Thank you. 2011年江苏省普通高校“专转本”统一考试试卷 大学英语 第?卷(共100分) 注意事项: 1(答第?卷前,考生务必将自己的姓名、准考证号、考试科目用铅笔涂写在答题卡上。 2(每小题选出答案后,用2B铅笔把答题卡上对应题目的答案标号涂黑。如需改动,用橡皮擦干净后, 再选涂其他答案标号。如果答案不涂写在答题卡上,成绩无效。 Part I Reading Comprehension(共20小题,每小题2分,共40分) Directions: There are 4 passages in this part. Each passage is followed by some questions or unfinished statements. For each of them there are four choices marked A, B, C and D. You should decide on the best choice and mark the corresponding letter on the Answer Sheet with a single line through the center. Passage 1 Questions l to 5 are based on the following passage. Everyone loves to eat! People in every country enjoy this activity, There are so many different kinds of food in the world - Chinese food, Italian food, South American food,Indian food, and many others. It's fun to eat and compare, different types of cuisine, Italian food is sometimes very spicy, but South American food is usually much spicier. Indian food can be the spiciest of all-sometimes it's so hot that it can make your mouth burn! But that's okay because then you can drink some good Italian or French wine to reduce the burning! Some wonderful food comes from the Eastern countries. Chinese food is popular in many parts of the world. Lots of people love Chinese style vegetables because they are so crispy. Because Chinese-style vegetables are usually cooked for a much shorter time, they are crisper than American-style vegetables. In fact, they are probably the crispest cooked vegetables you can find. Some people don't like American-style food. They think it is often less tasty and sometimes less nutritious than food in other countries. Americans spend nearly half their food dollars on meals away 121 from home. They eat more "fast food", such as hamburgers, hot dogs, pizzas, canned and frozen food, for fast food takes a short time to prepare. Often, Americans feel they are too busy to spend a lot of time cooking. It's easier to pick up a pack of fried chicken on the way home from work or take the famlily out for pizzas or burgers than to start opening cans or heating up frozen dinners after a long, hard day. Of course, in some other countries, like France, cooking is still an "art form". Some people believe that French cuisine is the best in the world. Certainly, each person has a different opinion, about which country has the tastiest food. But people in many places agree that French food is the most delicious and the best prepared. 1. The main idea of this passage is that____. A. American food is less delicious than food in other countries B. people in many countries prefer Chinese food C. it's interesting to compare a variety of food in different countries D. people enjoy different kinds of food in the world 2. We can infer from the passage that____. A. wine makes your mouth burn B. French chefs love to be artists C. South American food is not so spicy as Indian food D. the most wonderful food comes from Eastern countries 3. Which of the following statements is true according to the passage? ____. A. French food is delicious, but not well prepared B. Indian food has to be enjoyed with Italian or French wine C. Chinese-style vegetable are possibly the crispest D. American food is believed to be the most difficult to cook 4. Americans eat a lot of "fast food" because____. A. they believe American food is the best B. American food contains more nutrition C. they don't like food in other countries D. they don't think they have enough time to prepare meals 5. Which of the following statements is NOT true according to the passage? ____. A. Chinese-style vegetables are often cooked for a short time B. American food is often believed to be less tasty C. Everyone in the world dislikes Indian food D. People have different, opinions about French cuisine Passage 2 Questions 6 to 10 are based on the following passage. If American children have to use mosquito ringtone (静音) of cell phones at school, a university in Japan is doing the opposite: giving cell phones to students. And not just any cell phone - the iPhone 3G. According to Asiajin, about 550 students and staff members in the School of Sociallnformatics at Tokyo-based university Aoyama Gakuin received the iPhone 3G for free aspart of their study materials. This is the result of a deal that Aoyama Gakuin signed with SoftBank, the only sales representative of the iPhone in Japan. The number of students using the iPhone is expected to reach about l,000. 122 This is the first time a particular cell phone has been used on such a huge scale at a Japanese university. The phone will work as a study tool for students. As it comes with the global positioning system (GPS), the university plans to use it to check student attendance. Dropping classes is a big problem in Japan, where regular attendance is an important factor in determining a student's grade. Students often fake attendance by getting classmates to answer roll calls. Now, with the iPhone 3G, the school plans to keep better tabs on (监控) its students. Students are allowed to use the phone for attendance reporting (but only if they are actually in the classroom, a fact that will be proved by the phone's GPS), lecture podcasting, and online examinations. A student can't answer the roll call using the phone from any location other than the classroom. Students can, of course, still cheat the new system by leaving their phones with fellow classmates, but this is not very likely to happen, as people tend to keep a lot of private information on their phones that they don't want to share with others. As for calling and data plans, the university covers the basic fee. The hardware itself is free, but students will have to pay when they exceed downloading limits. 6. According to the passage, American children are ____at school. A. encouraged to use cell phones B. not encouraged to use cell phones C. offered a new type of free cell phones D. allowed to use free cell phones 7. Which of the following statements is NOT true of the School of Social Informatics? A. Students receive the iPhone 3G for free B. Students use the iPhone 3G as a study tool C. Students have received the iPhone 3G before D. Students can save private information on the iPhone 3G 8.Which of the following problems might be solved by using the iPhone 3G? ____. A. Fake attendance reported by classmates B. Failure of on-line examinations C. Answer to roll calls at school D. High cost of telephoning 9. The following functions of the iPhone 3G are mentioned in the text EXCEPT____. A. attendance reporting B. lecture podcasting C. online examinations D. downloading payment 10. This passage is mainly about ____. A. the uses of the iPhone 3G at a Japanese university B. the advantages of the iPhone 3G in Japanese universities C. the control of dropping classes in a Japanese school D. the use of GPS to locate students at school Passage 3 Questions 11 to 15 are based on the following passage. Coco Chanel, a French woman, is very famous for her success in the fashion world. The life of 123 Coco Chanel is a classic rags-to-riches story. Born in 1883, she was abandoned by her father after her mother died. She was raised by her aunts who taught her to sew and gave her the nickname Coco, which means "little pet". Starting out as a hat designer, Coco went against the fancy hats being worm at the time. "How can a brain function under those things," she wondered. Instead, she created simple, but beautiful designs which became popular with fashionable young women of Paris. Strangely enough, World War I gave her the chance to develop her idea of simple elegance from hats to clothing. Due to the shortage of material during the war, she took the opportunity to show that less can truly be more, introducing turtle-neck (叠领)sweater and pants for women. In 1923, Coco launched her line of perfumes. "A woman who does not wear perfume has no future!" she said, and women believed her. Since its first appearance, Chanel No. 5 has been the world's most popular perfume, with another bottle being sold every 30 seconds. And Coco herself inspired generations of women designers and businesswomen to pursue their dreams. 11. The world famous brand "Chanel" is named after____. A. a little animal B. a hat company C. the nickname of a French woman D. the surname of a French girl 12. Coco had her hats ____those of her time at the beginning of her designing. A. fancier than B. different from C. similar to D. more expensive than 13. According to the passage, Chanel's designs can be best described as____. A. fashionable B. luxurious C. simple and elegant D. A and C 14. The launch of Chanel perfumes is based on the belief that____. A. a woman who does not wear perfume has no future B. it will be the world's most popular perfume C. it will be sold every 30 seconds D. women are fond of all kinds of perfumes 15. Coco Chanel's success story tells us that____. A. becoming rich is not so difficult B. women are more likely to succeed C. people can pursue their dreams D. the poor are more likely to succeed Passage 4 Questions 16 to 20 are based on the following passage. Everyone knows that laziness is not good. We have probably all bad lectures telling that laziness is immoral, that it is wasteful, and that lazy people will never gain anything in life. But laziness can be more harmful than that, and it is often caused by more complex reasons. Some people who appear to be lazy are suffering from much more serious problems. They may not trust their fellow workers and they are unable to join in any group task for fear of being laughed at or for fear of having their ideas stolen. These people who seem lazy may be affected by a fear of failure that prevents successful work. Some people are so busy planning great deals that they are unable to deal with "easier" work on hand. 124 Still others are not avoiding work; strictly speaking, they are simply putting off their work. Laziness, however, can actually be helpful. Some people may look lazy while they are really thinking, planning and researching. We should all remember that great scientific discoveries happened by chance. Newton wasn't working on the farm when the apple hit him and he devised the theory of gravity. All of us would like to have someone "lazy" build the car we buy, particularly if that "laziness" were caused by the worker's taking time to check each step of his work and to do his job right. And sometimes, being "lazy" - that is, taking time off for a rest - is good for the overworked students or executives. Taking a rest can be particularly helpful to the sportsman who is trying too hard or the doctor who is simply working himself overtime too many evenings in the hospital. So be careful when you want to call someone lazy. That person may be thinking, resting, or planning his or her next work. 16. The main idea of this passage is that____. A. laziness is a bad habit that everyone wants to get rid of B. there are advantages and disadvantages in being lazy C. laziness is the sign of very serious emotional problems D. lazy people usually do their work more carefully 17. Which of the following statements is mentioned in the passage? ____. A. Laziness is a kind of mental disease B. Laziness is more beneficial than harmful C. Laziness cannot be explained D. Laziness is sometimes due to a fear of failure 18. Which of the following ideas does the passage support? ___. A. Most of the time laziness is a good quality B. Most assembly workers are lazy C. The word "laziness" is sometimes misused D. Most overworked people are lazy 19. The author's attitude towards laziness is ____. A. objective B. subjective C. critical D. humorous 20. As used ill this passage, the word "devised" (in Paragraph 2) probably means____. A. understood B. wrote C. created D. proved Part? Vocabulary and Structure(共40小题,每小题1分,共40分) Directions: There are 40 incomplete sentences in this part. For each sentence there are four choices marked A, B, C and D. Choose the ONE answer that best completes the sentence. Then mark the corresponding letter on the Answer Sheet with a single line through the center. 21. By the middle of the 21st century, the vast majority of the world's people ____in cities rather than in rural areas. A. are living B. will be living C. have lived D. will have lived 22. All flights ____because of the snowstorm, many passengers could do nothing but stay at the airport. A. had been cancelled B. were cancelled C. having been cancelled D. have been cancelled 23. The problem is believed ____at the meeting two days ago. A. to have discussed B. to have been discussed 125 C. being discussed D. having been discussed 24. It was so hot that they ____the electric fan ____all through the night. A. had; running B. had; run C. had; ran D. had; to run 25. Some of the committee members suggested ____ a party for the visitor, but the Chairman didn't approve. A. to arrange B. arrange C. arranging D. to be arranging 26. - Do you regret paying five hundred dollars just for the oil painting? - No. I would gladly have paid ____for it. A. twice as much B. twice so much C. as much twice D. so much twice 27. ____me most was that the young man who had lost both arms in an accident could play the piano beautifully with his feet. A. That amazed B. It amazed C. Which amazed D. What amazed 28. The team leader promised to attend the meeting ____he arrived at the office. A. for the time being B. the moment C. as much as D. the day 29. Bob ____hard, otherwise he ____this exam. A. must have worked.... would fail B. mustn't have worked... wouldn't fail C. can't work …wouldn't have failed D. can't have worked...wouldn't have failed 30. Scarcely ____asleep when a knock at the door awakened her. A. had she fallen B. she had fallen C. did she fall D. she fell 31. ____man realized that the brain controlled our thinking. A. It was not until the 18th century that B. It was the 18th century since C. Not until the 18th century did D. It was until the 18th century which 32. The president of the college, together with the deans, ____planning a conference for setting a series of regulations. A. were B. are C. is D. be 33. Advertising is distinguished from other forms of communication ____the advertiser pays for the message to be delivered. A. in that B. in which C. in order that D. in the way 34. My grandfather has a pair of ____. A. Spanish leather black boots B. black Spanish leather hoots C. Spanish black leather boots D. leather Spanish black boots 35. I don't think you can finish painting the fence alone in such a short time, ____? A. do I B. can you C. can't you D won't you 126 36. There were ____accidents on the road this year than last year, but this doesn't mean there is ____need for careful driving. A. less; less B. fewer; little C. less; fewer D. fewer; fewer 37. Needless to say, the second list of words ____more easily than the first one. A. can remember B. can be remembered C. will remember D. be remembered 38. When I went out jogging early this morning, I found the ground wet. It ____last night. A. might have rained B. may have rained C. can have rained D. must have rained 39. The reason why I came back is ____she would have been very angry if I hadn't. A. that B. because C. for D. that because 40. Don't come tomorrow. I'd rather you ____next weekend. A. will come B. came C. to come D. had come 41. How can a crowd ____while a woman is being attacked and robbed? A. stand for B. stand up to C. stand by D. stand up 42. Although the financial crisis spread all over the southeastern Asian countries, the market in China remained ____as usual. A. fixed B. fresh C. firm D. steady 43. She ____his anger though he did not say a word to her. A. understood B. heard C. sensed D. learned 44. The government has ____the prices of medicines for the benefit of the people. A. brought down B. went down C. closed down D. reduced down 45. In some cases, your instructor may tell you the topics ____or may give you a choice of topics to write about. A. in advance B. ahead of C. above all D. just right 46. ____there is little we can do to change the weather, we can at least know what kind of weather to expect. A. Where B. Lest C. Although D. Unless 47. The Browns have not ____yet and I doubt whether they will come. A. turned in B. turned out C. turned up D. turned to 48. She will have to look for somewhere else to work, for she can't ____such loud noise any longer. A. come up with B. catch up with C. keep up with D. put up with 49. It is reported that many automobile accidents ____careless driving. A. attributed to B. resulted from C. contributed to D. raised from 50. I am ____grateful for the kindness you have shown my son. A. excessively B. frequently C. certainly D. happily 51. The old man expects that his son will ____the company when he retires. A. take to B. take out C. take off D. take over 52. The police asked for the ____of the public in tracing the missing boy. A. cooperation B. combination C. possibility D. capacity 53. In computer programming, this model is ____to any of the others we have ever had. 127 A. more superior B. the more superior C. the most superior D. superior 54. ____the large amount of time devoted to English listening every day, most college students feel it hard to understand English news broadcasting. A. However B. If C. Despite D. Even 55. I have given up trying to convince him; there is no point ____arguing with him. A. by B. with C. for D. in 56. No one knew the man's marital status until he ____it in the course of a conversation. A. displayed B. revealed C. exhibited D. exposed 57. We will take a tour around Hangzhou next year ____we have enough money. A. lest B. until C. unless D. provided 58. ____the traffic jam, they would have arrived earlier. A. Except for B. But for C. Apart from D. Instead of 59. I'll be very glad that you agree to ____the pearl robbery in the shopping mall. A. look up B. look for C. look over D. look into 60. The mysterious guest on the show is ____the president of our university. A. no more than B. none other than C. not other than D. nobody than Part III Cloze(共 20小题,每小题1分,共 20分) Directions: There are 20 blanks in the following passage. For each blank there are four choices marked A, B, C and D. You should choose the ONE that best fits into the passage. Then mark the corresponding letter on the Answer Sheet with a single line through the center. In any comprehension text you will find words that you don't know, You can 61 them up in a dictionary, of course, 62 it is a good idea to get 63 the habit of using a dictionary as 64 as possible, particularly if you are preparing 65 an examination. In fact, if you read the text 66 and think, it is usually possible to guess the 67 of most words that you don't know. Look 68 the context of each word - the sentence that it is 69 , and the sentences that come before and after. Look to see 70 the word is repeated 71 in the text; the more often it is 72 , the easier it is to understand. Some words 73 be guessed from looking at their 74 . But don't expect to be able to guess 75 of the new words in a text. There will be 76 that you can only get a general 77 of, and a few will be impossible. Don't 78 too much time worrying about these: the most 79 thing is to understand the text as a (an)80 as well as possible, and one or two difficult words will not usually make much difference. 61. A. check B. look C. find D. try 62. A. since B. because C. but D. therefore 63. A. into B. by C. from D. through 64. A. frequently B. rarely C. much D. farther 65. A. in B. into C. against D. for 66. A. quickly B. really C. strictly D. carefully 67. A. meaning B. usage C. context D. function 68. A. for B. after C. at D. on 69. A. with B. between C. for D. in 70. A. if B. what C. how D. where 128 71. A. later B. late C. lately D. latter 72. A. appeared B. counted C. used D. shown 73. A. must B. would C. should D. can 74. A. forms B. faces C. builds D. blocks 75. A. some B. any C. many D. all 76. A. some B. little C. none D. much 77. A. model B. example C. concept D. idea 78. A. waste B. have C. use D. try 79. A. interesting B. important C. pressing D. advantageous 80. A. unity B. entire C. whole D. complete 第 ?卷(共 50分 ) Part IV Translation(共35分) Section A(共 5小题,每小题4分,共20分) Directions: Translate the following sentences into Chinese. You may refer to the corresponding passages in Part?. 81. Indian food can be the spiciest of all - sometimes it's so hot that it can make your mouth burn! But that's okay because then you can drink some good Italian or French wine to reduce the burning! 82. The number of students using the iPhone is expected to reach about l,000. This is the first time a particular cell phone has been used on such a huge scale at a Japanese university. 83. Due to the shortage of material during the war, she took the opportunity to show that less can truly be more, introducing turtle-neck (叠领) sweater and pants for women. 84. They may not trust their fellow workers and they are unable to join in any group task for fear of being laughed at or for fear of having their ideas stolen. 85. Some people may look lazy while they are really thinking, planning and researching. We should all remember that great scientific discoveries happened by chance. Section B(共5小题,每小题3分,共15分) Directions: Translate the following sentences into English. 86.日本地震后,人们开始担忧核能安全了。 87.是朋友的关爱和信任帮她度过了艰难的日子。 88.如果我负责这个项目,我会更好地利用现有的资金。 89.尽管石油的价格大幅度上涨,还是有很多人买车。 90.要在学习上取得进步,学生需要具备独立学习的能力,而不是依赖老师。 Part V Writing(共15分) 129 Directions: For this part, you are allowed 30 minutes to write an essay with the title How to Relieve My Pressure. You are required to write at least 120 words, following the outline given below: 1.目前大学生面临各种压力,如学习压力、经济压力、就业压力等等; 2(请描述你生活中的主要压力; 3(你通常是如何缓解这种压力的。 How to Relieve My Pressure 2011年江苏省普通高校专转本统一考试试卷答案与解析 Part I Reading Comprehension Passage 1 【解析】 文章大意: 每个人都喜欢吃东西。世界上有许许多多不同种类的食物:中国美食、意大利美食、南美美食、印第安美食等等。品尝不同种类的美食是非常有趣的。意大利美食口味辛辣,一般来说南美美食口味更辣,印第安是各种菜系中最辣的,能辣的让人的嘴唇像是着火了。不过没关系,你可以喝点意大利或者法国啤酒来降降火。 很多经典美食来自东方。中国食物在世界各地流行。很多人喜欢中式蔬菜的脆爽。之所以如此脆爽是因为中式蔬菜比美式蔬菜烹饪的时间短。事实上,中式蔬菜可能是世界上最脆的食物。 一些人不喜欢美式食物。他们认为美式食物相对其他国家食物口感较差缺少营养。美国人食物消费一半是花在肉类上的。他们是很多快餐,比如汉堡,热狗,披萨,灌装、冷冻食品。因为这些快餐花费时间少。美国人认为没时间烹饪。辛苦工作一天后在回家路上带着快餐吃或者全家到外面去吃披萨汉堡比开火做饭更便利。 当然,在其他国家,像是法国,烹饪是一种艺术形式。很多人相信法国大餐是世界上最好吃的美食。当然也有人有不同意见。但是大部分国家的人认为法国美食是世界上最美味准备最完美的食物。 1(【答案】 D 【解析】根据短文内容,A、B、C项正确,但是不是文章的主旨,只是陈述文章中的某一个方面。 2(【答案】 C 【解析】 A项啤酒使人嘴唇像火烧,与文章描述正好相反。B项法国厨师喜欢成为艺术家文中未提及到。D项最好的食物来自东方。根据文中描述,大部分人认为法国食物是世界上最棒的。C项南美食物不像印第安食物一样的辣。正确。 3(【答案】C 【解析】 A项法国食物美味但是不精心准备,与文章描述相反。B项印第安食物与意大利或者法国啤酒一起吃比较好。文中只是描述可以这样一起吃。并没有评价。D项人们认为美国食物是世界上最难烹饪的食物。与事实相反。C项中国式食物可能世界上最脆的食物,正确。 4(【答案】 D 【解析】 根据文中描述,美国人认为自己很忙没时间烹饪美食,在快餐与烹饪美食之间他们认为快餐更便利节省时间。 5(【答案】 C 【解析】A项中国式蔬菜烹饪时间较短。与文中描述一致。B项美国食物口感较差,与文中描述一致。D项人们对法国大餐有不同的看法。与文中描述一致。C项世界上每个人都不喜欢印第安食物。文中未提及。 Passage 2 文章大意: 如果要求美国儿童在校使用小型静音手机,日本一所大学却相反:给学生发放手机。不仅仅是手机 130 还有iPhone 3G。 根据Asiajin报到,青山学院大学信息学院的550名学生和在校职工收到iPhone 3G,用做学习资料的一部分。这是根据日本软银(日本唯一的iPhone销售代理)和青山学院大学签订 合同 劳动合同范本免费下载装修合同范本免费下载租赁合同免费下载房屋买卖合同下载劳务合同范本下载 。使用iPhone的学生数量预计达到1000人。这也是在日本大学校园第一次广泛的使用手机。 这种手机作为学习工具之一。使用它的全球定位系统,学校计划用它来考察学生的出勤。在日本出勤是个很大的问题。而日常的出勤是评价学生分数的重要因素。以前学生通过让同学点名代喊假出勤。 现在,使用iPhone后学校计划对学生行踪很好监控。学校允许学生使用手机听报告演讲、在线测试。除了教室以外,学生不能在其他地方使用手机回应点名。 当然学生通过把手机交给同学依然能够欺骗定位系统。但是这种情况不可能发生。因为人们都在手机里面保存很多私人信息不会与别人分享。 至于点名计划,学校承担基本费用。硬件是免费的。但是当学生超过下载限制,学生依然需要付费。 6(【答案】 B 【解析】 根据文章内容可知美国不鼓励学生使用手机,更不会提供学生新型手机。 7(【答案】 C 【解析】根据文章内容,该学校提供给学生免费的iPhone 3G作为学习工具之一。iPhone 3G具有存储信息的功能,学生可以在上面存储私人信息。 8(【答案】A 【解析】 根据文章内容,通过使用iPhone 3G,学校可以控制学生的行踪考察学生的出勤。至于考 试成败、高额电话费问题、没有提及。学校点名问题未必能很好解决。同学代点名的问题可以解决。因 为iPhone 3G里面存储了很多私人信息。 9(【答案】 D 【解析】 D项不是iPhone 3G的功能。其他几项都有提及。 10.【答案】 A 【解析】 全文讲述的是日本的一个大学给学生发放iPhone 3G的相关情况。 Passage 3 文章大意: Coco Chanel是闻名于时尚界的一名法国女性。她的一生是典型的从贫到富的一个传奇。1833年出生,母亲死后她就被她的父亲抛弃。她被阿姨抚养长大。她的阿姨教她缝纫,给她起了名字叫做Coco意思是小宠物。 起初她做帽子设计师,一反当时流行华丽花俏的设计理念,她设计了简单但是美丽的帽子,很快就在巴黎时尚女性中流行。 第一次世界大战使他的简单美设计思想得到了发展。战争中物资短缺,她利用机会设计了女性的叠领的上衣和宽松的裤子。 1923年Coco开发了香水生产线。她说,一个不使用香水的女性是没有未来的。女人们都相信她。从开始出现,Chanel No.5一直是世界上最流行的香水。每30秒卖出一瓶。Coco的传奇故事激励了女性设计师、女性商人去追求他们自己的梦想。 11.【答案】 D 【解析】设计师的名字是Coco Chanel,以她的名字命名了这个品牌。 12.【答案】 B 【解析】根据文章内容可知,Coco与当时的设计理念有着很大的不同。 13.【答案】 D 【解析】 根据文章内容可知Coco的时尚理念是―时尚、简单‖ 14.【答案】A 【解析】 Chanel香水生产是基于Coco的一个观点:一个不使用香水的女性是没有未来的。 131 15.【答案】 C 【解析】通过Coco的人生经历可以看出来,只有追逐梦想才能实现梦想。 Passage 4 文章大意: 每个人都知道懒惰是不好的。所有的演讲中都告诉我们懒惰是不正常的,浪费生命,懒惰的人一无所获。但是懒惰有更多的坏处,诸多复杂的原因却引起了懒惰。有些人看上去很懒是因为正遭受严重的困扰。他们不信任同事,担心思想被别人使用,或者被同事嘲笑不敢加入团队工作。这些人看上去懒惰其实是害怕不能完成工作任务的失败。一些人忙于执行大计划而怠于做琐碎小事。其他人不是逃避工作,严格的说他们是将工作推后。 然而懒惰有时候的确有用。一些人在思考设计研究的时候看上去比较懒散。我们应该了解很多著名的科学发现都是偶然发生的。当苹果击中牛顿的时候他没有在农场里工作。他发现了万有引力定律。我们都喜欢购买―懒人‖设计的汽车。有时,懒惰也是休息的一种方式。这对超负荷工作的学生员工来说很有用。对于刻苦锻炼的运动员和在医院连续加夜班的医生来说,休息一下特别有帮助。所以你要注意,当你说一个人懒惰的时候他可能正在思考休息或者计划未来的工作呢。 16.【答案】B 【解析】 文章一开始介绍对懒惰的通常认识。然后表述不同看法。举出一些看似懒惰其实不然的一些实例。通篇围绕懒惰的优缺点进行描述。 17.【答案】 D 【解析】 A项懒惰是一种精神疾病,文中未提及。B项懒惰好处多与坏处。文中未提及。C项懒惰有借口文中未提及。D项懒惰有时候是害怕失败引起的。文中提及到得。 18.【答案】 C 【解析】 文章支持的观点是懒惰一词有时候被误用。正是全文所要描述的主要内容。 19.【答案】 A 【解析】 作者对懒惰的态度是主观的。他主观设想了看上去懒惰的人实际所处的状态。 20.【答案】 C 【解析】 devised在第二段中的意思为―发现、创造‖。 Part?Vocabulary and Structure 21.【答案】 D 【解析】根据句意―到了21世纪中叶,大多数人将会居住在城市而不是农村‖,应使用将来完成时。 22.【答案】A 【解析】 根据句意―由于暴风雪所有航班都取消了,许多旅客只能滞留在机场‖,应使用被动语态。过去完成时。 23.【答案】 B 【解析】 根据句意―两天前在会议上,大家都认为将会讨论这个问题‖,应使用被动语态、过去将来完成时。 24.【答案】A 【解析】 根据句意―天气太热了他们一整晚都开着电风扇。‖应使用过去完成进行时。 25.【答案】 C 【解析】句意―一些成员建议给参观者安排一个聚会,但是负责人不同意。‖suggest后面直接跟动名词。 26.【答案】A 【解析】句意―你不后悔在油画上面画了五百多美元,不后悔,即使双倍价格我也愿意支付。‖固定搭配。 27.【答案】 D 132 【解析】 句意―年轻人在事故中失掉胳膊能够用脚弹奏钢琴,这让我震惊‖。What引导的主语从句。 28.【答案】 B 【解析】 句意―组长答应一到办公室就来参加会议‖,the moment引导的状语从句。 29.【答案】 D 【解析】 句意―鲍勃不能努力学习,否则他不会不能通过考试。‖前句情态动词引导的完成时态,后句过去将来完成时。 30.【答案】A 【解析】 句意―他刚要入睡就有敲门声吵醒她。‖Scarcely when意为刚……,就……,when引导一般过去时从句,Scarcely引导的主句使用过去完成时。 31.【答案】A 【解析】句意―直到18世纪人们才意识到大脑控制着我们的思维。‖―It is / was not until.... that.‖,译作―直到……才……‖,它是―not until…,‖的专用强调句型。 32.【答案】 C 【解析】 句意―校长与系主任正在计划召开会议讨论了一系列的 管理制度 档案管理制度下载食品安全管理制度下载三类维修管理制度下载财务管理制度免费下载安全设施管理制度下载 。‖主语是校长,应该使用单数形式。 33.【答案】A 【解析】 句意―广告与其他形式联系方式区分开来因为广告主付费传递信息。‖in that意为―因为‖符合题设要求。 34.【答案】 B 【解析】 句意―我爷爷有一双黑色西班牙皮靴。‖在通常情况下,形容词修饰名词一般的顺序为―限定词+形容词+名词‖。而多个形容词的排列则大致遵循以下原则:描绘形容词?大小(长短高低)形容词?形状形容词?年龄(新旧)形容词?颜色形容词?国籍形容词?材料形容词?用途(类别)形容词叶名词。 35.【答案】B 【解析】 句意―我认为你不能再那么短的时间内油漆篱笆,对吗,‖本题目考察的是宾语从句的反义疑问句。I think( believe,suppose)和I don't( think,believe,suppose)引导的从句是反意从句。 36.【答案】 B 【解析】 句意―今年的事故少于去年,但这不意味可以减少小心驾驶的需要。‖accidents可数名词,需要用few修饰,need不可数名词,需要用little修饰。 37.【答案】 B 【解析】 句意―不用说,第二行的单词比第一行的更容易记忆。‖属于情态动词的被动语态形式。 38.【答案】 D 【解析】 句意―当我一早出去慢跑发现地面是湿的。昨晚一定是下雨了。‖must have done表示对过去某事肯定的猜测。 39.【答案】A 【解析】 句意―我返回的原因是如果我不回来她肯定生气。‖通常情况下名词reason后面的表语从句是用that引导的。 40.【答案】 B 【解析】 句意―明天不要来了,我宁愿你下个周末过来。‖would rather somebody did something是指宁愿某人做某事,这里用did是因为这种用法表达的是虚拟语气,故动词用过去式。 41.【答案】 C 【解析】 句意―当一个妇女被袭击抢劫的时候,一群人怎么能够袖手旁观,‖根据句意C项符合。A项意为代表,B项意为勇敢面对,D项意为站立。 42.【答案】 D 【解析】 句意―尽管经济危机在所有的东南亚国家中发生,但是中国的市场依然稳步增长。‖A项意为133 固定的,B项意为新鲜的,C项意为坚定的。此处用steady更为适合。 43.【答案】 C 【解析】句意―尽管他没有说一句话,她她依然感觉到他的愤怒。‖C项为正确答案。 44.【答案】A 【解析】 句意―为了人民的利益政府降低了药品价格。‖A项意为降低,B项意为落下,下降,C项意为关闭(D项意为减少。 45.【答案】 【解析】 句意―在某些情况下你的导师会提前告诉你主题或者给你一些主题供你选择去写论文。‖A项意为提前,符合句意。B项意为在……之前为介词词组,C项意为总而言之,D项意为正合适。 46.【答案】 C 【解析】句意―尽管我们改变天气可能性很小,但是我们至少可以预测天气。‖前后句表示转折关系C项正确。 47.【答案】 C 【解析】 句意―布朗一家还没有出现我怀疑他们是否回来。‖C项正确。A项意为上交,B项意为结果是,D项意为求助,转向。 48.【答案】 D 【解析】 句意―她将要找另外的地方工作因为她不能再忍受这么大的噪音。‖D项正确,A项意为赶上,提出,B项意为赶上,对……产生预期的坏影响,C项意为跟上,和……保持联系。 49.【答案】 B 【解析】句意―据报道,许多机动车事故是粗心大意驾驶引发的。‖B项正确。A项意为把……归因于,把……归功于,C项意为有助于,促进,加剧。 50.【答案】 A 【解析】 句意―我非常感谢您对我儿子的友善。‖A项正确。B项项意为频繁地,C项意为当然,D项意为高兴地。 51.【答案】 D 【解析】句意―老人希望他退休后儿子可以接管他的公司。‖D项正确。A项意为开始从事,B项意为取出,C项意为起飞。 52.【答案】 C 【解析】 句意―警方号召公众共同合作追踪失踪男孩。‖C项正确。 53.【答案】 D 【解析】 句意―在计算机程序设计中,这个模块比我们以前遇到的都高级。‖考查形容词的比较级形式。 54.【答案】 C 【解析】 句意―尽管媒体花费大量时间练习听力,大部分大学生仍然感觉英语广播很难懂。‖C项 正确。despite意为尽管、不管、任凭,however意为但是、不过、然而。 55.【答案】 D 【解析】 句意―我已经放弃说服他了的确是没有理由与他争辩。‖D项正确。there is no point in doing sth.表示―做某事是没有意义的‖。 56.【答案】 B 【解析】 句意―在他谈话中揭露之前没有人了解这个人的婚姻状况。‖B项正确。A项意为展示,C项意为展览,D项意为暴露。 57.【答案】 D 【解析】 句意―一旦我们有足够的钱,下一年我们将会到杭州旅游。‖D项正确。B、C项前面主句都需要用否定形式。 58.【答案】 B 134 【解析】 句意―要不是交通堵塞,他们可以到的早一点。‖B项正确。A项意为除…之外,C项意为除……之外,D项意为代替。 59.【答案】 D 【解析】 句意―很高兴你能同意调查商场的珠宝劫案。‖D项正确。A项意为查询,B项意为寻找,C项意为仔细检查。 60.【答案】B 【解析】 句意―展会上的神秘嘉宾不是别人是我们大学的校长。‖B项正确。 Part ? Cloze 文章大意: 很多阅读文章你都会发现很多单词不认识。当然你可以通过字典查询。然后越少用字典对你越好特别是在准备通过考试的时候。事实上,如果你仔细阅读认真思考,你可以猜测你不认识单词的意思。看每个单词的上下文——单词所在的句子,这个句子的前后句。去看这个单词是否在文章的下文中重复出现,它出现的次数越多,越容易理解它的意思。一些单词可以通过查看他们的形式猜测词义。但是不要妄想能够猜测文中的所有生单词。有些单词你只能猜测一个大概的词义,一些甚至什么都猜不到。不要浪费时间担心这个,最重要的事情是尽可能的整体理解这篇文章。一两个生词没有影响的。 61.【答案】 B 【解析】look up意为查询。 62.【答案】 C 【解析】 文中表示转折关系。 63.【答案】A 【解析】get into意为(使)穿上,(使)陷入,学会。 64.【答案】 B【解析】 根据句意B项正确。 65.【答案】 D 【解析】 prepare for意为为……作准备,计划。 66.【答案】 D【解析】 根据句意D项正确。 67.【答案】A 【解析】 根据句意,A项正确。 68.【答案】 C 【解析】look at意为看。 69.【答案】 D 【解析】根据句意D项正确。 70.【答案】A【解析】 根据句意A向正确。 71.【答案】A 【解析】 根据句意A向正确。 72.【答案】 C 【解析】 文中描述越多使用越容易了解词义,因此C项正确。 73.【答案】 D 【解析】 根据句意表示能够做什么,D项的情态动词正确。 74.【答案】A 【解析】 根据句意A项正确。 75.【答案】 D 【解析】根据上下文,D项正确。 76.【答案】A 【解析】根据上下文A项正确。 77.【答案】D 【解析】 A项意为模型,雏形,B项意为例子,C项意为观念思想,D项意为概念。idea、concept -般含义为―想法‖、―概念‖。idea的使用范围最大,可以表示―念头‖、―想法‖、―主意‖等。 78.【答案】A 【解析】 waste与time搭配表示浪费时间。其他几个词与time搭配不符合上下文。 79.【答案】 B【解析】根据句意,文中想要表述的是最重要的事情。 80.【答案】 C 【解析】whole表示一个整体,用在文中最为合适。 Part ? Translation Section A 81.【答案】 印度食品可能是世界上最辣的——有时候辣到你感觉嘴里都要起火了。但是吃印度食品没有关系,因为吃完之后你可以喝一些好的意大利或法国白酒,这能减少烧灼感。 【解析】 注意几个关键词的翻译―can be‖、―burn‖、―reduce the burning‖。 82.【答案】 使用iphone的学生数量有望达到1000。这是第一次某个特定的手机能够在一所大学如此大范围的使用。 135 【解析】 ―The number of‖意为……的数目。―be excepted to‖直译被期望达到,意译有望达到。注意―This is the first time that‖从句的翻译。 83.【答案】 由于战争期间物质短缺,她利用这个机会展示用更少的东西可以获得更多,她引进了叠 领毛衫和女式短裤。 【解析】 ―Due to‖意为由于,―take the opportunity to‖意为利用机会做某事,―introducing‖在本句话中意 为引进开发。 84.【答案】 他们有可能不相信自己的同事,他们担心被嘲笑或害怕自己的观点被剽窃,因此不能参 与任何需要团队合作的任务。 【解析】 ―be unable to do sth. for some reason‖意为不能做某事是因为…… 85.【答案】 有些人在进行思考,制定计划或开展研究时,看似懒惰。我们都必须牢记:重大的科学 发现是碰巧出现的。 【解析】 ―while‖意为同时。 Section B 86.【答案】After the great earthquake struck Japan, people began to worry about the safety of nuclear energy. 【解析】 ―开始担忧‖翻译为begin to worry about。 87.【答案】 It was the care and trust of friends that helped her live through the hard time. 【解析】 主语从句。―度过‖翻译为live through。 88.【答案】 If it were l who were responsible for the project,1 would make a better usage of the funds at hand( 【解析】 ―更好地利用‖翻译为make a better usage of。 89.【答案】 Despite the drastic rises in oil prices, many people choose to buy cars. 【解析】 ―尽管‖翻译为despite,―大幅度‖翻译为drastic。 90.【答案】 To get progress in study, students need to be equipped with the ability of independent learning rather than replying on the teachers. 【解析】 ―而不是‖翻译为rather than。 Part V Writing How to Relieve My Pressure Today's college students face many pressures, mainly from parents' expectation, study, economy, employment and all kinds of competition. For me, my pressure is to study well and gain the permission from others, then find a good job and make my parents to live a better life. I don’t want to think too much. what I should do is as follows : First, set a clear aim of a struggle. The aim can give me motivation, and make me become active. Second, have the opinion to study hard and master the major skills well and strengthen my ability in all sides. Third, to make as more friends as possible, which will help each other. The last is to take part in outdoors activities, it can relax me in pressure life and talk to my family more. If I find pressure is nearly beyond my control, I'd turn to professional psychological guidance. Coincidence comes from hard working. 136 2012年江苏省普通高校“专转本”统一考试试卷 大学英语 (非英语类专业) Part I Reading Comprehension (共20小题,每小题2分,共40分) Directions: There are 4 passages in this part. Each passage is followed by some questions or unfinished statements. For each of them there are 4 choices marked A, B, C and D. You should decide on the best choice and mark the corresponding letter on the Answer Sheet with a single line through the center. Passage One Questions 1 to 5 are based on the following passage. Different countries and different races have different manners. Before entering a house in some Asian countries, it is good manners to take off your shoes. In European countries, even though shoes sometimes become very muddy, this is not done. A guest in a Chinese house sometimes does not finish a drink. He leaves a little, to show that he has had enough. In a Malay house, too, a guest always leaves a little food. In England, a guest always finishes a drink to show that he enjoys it. We should like to find out the customs of other races, so that they will not think us ill-mannered. But people all over the world agree that being well-mannered really means being kind and helping others, especially those older or weaker than ourselves. If you remember this, you will not go very far wrong. Here are some examples of the things that a well-mannered person does or does not do. He never laughs at people when they are in trouble. He is always kind either to people or to animals. When people are waiting for a bus, or in a post office, he lines up to wait his turn. In the bus, he gives his seat to an older person or a lady who is standing. If he accidentally bumps into (碰撞)someone, or gets in their way, he says ―Excuse me‖ or ―I’m sorry‖. He says ―Please‖ when making a request, and ―Thank you‖ when he receives something. He stands up when speaking to a lady or an older person, and he does not sit down until the other person is seated. He does not talk too much about himself. When eating, he does not speak with his mouth full of food. 1. According to the passage, a knowledge of the customs of other races ______. A. is very useful B. is unnecessary C. is unimportant D. does not mean much 2. A person with good manners thinks of ______. A. the older before himself B. himself before others C. no one but himself D. his personal interests 3. Which of the following is NOT true? A well-mannered person usually ______. A. says ―Please‖ when making a request B. makes an apology for bumping into someone accidentally C. sits where he is when speaking to a lady D. tries to help those who are in trouble 4. If you want to be well-mannered, ______. 137 A. you laugh at people when they are in trouble B. it’s all right to speak with your mouth full of food C. you should stop someone when he is talking D. you should speak after someone else has finished talking 5. As different countries have different manners, ______ A. it’s unnecessary to learn their manners B. we should try to find out the differences in the customs C. it should be wrong to go out of one’s way to do anything D. learning a little language would be helped Passage Two Questions 6 to 10 are based on the following passage. I entered St. Thomas’s Hospital as a medical student at the age of 18 and spent five years there. I was an unsatisfactory student, for my heart was not in it. I had always wanted to be a writer, and in the evenings, after my tea, I wrote and read. Before long, I wrote a novel, called Liza of Lambeth, which I sent to a publisher and was accepted. It appeared during my last year at the hospital and had something of a success. I felt I could afford to give up medicine and make writing my profession; so, three days after I graduated from the school of medicine, I set out for Spain to write another book. Looking back now, and knowing the terrible difficulties of making a living by writing, I realize I was taking a fearful risk. The next ten years were very hard, and I earned an average of ,100 a year. Then I had a bit of luck. The manager of the Court Theatre put on a play that failed. The next play he arranged to put on was not ready, and he was at his wit’s end. He read a play of mine and, though he did not much like it, he thought it might just run for the six weeks till the play he had in mind could be produced. It ran for fifteen months. Within a short while, I had four plays running in London at the same time. Nothing of the kind had ever happened before. I was the talk of the town. 6. When the author was a medical student, he ______. A. had some trouble with his heart B. was a very good student C. wanted to be a writer after graduation D. was satisfied with what he was doing at the time 7. When the author wrote his first novel, ______. A. he sent it to a publisher but it was not accepted B. he was still studying at the medical school C. he succeeded in publishing it though it was not a success. D. he had graduated from the medical school. 8. The author gave up medicine because at that time ______. A. he thought he could make a living by writing B. he knew the success of the book was natural C. he knew it was no risk to be a writer D. he was quite rich after the success of his book 138 9. For the first ten years of his writing career after his graduation, the author earned an average of ,100 a year, which was ______. A. a great sum B. a bit of luck C. a small sum D. a moderate success 10. The manager of the Court Theatre agreed to put on the author’s play because ______. A. he thought the author was a good playwright B. he liked the author’s plays very much C. he failed to arrange a new play in time D. he heard that the author had studied medicine before Passage Three Questions 11 to 15 are based on the following passage. In the last 500 years, nothing about people –-- not their clothes, ideas or languages --- has changed as much as what they eat. The original chocolate drink was made from the seeds of the cocoa tree by South American Indians. The Spanish introduced it to the rest of the world. And although it was very expensive, it quickly became fashionable. In London, shops where chocolate drinks were served became important meeting places. Some still exist today. The potato is also from the New World. Around 1500, the Spanish brought it from Peru to Europe, where it was soon widely grown. Ireland became so dependent on it that thousands of Irish people starved when the crop failed during the ―Potato Famine‖ of 1845-46, and thousands more were forced to emigrate to America. There are many other foods that have traveled from South America to the Old World. But some others went in the opposite direction. Brazil is now the world’s largest grower of coffee, and coffee is an important crop in Columbia and other South American countries. But it is native to Ethiopia. It was first made into a drink by Arabs during the 1400s. According to an Arabic legend, coffee was discovered by a goatherd named Kaldi. He noticed that his goats were attracted to the red berries on a coffee bush. He tried one and experienced the ―wide-awake‖ feeling that one-third of the world’s population now starts the day with. 11. According to the passage, ______ has changed the most in the last 500 years. A. food B. chocolate C. potato D. coffee 12. ―Some‖ in ―Some still exist today‖ (Para.1) means ______. A. some cocoa trees B. some chocolate drinks C. some shops D. some South American Indians 13. Thousands of Irish people starved during the ―Potato Famine‖ because ______. A. they were so dependent on the potato that they refused to eat anything else B. they were forced to emigrate to America C. the weather conditions in Ireland were not suitable for growing the potato D. the potato harvest was bad. 14. Coffee originally came from ______. A. Brazil B. Columbia C. Ethiopia D. Arabia 139 15. The Arabic legend is used to prove that ______. A. coffee was first discovered by Kaldi B. coffee was first discovered by Kaldi’s goats C. coffee was first discovered in South American countries D. coffee drinks were first made by Arabs Passage Four Questions 16 to 20 are based on the following passage. Around the world more and more people are taking part in dangerous sports and activities. Of course, there have always been people who have looked for adventure ---- those who have climbed the highest mountains, explored unknown parts of the world or sailed in small boats across the greatest oceans. Now, however, there are people who seek an immediate thrill from a risky activity which may only last a few minutes or even seconds. I would consider bungee jumping to be a good example of such an activity. You jump from a high place (perhaps a bridge, or a hot-air balloon) 200 metres above the ground with an elastic rope tied to your ankles. You fall at up to 150 kilometres an hour till the rope stops you from hitting the ground. It is estimated that 2 million people around the world have now tried bungee jumping. Other activities as risky as bungee jumping involve jumping from tall buildings and diving into the sea from the top of high cliffs. Why do people take part in such activities as these? Some psychologists suggest that it is because life in modern societies has become safe and boring. Not very long ago, people’s lives were constantly under threat. They had to go out and hunt for food, diseases could not easily be cured, and life was a continuous battle for survival. Nowadays, according to many people, life offers little excitement. They live and work in a comparatively safe environment; they buy food in shops, and there are doctors and hospitals to look after them if they become ill. The answer for some of these people is to seek danger in activities such as bungee jumping. 16. The best title for the passage would be ______. A. Dangerous Sports: What and Why B. The Boredom of Modern Life C. Bungee Jumping: Is It Really Dangerous? D. The Need for Excitement 17. More and more people today ______. A. are trying activities such as bungee jumping B. are climbing the highest mountains C. are coming close to death in sports D. are exploring unknown places 18. In bungee jumping, you ______. A. jump as high as you can B. slide down a rope to the ground C. attach yourself to a rope and fall towards the ground D. fall towards the ground without a rope 19. People probably take part in dangerous sports nowadays because ______. A. they have a lot of free time 140 B. they can go to hospital if they are injured C. their lives lack excitement D. they no longer need to hunt for food 20. The writer of the passage has a ______ attitude towards dangerous sports. A. positive B. negative C. neutral D. nervous Part II Vocabulary and Structure (共40小题,每小题1分,共40分) Directions: There are 40 incomplete sentences in this part. For each sentence there are 4 choices marked A, B, C and D. Choose the ONE that best completes the sentence. Then mark the corresponding letter on the Answer Sheet with a single line through the center. 21. Wounded ______ he was, he refused to come back from the front. A. since B. for C. as D. although 22. After the class meeting, she ______ us about her life in the United States. A. went on telling B. went to tell C. went telling D. went on to tell 23. The owner and editor of the newspaper ______ the conference. A. were attending B. was to attend to C. is to attend D. are to attend 24. It ______ John and Peter who helped me with my work the other day. A. were B. is C. had been D. was 25. It’s high time ______ him a severe lesson for being late. A. you give B. you gave C. you’ll give D. you should give 26. ______, we decided to leave at once, as we didn’t want to risk missing the last bus. A. Being pretty late B. As it being pretty late C. It being pretty late D. It was being pretty late 27. No announcement has been made concerning _____ on the next flight. A. who go B. who is going C. is who going D. who went 28. The electric furnace _____ before it can be used. A. needs repairing B. should be in repair C. must repair D. has to be repairable 141 29. We can’t enter the room because its door ______. A. locked B. locks C. is locked D. is locking 30. ______ Charles starts a job, he will do it ______ it is finished. A. Since, where B. Each time, once C. Once, till D. The moment, as 31. Seldom ______ any mistakes during my past 10 years of working here. A. I did make B. did I make C. I would make D. I made 32. ______ in the park, I came across my former English teacher. A. If wandered B. If wandering C. While wandered D. While wandering 33. I’m sorry ______ so long. I forgot to tell you in advance that I would have a meeting this afternoon. A. to keep you waited B. to keep you waiting C. to have kept you waiting D. to have kept you waited 34. I bitterly regret ______ him the sad story. A. having told B. to have told C. to tell D. told 35. The house that ______ in the floods must be rebuilt before winter comes. A. has been washed away B. has to wash away C. has them washed away D. has washed them away 36. ―Do you think he will make a good president?‖ ―He is just ______ Bill.‖ A. as qualified as B. more qualified like C. the same qualified as D. much qualified than 37. Although ______ alone at home, he didn’t feel lonely. A. leaving B. leave C. leaves D. left 38. The crime was not discovered till 48 hours later, ______ gave the criminals plenty of time to get away. A. which B. that C. as D. what 39. After 15 years abroad, his brother came back only ______ his hometown severely damaged in an earthquake. A. to be finding B. finding C. found D. to find 142 40. ______ the final exam, I would go shopping this afternoon. A. As for B. Because of C. But for D. Due to 41. By the time you get home, we ______ in Qingdao for one week. A. will be staying B. will have stayed C. would have stayed D. have stayed 42. If we ______ this speed, we will arrive there in about two days. A. keep up B. keep from C. keep out D. keep away 43. They are shocked to find ______ water has been polluted. A. large quantities of B. a great number of C. a good deal of D. a plenty of 44. This university is outstanding ______ its faculty and reference resources. A. in place of B. in terms of C. by means of D. by way of 45. Liquids are like solids ______ they have a definite volume. A. on that B. for that C. with that D. in that 46. Her sense of inferiority ______ her unfortunate family background. A. results from B. brings about C. results in D. stands for 47. With more members signing up, our recreation club needs more ______. A. sporting equipment B. sports equipment C. sports equipments D. sport equipments 48. He is such a strong-willed person that there is nothing in the world that can ______ him. A. win B. defeat C. rescue D. attach 49. She liked shopping but she was not in the ______ for it because she was a little exhausted. A. mood B. mind C. state D. spirit 50. Go back to the factory on time, or you will be ______. A. hired B. rented C. employed D. fired 51. I would much ______ it if you could help me carry the luggage. 143 A. appreciate B. apply C. admit D. achieve 52. It has been two years since her brother was ______ with her parents. A. on contract B. in contact C. keep contact D. come into contact 53. One must obtain ______ from the landowner to hunt in this forest. A. admiration B. requirement C. permission D. association 54. The pill my mother gave me ______ my toothache. A. regained B. restricted C. relieved D. released 55. He is in ______ about the job he is going to lose. A. despair B. regret C. defeat D. risk 56. The famous actress tried hard to answer any ______ questions about her unhappy marriage. A. embarrassed B. embarrassing C. emergent D. urgent 57. The young man fought to save a drowning girl. He was ______ admiration. A. worthy of B. worthwhile C. worth of D. worth 58. My father has many friends because he has a warm and ______ personality. A. outside B. outlook C. outstanding D. outgoing 59. I don’t like that fellow. He always talks with ______ of self-importance. A. an appearance B. a face C. an air D. a color 60. It was ______ of him to leave the classroom when the teacher criticized him. A. childlike B. childish C. childhood D. childless Part III Cloze(共20小题,每小题1分,共20分) Directions: There are 20 blanks in the following passage. For each blank there are 4 choices marked A, B, C and D. You should choose the ONE that best fits into the context. Then mark the corresponding letter on the Answer Sheet with a single line through the center. 144 Many people wrongly believe that when people reach old age, their families place them in nursing homes. They are left in the ___61___ of strangers for the rest of their lives. Their ___62___ children visit them only occasionally, but more often, they do not have any ___63___ visitors. The truth is that this idea is an ___64___ story. In fact, family members provide over 80 percent of the care ___65___ elderly people need. Samuel Prestoon, a sociologist, studied ___66___ the American family is changing. He reported that by the time the ___67___ American couple reaches 40 years of age, they have more parents than children. ___68___, because people today live longer after an illness than people did years ___69___, family members must provide long term care. More psychologists have found that all caregivers ___70___ a common characteristic: All caregivers believe that they are the best ___71___ for the job. In other words, they all felt that they ___72___ do the job better than anyone else. Social workers ___73___ caregivers to find out why they took ___74___ the responsibility of caring for an elderly relative. Many caregivers believed they had a(n) ___75___ to help their relative. Some stated that helping others ___76___ them feel more useful. Others hope that by helping ___77___ now, they would deserve care when became old and ___78___. Caring for the elderly and being taken care of can be a ___79___ satisfying experience for everyone who might be ___80___. 61. A. hands B. arms C. bodies D. homes 62. A growing B. grown C. being grown D. having grown 63. A. constant B. lasting C. regular D. normal 64. A. imaginary B. imaginable C. imaginative D. imagery 65. A. that B. this C. what D. who 66. A. when B. how C. what D. where 67. A. common B. regular C. standard D. average 68. A. Further B. However C. Moreover D. Whereas 69. A. before B. ago C. later D. lately 70. A. share B. enjoy C. divide D. consent 71. A. person B. people C. character D. man 72. A. would B. will C. could D. can 73. A. questioned B. interviewed C. inquired D. informed 74. A. in B. up C. on D. off 75. A. admiration B. action C. necessity D. duty 76. A. caused B. enabled C. made D. got 77. A. someone B. anyone C. everyone D. anybody 78. A. elderly B. dependent C. dependable D. independent 79. A. similarly B. differently C. mutually D. certainly 80. A. involved B. excluded C. included D. considered Part IV Translation(共35分) Section A (共5小题,每小题4分,共20分) Directions: Translate the following sentences into Chinese. You may refer to the corresponding passages in Part I. 81. Before entering a house in some Asian countries, it is good manners to take off your shoes. 82. We should like to find out the customs of other races, so that they will not think us ill-mannered. 83. He read a play of mine and, though he did not much like it, he thought it might just run for the six weeks till the play he had in mind could be produced 145 84. Brazil is now the world’s largest grower of coffee, and coffee is an important crop in Columbia and other South American countries. 85. Now, however, there are people who seek an immediate thrill from a risky activity which may only last a few minutes or even seconds. Section B (共5小题,每小题3分,共15分) Directions: Translate the following sentences into English. 86. 因为下大雪,足球比赛推迟到下周日举行。 87. 我把电话号码给你,以便你到达这里后和我联系。 88. 你必须每天练习讲英语,只有这样你才能提高英语口语水平。 89. 遗憾的是并非所有人在与陌生人交往时都感到自在。 90. 作为一名教师,你必须调整教学方法,以满足学生的需要。 Part V Writing (15分) Directions: For this part,you are allowed 30 minutes to write an essay with the title On Addiction to E-books. You are required to write at least 120 words,following the outline given below: 1. 很多大学生沉迷于阅读电子书籍; 2. 过度沉迷带来的危害; 3. 你的建议。 ? 以下部分为非专业试卷三年级与二年级不同内容 ? Part III Cloze(共20小题,每小题1分,共20分) Directions: There are 20 blanks in the following passage. For each blank there are 4 choices marked A, B, C and D. You should choose the ONE that best fits into the context. Then mark the corresponding letter on the Answer Sheet with a single line through the center. One day a police officer managed to get some fresh mushrooms. He was so ___61___ what he had bought that he offered to ___62___ the mushrooms with his brother officers. When their breakfast arrived the next day, each officer found some mushrooms on his place. ―Let the dog ___63___ a piece first,‖ suggested one ___64___ officer who was afraid that the mushrooms might be poisonous. The dog seemed to ___65___ his mushrooms, and the officers then began to eat their meal saying that the mushrooms had a very strange ___66___ quite pleasant taste. An hour ___67___, however, they were all astonished when the gardener rushed in and said ___68___ the dog was dead. ___69___, the officers jumped into their cars and rushed into the nearest hospital. The officers had a very ___70___ time getting rid of the mushrooms that ___71___ in their stomachs. When they ___72___ to the police station, they sat down and started to ___73___ the mushroom poisoning. Each man explained the pains that he had felt and they ___74___ that these had grown worse on their ___75___ to the hospital. The gardener was called to tell ___76___ the poor dog had died. ―Did it ___77___ much before death?‖ asked one of the officers, ___78___ very pleased that he had ___79___ a painful death himself. ―No,‖ answered the gardener. He looked rather ___80___: ―It was killed the moment a car hit it.‖ 61. A. sure of B. careless about C. pleased with D. disappointed at 62. A. share B. grow C. wash D. cook 146 63. A. check B. smell C. try D. examine 64. A. frightened B. shy C. cheerful D. careful 65. A. refuse B. hate C. want D. enjoy 66. A. besides B. but C. and D. or 67. A. later B. after C. past D. over 68. A. cruelly B. curiously C. seriously D. finally 69. A. Immediately B. Carefully C. Suddenly D. Slowly 70. A. hard B. busy C. exciting D. unforgettable 71. A. stopped B. dropped C. settled D. remained 72. A. hurried B. drove C. went D. returned 73. A. study B. discuss C. record D. remember 74. A. made B. agreed C. cared D. replied 75. A. road B. street C. way D. direction 76. A. how B. in that C. which D. in which 77. A. suffer B. eat C. harm D. spit 78. A. to feel B. feeling C. felt D. having felt 79. A. attacked B. escaped C. had D. caught 80. A. happy B. interested C. surprised D. excited Part IV Translation(共35分) Section A (共5小题,每小题4分,共20分) 84. In London, shops where chocolate drinks were served became important meeting places 85. Other activities as risky as bungee jumping involve jumping from tall buildings and diving into the sea from the top of high cliffs. Section B (共5小题,每小题3分,共15分) 88. 无论你怎么说,我也不会放弃这个计划。 89. 这篇作文写得很好,只是有些拼写错误。 90. 信不信由你,正是一个好心人的帮助才使他完成了大学学业。 Part V Writing(15分) Directions: For this part,you are allowed 30 minutes to write an essay with the title My Views on Net Chatting. You are required to write at least 120 words,following the outline given below: 1. 大学生中网聊的现象越来越普遍; 2. 网聊对大学生的影响; 3. 你对网聊的看法。 参考答案: 1-5AACDB 6-10CBACC 11-15ACDCA 16-20DACCC 21-25CDCDB 26-30ABACC 31-35BDBCA 36-40CDADC 41-45BADBD 46-50ADBAD 51-55ABCDA 56-60BADCB 60-65CACDD 66-70BACAA 71-75DDBBC 76-80DABBC 81. 在某些亚洲国家,进入房子之前脱鞋是很有礼貌的。 82. 我们应该了解别的国家的风俗习惯,这样的话他们会认为我们是有礼貌的。 83. 他曾读过我的一个剧本,尽管他对此不是很中意。他觉得让这个剧本先上映六周知道他心仪的剧本随147 后能写出来 84. 在伦敦,销售巧克力饮品的商店已然成为重要的聚会场所。 85. 其他的和蹦极一样冒险的极限活动有很多,包括从很高的建筑物上往下跳,从陡峭的悬崖下俯冲到海 里 86. Because of the heavy snow, the football match will be put off/postponed till next Sunday. 87. I give you my telephone number, so that you can contact me when you arrive here. 88. No matter what you say, I will never give up this plan. 89. This composition/writing is well-written except for some spelling mistakes. 90. Believe it or not, it is a warm-hearted man's help that enables him to finish his college study. Part V. 作文: My Views on Net Chatting With the rapid development of our modern society, an increasing number of college students are attracted by the net chatting. Indeed, almost all the college students have their own computers. Compared with former college students, modern college students have easier access to internet by computers or mobile phones. Moreover, they have more chatting friends on the internet than that of in reality. Net chatting has exerted several great impact on college students as following. In the first place, a growing number of students are lost in net chatting and they spend more time on chatting instead of studying hard. In the second place, internet is a virtual world. Students who love online chatting will be easily cheated by some evil persons from the internet. Last but not the least, by chatting on the internet, students are more likely to lose the ability to make friends in reality than as usual. As far as I am concerned, I maintain that we should keep ourselves away from net chatting. Admittedly, chatting on the internet is easy and convenient. However, it will take up much of our study time. As students, we have more healthy way of meeting new friends and more interesting activities to take part in. Be aware of that, we are the master of ourselves 2013年江苏省普通高校“专转本”统一考试试卷 大学英语 Part I Reading Comprehension (共20小题~每小题2分~共40分) Directions: There are 4 passages in this part. Each passage is followed by some questions or unfinished statements. For each of them there are 4 choices marked A, B, C and D. You should decide on the best choice and mark the corresponding letter on the Answer Sheet with a single line through the center. Passage One Questions 1 to 5 are based on the following passage. When Mrs. Joseph Groeger died recently in Vienna, Austria, people asked the obvious question. ―Why did she live to be 107?‖ Answers were provided by a survey conducted among 148 Viennese men and women who had reached the age of 100. What was surprising was that the majority had lived most of their lives in cities. Although cities are often regarded as unhealthy places, city living provides benefits that country living may lack. One factor seems to be important to the longevity(长寿) of those interviewed. This factor is exercise. In the cities it is often faster to walk short distances than to wait for a bus. Even taking public transportation often requires some walking. Smaller apartment houses 148 have no elevators, and so people must climb stairs. City people can usually walk to local supermarkets. Since parking spaces are hard to find, there is often no choice but to walk. On the other hand, those who live in the country and suburbs do not have to walk every day. In fact, the opposite is often true. To go to school, work, or almost anywhere else, they must ride in cars. 1. The Vienna survey may help to explain __________. A. the complaints of people in apartment houses B. the causes of Mrs. Groeger’s death C. the longevity of people like Mrs. Groeger D. the image of cities in general 2. The second paragraph lists some ___________. A. disadvantage of walking B. occasions for walking in city life C. problems of city living D. comments made by city people 3. To reach the third floor of a building, it would probably be most healthful ___________. A. to take the elevator B. to walk up the stairs C. to ride in a car D. to find an alternative walking 4. People who live in the country usually do more driving than walking because ___________. A. they don’t live near business areas B. they don’t need exercise C. they don’t like walking D. they can’t afford to take the bus 5. We can draw a conclusion from this passage that _________. A. air pollution is not serious B. anyone can live to be 107 C. country people should move to cities D. walking is a healthful exercise Passage Two Questions 6 to 10 are based on the following passage. The Bedouin people, a tribe living in the deserts of Arabia and Northern African, think most highly of people who show loyalty. To them loyalty does not mean that one is devoted to a country, a place, or a leader. Loyalty means being faithful to one’s family and tribe. The Bedouin people are proud of their ancestors(祖先). They do not admire a hero from an ordinary or poor family as much as one who comes from an honored family. They especially respect those who have inherited(继承) a good family name and then have passed it on to their children. A man’s position among his people depends upon his ancestors, relatives, and fellow tribesmen. If they are honored, he is also honored. If they are disagreed, he, too, is disgraced. Therefore, one carefully guards the honor of his family and his tribe. A man can protect his family’s honors by being brave and generous and by giving protection 149 to those who ask for it. He also guards it by carefully watching the women of his family. A Bedouin woman cannot bring honor to her family, but she can bring disgrace. Even if a woman only looks as if she has done something wrong, she may be killed. The honor of her family depends upon her virtue(美德). 6. The best title for this passage can be __________. A. Life of the Bedouin People B. Women’s Position in the Bedouin Family C. Bedouin People’s Idea of Loyalty and Honor D. How to Guard Honors 7. The Bedouin people think most highly of one who is ______. A. devoted to his country and his leader B. from an ordinary family C. proud of his ancestor and relatives D. faithful to his family and tribe 8. It is learned from the passage that ___________. A. a hero from an honored family is more admired B. one can not live in the tribe without a good family name C. all the Bedouin people are kind, brave and generous D. family traditions can never be passed to children 9. The word ―disgraced‖ (Para. 3) means ― __________‖. A. disappeared B. disappointed C. dishonored D. discouraged 10. It is implied in the passage that the Bedouin women are __________. A. treated as the equals of men B. respected for what they do C. not interested in position D. not respected as much as men Passage Three Questions 11 to 15 are based on the following passage. Up until about 100 years ago, newspaper in the United States attracted only the most serious readers. They used no illustrations (插图) and the articles were about politics or business. Two men changed that---Joseph Pulitzer of the New York World and William Randolph Hearst of the New York Journal. Pulitzer bought the New York World in 1883. He changed it from a traditional newspaper into a very exciting one overnight. He added lots of illustrations and he told his reporters to write articles on every crime they could find. And they did. One woman reporter even pretended she was mad and was sent to a hospital. She then wrote many articles about the poor treatment of patients in those hospitals where madmen were kept. In 1895, Hearst came to New York from California. He wanted the Journal to be more exciting than the World. He also wanted it to be cheaper, so he lowered the price by a penny. Hearst attracted attention because his newspaper heading were bigger than any one else’s. He often said, ―Big print makes big news.‖ Pulitzer and Hearst did anything they could to sell newspaper. For example, Hearst sent 150 Frederic Remington, the famous illustrator, to draw pictures of the Spanish-American War. When he got there, he told Hearst that no fighting was going on. Hearst answered, ― You provide the pictures. I’ll provide the war. 11. About a century ago, American newspaper carried news about __________. A. all kinds of exciting news B. crimes and mad people C. the poor treatment of patients D. serious matters only 12. According to Paragraph 2, which of the following statements is TRUE? A. Pulitzer and a woman news reporter changed the world overnight. B. Pulitzer bought the Journal with the help of his reporter. C. A woman news reporter pretended to be mad. D. A woman reporter became mad and was sent to a hospital. 13. Hearst attracted readers’ attention by ______________. A. using bigger headings B. reporting big war news C. raising the price of the newspaper D. reporting politics and business only 14. __________ was asked to draw war pictures. A. Hearst B. Remington C. Pulitzer D. A woman reporter 15. This passage tells us that ________. A. Pulitzer and Hearst were the most famous reporters of the United States B. newspaper owners sometimes hired mad men as their news reporters C. the world and the journal were not popular newspaper at that time D. Pulitzer and Hearst used every possible means to win over each other Passage Four Questions 16 to 20 are based on the following passage. It is hard to imagine life without Arabic numbers(数字). No other number system ever invented has a simple way to write a number like 1984. In Roman numeral, it comes out like this MCMLXXXIV. Imagine how difficult it would be for a school pupil to remember it! The Arabs, however, call their numbers ―Indian numerals‖. This is probably because they got the original idea a long time ago from India. There were no printed newspapers or scientific magazine then, so mathematics traveled along the same routes that silk did, as businessmen sold and bought goods. The Arabs took the new numbers and made improvements that quickly led to advances in technology. th The achievement of Arabic technology can be seen from a passage in a 10-century book about an Arabic city:‖ The neighboring country showed the results of careful work in agriculture, trade, and industry. Rice and many other plants were grown in fields with water supplied by Arab engineers. The city included 900 public baths, stone-covered streets, fountains, and beautiful buildings.‖ At this time, London was a small town with dirty streets, and Berlin was a farming 151 village. In fact, cities outside the Arab world did not reach the same level of social development until centuries later. 16. This passage tells us that the Arabs were a people _______________. A. inventing Roman numerals B. having advanced technology long ago C. growing corn and many other plants then D. supplying other countries with colorful silk 17. It takes much more efforts to ___________. A. learn Roman numerals than Arab numerals B. introduce numerals through printed materials C. make great use of silk roads D. teach businessmen numerals 18. The Arabs call their numerals ― Indian numerals‖ because _________. A. the numerals are mainly used in China B. they copied numerals from Indians C. Arabic numerals were first used by Indian businessmen D. they got some ideas about number from the ancient Indians 19. According to the passage, the Arab people ___________. A. did business with the Indians B. learned some ideas about business from Indians thC. built public baths in London in the 10 century D. traveled a long way to learn mathematics in Rome th20. From a 10-century book, we can learn that ___________. A. Berlin was formerly a dirty with sufficient rice and plants thB. London was under the rule of the Romans in the 10 century C. the Arab world was more advanced than other countries around D. London chiefly developed industry ten centuries ago Part II Vocabulary and Structure ,共40小题~每小题1分~共40分, Directions: There are 40 incomplete sentences in this part. For each sentence there are 4 choices marked A, B, C and D. Choose the ONE that best completes the sentence. Then mark the corresponding letter on the Answer Sheet with a single line through the center. 21. Jessica ought to stop working; she has a headache because she __________ too long. A. reads B. is reading C. has been reading D. had read 22. __________ us harm or good remains to be seen in the near future. A. If it will do B. Whether it will do C. That it will do D. What it will do 23. Able-bodied people just don’t realize how difficult it is _________ good jobs. A. of disabled people to get B. of disabled people getting C. for disabled people to get D. for disabled people getting 24. I’ll go camping with you if I ___________ free tomorrow. A. am B. will be C. being D. was 25. After the concert, the rock music star ___________ hundreds of fans outside the theater. A. surrounded by B. was surrounded by 152 C. being surrounded by D. is surrounded by 26. That book is really worth _________ if you’re going to take the final examination. A. to read B. reading C .to be read D. being read 27. __________ surprised me most was _________ such a little boy of seven could play the violin so well. A. That; what B. That; what C. What; that D. What; what 28. Staying in a hotel cost ________ renting a room in an apartment for a week. A. twice many than B. twice as much as C. twice the same as D. twice as many as 29. It is ________ that we’d like to go out for a walk. A. a lovely day B. too lovely a day C. so lovely a day D. such lovely a day 30. I can’t find my iPhone 5 in the dormitory. It __________ by somebody. A. must have taken away B. may have taken away C. may take away D. must have been taken away 31. When Mr. Smith got there, he found that about ___________ of his friends had been incited. A. twenty-third B. two-third C. two-thirds D. twenty-thirds 32. In 2012, Mo Yan became the first Chinese writer _________ won the Nobel Prize for Literature. A. that B. whom C. whose D. which 33. I didn’t see you at the meeting. If you had come, you _________ our sales manager. A. would meet B. would have met C. will have met D. had met 34. Ladies and gentlemen! It’s my pleasure __________ Mr. Li, our new Premier to you all. A. introducing B. introduce C. to be introducing D. to introduce 35. The audience burst into applause ___________ the singer finished her song. A. after moment B. in a moment C. the moment D. for a moment 36. ― Where is the girl we talked about yesterday?‖ ―Look! Here ___________.‖ A. she comes B . comes she C. she came D. came she 37. The number of employees at the factory __________ to a minimum so as to lower production costs. A. is cutting B. are cutting C. has been cut D. have been cut 38. Many young people nowadays make _________ a rule to buy flower and chocolate for their lovers on Valentine’s Day. A. themselves B. it C. them D. this 39. Dr. Green, together with his wife and two daughters, __________ to arrive in Nanjing tonight. A. will be B. is C .are D. are going 40. His father has been suffering from lung cancer for half a year, so the doctor Suggest that he ___________ an operation as soon as possible. A. takes B. would take C. took D. should take 41. Being out of work, Jane can no longer __________ her friends to dinners as she used to. A. urge B. treat C. attract D. apply 42. It’s a great pity that the girl _________ halfway and dropped out of the competition. A. gave off B. gave out C. gave over D. gave up 43. The fact that something is cheap doesn’t _________ mean it is of low quality. 153 A. especially B. practically C. necessarily D. specially 44. In learning English, many students ___________ an e-dictionary to a paper one. A. like B. expect C. prefer D. respect 45. The personnel manager will __________ the applications and pick out the best. A. look up B. look after C. look out D. look through 46. Seventy percent approved ________ the way she handled the situation while thirty percent disapproved. A. with B. about C. of D. up 47. The price of houses goes up again, partly _________ the requirement of young people. A. in result of B. with C. because D. due to 48. They decided to ___________ the contract because a number of the conditions had not been met. A. attend B. argue C. control D. cancel 49. __________ your English gets better, you’ll begin to feel more comfortable studying in Canada. A. Once B. As far as C. Whereas D. For 50. Although I told him to be careful over and over again, he never took any__________ of what I said. A. attention B. warning C. notice D. observation 51. Freshmen are usually __________ about everything when they first come to the college. A. troublesome B. curious C. different D. familiar 52. Salina came home so thin and weak that her family could hardly ________ her. A. remind B. recognize C. remember D. understand 53. The manager explained that they didn’t __________ people to bring theirs pets into the restaurant. A. allow B. let C. make D. have 54. ___________ I go, I always seem to bump into him. A. Whatever B. Wherever C. However D. Somehow 55. Our working hours are _________, we can go to work in the morning or in the afternoon. A. floating B. efficient C. flexible D. personal 56. To a great __________, the way you learn at college is different from that when you were in high school. A. amount B. extent C. reason D. goal 57. A skilled tennis player knows how to __________ his competitor’s weakness so as to get the final victory. A. beat B. take risk of C. defeat D. take advantage of 58. Try this camera, you’ll find it can be ___________ to take pictures in cloudy conditions. A. adjusted B. treated C. adopted D. removed 59. Being short of food and fuel, we are _________ with a very difficult situation. A. handled B. made C. faced D. encouraged 60. John was late for class again, but he couldn’t _________ his lateness. A. count for B. account for C. count on D. account on Part III Cloze,共20小题~每小题1分~共20分, 154 Directions: There are 20 blanks in the following passage. For each blank there are 4 choices marked A, B, C and D. You should choose the ONE that best fits into the context. Then mark the corresponding letter on the Answer Sheet with a single line through the center. Smoking is considered dangerous to the health. Our tobacco-seller Mr. Johnson, therefore always asking his customers if they are very young, whom the cigarettes are bought __61__. One day, a little girl whom he had never seen before walked boldly __62__ his shop and demanded twenty cigarettes. She had the large __63__ of money in her hand and seemed very ___64___ herself. Mr. Johnson was so __65___ by her confident manner that he __66___ to ask his usual question. ___67___, he asked her what kind of cigarettes she wanted. The girl ___68___ readily and handed him the money. While he was giving her the __69___, Mr. Johson said laughingly that as she was so __70___ she should ___71___ the packet(小盒) in her pocket in __72___ a policeman saw it. However, the little girl did not seem to ___73____ this very funny. Without ___74__ she took the __75___ and walked towards the door. Suddenly she stopped, turned __76___ and looked steadily at Mr. Johnson. __77___ was a moment of silence and the tobacco-seller wondered ___78____ she was going to say. And at once, in a clear, joking ___79__, the girl declared, ―___80__ dad is a policeman,‖ and with that she walked quickly out of the shop. 61. A. with B. to C. by D. for 62. A. with B. for C. into D. down 63. A. amount B. quality C. bits D. part 64. A. since B. of C. without D. against 65. A. worried B. interested C. surprised D. pleased 66. A. forgot B. came C. went D. dreamed 67. A. Therefore B. Instead C. Somehow D. Anyhow 68. A. Replied B. Screamed C. spoke D. talked 69. A. change B. warning C. cheque D. advice 70. A. old B. young C. amazed D. excited 71. A. hide B. throw C. tip D. take 72. A. way B. peace C. case D. end 73. A. search B. get C. pass D. find 74. A. look B. looked C. smiling D. smiled 75. A. packet B. clothes C. knife D. bill 76. A. way B. over C. round D. aside 77. A. They B. She C. There D. He 78. A. where B. what C. when D. if 79. A. words B. voice C. sound D. statement 80. A. The B. For C. As D. My Part IV Translation(共35分) Section A (共5小题~每小题4分~共20分) Directions: Translate the following sentences into Chinese. You may refer to the corresponding passages in Part I. 81. Although cities are often regarded as unhealthy places, city living provides benefits that country living may lack. (Passage One) 82. A man’s position among his people depends upon his ancestors, relatives, and fellow tribesmen. 155 (Passage Two) 83. She then wrote many articles about the poor treatment of patients in those hospitals where madmen were kept. (Passage Three) 84. The Arabs took the new numbers and made improvements that quickly led to advances in technology. (Passage Four) 85. In fact, cities outside the Arab world did not reach the same level of social development until centuries later. (Passage Four) Section B (共5小题~每小题3分~共15分) Directions: Translate the following sentences into English. 86. 根据以往的经验,他可能赢得这场比赛。 87. 不管发生什么事,我们都不应该嘲笑犯错误的人。 88. 他没有出现在晚会上,毕竟他是个非常忙碌的人。 89. 你应该把时间和精力放在解决问题上,而不是抱怨。 90. 就结构而言,这篇文章相当不错。 Part V Writing (15分) Direction: For this part, you are allowed 30 minutes to write an essay with the title My View on Mobile Phone. You are required to write at least 120 words, following the outline given below: 1. 平时你用手机做什么; 2. 假设没有手机,你会做什么; 3. 如何合理地使用手机 大三考试真题 Part III Cloze,共20小题~每小题1分~共20分, Directions: There are 20 blanks in the following passage. For each blank there are 4 choices marked A, B, C and D. You should choose the ONE that best fits into the context. Then mark the corresponding letter on the Answer Sheet with a single line through the center. If you travel around the world, you will be surprised to find that foreign customs can be different __61_ your own. A visitor to India would do well to remember that people there __62_ it impolite to use the left hand __63__ passing food at table. The left hand is supposed to be __64__ for washing yourself. Also in India, you might __65___ a man apparently shaking his head at another and __66__ that he is disagreeing. But in many __67___ of India a shake of the head is a __68__ that shows agreement or acceptance. Nodding your head when you are offered a __69___ in Bulgaria is likely to leave you thirsty. In that country you shake your head to indicate ―yes‖--- a nod indicates ―no‖. The Arabs are ___70___ for their hospitality(好客). At a meal in the Arabic countries, you will __71___ that any drinking vessel(器皿) is filled again __72___ again as soon as you drain it. The way to ___73___ that you have had enough is to take the cup or glass in your hand and give it a little shake from side to side or place your hand __74___ the top. In Europe it is quite usual to __75___ your legs when sitting talking to someone, even at an important meeting. Doing this when meeting an important person in Thailand, however, could __76___ him or her unhappy. It is considered too informal an attitude for such __77___ occasion. Also when in Thailand you are __78___ supposed to touch the head of an adult----it’s just not 156 done. In Japan, it is quite usual for __79___ to plan evening entertainment for themselves and leave their wives at home. In Europe such attitudes are disappearing. Customs vary from country to __80___. Visitors ma be at a loss as to what to do in a foreign environment. In such circumstances, the rule is: When in Rome, do as the Romans do. 61. A. from B. away C. within D. in 62. A. achieve B. consider C. test D. help 63. A. for B. on C. down D. up 64. A. noticed B. used C. bought D. used 65. A. listen B. hope C. miss D. see 66. A. neglect B. think C. ignore D. inspect 67. A. countries B. people C. parts D. hills 68. A. spot B. picture C. sign D. mean 69. A. drink B. pen C. hat D. hand 70. A. interested B . terrible C. careful D. famous 71. A. examine B. watch C. find D. receive 72. A. and B. till C. also D. still 73. A. discuss B. turn C. show D. forget 74. A. in B. below C. over D. under 75. A. cross B. put C. take D. hurt 76. A. pay B. stop C. make D. point 77. A. too B. the C. a D. an 78. A. not B. nor C. none D. no 79. A. men B. women C. boys D. girls 80. A. area B. place C. country D. apace Part IV Translation(共35分) Section A (共5小题~每小题4分~共20分) Directions: Translate the following sentences into Chinese. You may refer to the corresponding passages in Part I. 81. Since parking spaces are hard to find, there is often no alternative to walking.(Passage One) 82. A man can protect his family’s honors by being brave and generous and by giving protection to those who ask for it. (Passage Two) Section B (共5小题~每小题3分~共15分) Directions: Translate the following sentences into English. 86. 近年来,网络语言深得青年人的喜爱。 87. 他很可能听说过这个名字,但现在想不起来了。 88. 只有这样,你才能掌握一门语言的基本词汇。 157 158
本文档为【英语真题】,请使用软件OFFICE或WPS软件打开。作品中的文字与图均可以修改和编辑, 图片更改请在作品中右键图片并更换,文字修改请直接点击文字进行修改,也可以新增和删除文档中的内容。
该文档来自用户分享,如有侵权行为请发邮件ishare@vip.sina.com联系网站客服,我们会及时删除。
[版权声明] 本站所有资料为用户分享产生,若发现您的权利被侵害,请联系客服邮件isharekefu@iask.cn,我们尽快处理。
本作品所展示的图片、画像、字体、音乐的版权可能需版权方额外授权,请谨慎使用。
网站提供的党政主题相关内容(国旗、国徽、党徽..)目的在于配合国家政策宣传,仅限个人学习分享使用,禁止用于任何广告和商用目的。
下载需要: 免费 已有0 人下载
最新资料
资料动态
专题动态
is_654168
暂无简介~
格式:doc
大小:983KB
软件:Word
页数:364
分类:生活休闲
上传时间:2018-10-06
浏览量:52